УскорСниС Ρ‡Π΅Ρ€Π΅Π· радиус: Π€ΠΎΡ€ΠΌΡƒΠ»Π° Ρ†Π΅Π½Ρ‚Ρ€ΠΎΡΡ‚Ρ€Π΅ΠΌΠΈΡ‚Π΅Π»ΡŒΠ½ΠΎΠ³ΠΎ ускорСния Π² Ρ„ΠΈΠ·ΠΈΠΊΠ΅

Π‘ΠΎΠ΄Π΅Ρ€ΠΆΠ°Π½ΠΈΠ΅

Π“Π»Π°Π²Π° 7. Π’Ρ€Π°Ρ‰Π°Ρ‚Π΅Π»ΡŒΠ½ΠΎΠ΅ Π΄Π²ΠΈΠΆΠ΅Π½ΠΈΠ΅. ΠšΠΈΠ½Π΅ΠΌΠ°Ρ‚ΠΈΠΊΠ° ΠΈ Π΄ΠΈΠ½Π°ΠΌΠΈΠΊΠ°

Как ΠΏΡ€Π°Π²ΠΈΠ»ΠΎ, Π² любом Π²Π°Ρ€ΠΈΠ°Π½Ρ‚Π΅ задания Π•Π“Π­ ΠΏΠΎ Ρ„ΠΈΠ·ΠΈΠΊΠ΅ прСдставлСны нСсколько Π·Π°Π΄Π°Ρ‡ Π½Π° Π²Ρ€Π°Ρ‰Π°Ρ‚Π΅Π»ΡŒΠ½ΠΎΠ΅ Π΄Π²ΠΈΠΆΠ΅Π½ΠΈΠ΅. ΠŸΡ€ΠΈΠ²Π΅Π΄Π΅ΠΌ основныС опрСдСлСния ΠΈ Π·Π°ΠΊΠΎΠ½Ρ‹, Π½Π΅ΠΎΠ±Ρ…ΠΎΠ΄ΠΈΠΌΡ‹Π΅ для Ρ€Π΅ΡˆΠ΅Π½ΠΈΡ Ρ‚Π°ΠΊΠΎΠ³ΠΎ Ρ€ΠΎΠ΄Π° Π·Π°Π΄Π°Ρ‡. Π£Π³Π»ΠΎΠ²ΠΎΠΉ ΡΠΊΠΎΡ€ΠΎΡΡ‚ΡŒΡŽ Ρ‚Π΅Π»Π°, ΡΠΎΠ²Π΅Ρ€ΡˆΠ°ΡŽΡ‰Π΅Π³ΠΎ Π²Ρ€Π°Ρ‰Π°Ρ‚Π΅Π»ΡŒΠ½ΠΎΠ΅ Π΄Π²ΠΈΠΆΠ΅Π½ΠΈΠ΅, называСтся ΠΎΡ‚Π½ΠΎΡˆΠ΅Π½ΠΈΠ΅ ΡƒΠ³Π»Π° ΠΏΠΎΠ²ΠΎΡ€ΠΎΡ‚Π° ΠΊ Ρ‚ΠΎΠΌΡƒ Π²Ρ€Π΅ΠΌΠ΅Π½ΠΈ , Π·Π° ΠΊΠΎΡ‚ΠΎΡ€ΠΎΠ΅ этот ΠΏΠΎΠ²ΠΎΡ€ΠΎΡ‚ ΠΏΡ€ΠΎΠΈΠ·ΠΎΡˆΠ΅Π»

(7.1)

Π’ этом ΠΎΠΏΡ€Π΅Π΄Π΅Π»Π΅Π½ΠΈΠΈ ΡƒΠ³ΠΎΠ» Π΄ΠΎΠ»ΠΆΠ΅Π½ ΠΈΠ·ΠΌΠ΅Ρ€ΡΡ‚ΡŒΡΡ Π² Ρ€Π°Π΄ΠΈΠ°Π½Π°Ρ…, поэтому Ρ€Π°Π·ΠΌΠ΅Ρ€Π½ΠΎΡΡ‚ΡŒ ΡƒΠ³Π»ΠΎΠ²ΠΎΠΉ скорости Ρ€Π°Π΄/с (ΠΈΠ»ΠΈ 1/с ΠΏΠΎΡΠΊΠΎΠ»ΡŒΠΊΡƒ Ρ€Π°Π΄ΠΈΠ°Π½ – бСзразмСрная Π²Π΅Π»ΠΈΡ‡ΠΈΠ½Π°). Π’ ΠΏΡ€ΠΈΠ½Ρ†ΠΈΠΏΠ΅, ΠΎΠΏΡ€Π΅Π΄Π΅Π»Π΅Π½ΠΈΠ΅ (7.1) позволяСт Π½Π°ΠΉΡ‚ΠΈ ΠΊΠ°ΠΊ ΡΡ€Π΅Π΄Π½ΡŽΡŽ (для Π±ΠΎΠ»ΡŒΡˆΠΈΡ… ΠΈΠ½Ρ‚Π΅Ρ€Π²Π°Π»ΠΎΠ² Π²Ρ€Π΅ΠΌΠ΅Π½ΠΈ ), Ρ‚Π°ΠΊ ΠΈ ΠΌΠ³Π½ΠΎΠ²Π΅Π½Π½ΡƒΡŽ (ΠΏΡ€ΠΈ ) ΡƒΠ³Π»ΠΎΠ²ΡƒΡŽ ΡΠΊΠΎΡ€ΠΎΡΡ‚ΡŒ. Однако Π² школьном курсС Ρ„ΠΈΠ·ΠΈΠΊΠΈ рассматриваСтся Ρ‚ΠΎΠ»ΡŒΠΊΠΎ Π΄Π²ΠΈΠΆΠ΅Π½ΠΈΠ΅ с постоянной ΡƒΠ³Π»ΠΎΠ²ΠΎΠΉ ΡΠΊΠΎΡ€ΠΎΡΡ‚ΡŒΡŽ, для ΠΊΠΎΡ‚ΠΎΡ€ΠΎΠ³ΠΎ ΠΎΠΏΡ€Π΅Π΄Π΅Π»Π΅Π½ΠΈΠ΅ (7.1) Π΄Π°Π΅Ρ‚ ΠΎΠ΄ΠΈΠ½ ΠΈ Ρ‚ΠΎΡ‚ ΠΆΠ΅ Ρ€Π΅Π·ΡƒΠ»ΡŒΡ‚Π°Ρ‚ для Π»ΡŽΠ±Ρ‹Ρ… ΠΈΠ½Ρ‚Π΅Ρ€Π²Π°Π»ΠΎΠ² Π²Ρ€Π΅ΠΌΠ΅Π½ΠΈ . ΠŸΡ€ΠΈΠΌΠ΅Π½ΡΡ ΠΎΠΏΡ€Π΅Π΄Π΅Π»Π΅Π½ΠΈΠ΅ (7.

1) ΠΊ ΠΏΠΎΠ»Π½ΠΎΠΌΡƒ ΠΎΠ±ΠΎΡ€ΠΎΡ‚Ρƒ Ρ‚Π΅Π»Π° (ΡƒΠ³ΠΎΠ» ΠΏΠΎΠ²ΠΎΡ€ΠΎΡ‚Π° – Ρ€Π°Π΄ΠΈΠ°Π½), ΠΏΠΎΠ»ΡƒΡ‡ΠΈΠΌ связь ΡƒΠ³Π»ΠΎΠ²ΠΎΠΉ скорости ΠΈ ΠΏΠ΅Ρ€ΠΈΠΎΠ΄Π° вращСния

(7.2)

Π£Π³Π»ΠΎΠ²ΡƒΡŽ ΡΠΊΠΎΡ€ΠΎΡΡ‚ΡŒ ΠΌΠΎΠΆΠ½ΠΎ ввСсти Π½Π΅ Ρ‚ΠΎΠ»ΡŒΠΊΠΎ для Ρ‚ΠΎΡ‡Π΅Ρ‡Π½ΠΎΠ³ΠΎ Ρ‚Π΅Π»Π°, Π½ΠΎ ΠΈ для протяТСнного Ρ‚Π΅Π»Π°. Π”Π΅ΠΉΡΡ‚Π²ΠΈΡ‚Π΅Π»ΡŒΠ½ΠΎ, ΠΏΡ€ΠΈ Π²Ρ€Π°Ρ‰Π΅Π½ΠΈΠΈ Π½Π΅Ρ‚ΠΎΡ‡Π΅Ρ‡Π½ΠΎΠ³ΠΎ Ρ‚Π΅Π»Π° Π²ΠΎΠΊΡ€ΡƒΠ³ любой оси всС Π΅Π³ΠΎ Ρ‚ΠΎΡ‡ΠΊΠΈ ΠΏΠΎΠ²ΠΎΡ€Π°Ρ‡ΠΈΠ²Π°ΡŽΡ‚ΡΡ Π·Π° ΠΎΠ΄ΠΈΠ½Π°ΠΊΠΎΠ²ΠΎΠ΅ врСмя Π½Π° ΠΎΠ΄ΠΈΠ½Π°ΠΊΠΎΠ²Ρ‹ΠΉ ΡƒΠ³ΠΎΠ». ΠŸΠΎΡΡ‚ΠΎΠΌΡƒ ΠΌΠΎΠΆΠ½ΠΎ Π³ΠΎΠ²ΠΎΡ€ΠΈΡ‚ΡŒ ΠΎΠ± ΡƒΠ³Π»ΠΎΠ²ΠΎΠΉ скорости всСго Ρ‚Π΅Π»Π°.

Из Ρ„ΠΎΡ€ΠΌΡƒΠ»Ρ‹ (7.2) Π»Π΅Π³ΠΊΠΎ ΠΏΠΎΠ»ΡƒΡ‡ΠΈΡ‚ΡŒ связь ΡƒΠ³Π»ΠΎΠ²ΠΎΠΉ ΠΈ ΠΎΠ±Ρ‹Ρ‡Π½ΠΎΠΉ скорости Π²Ρ€Π°Ρ‰Π°ΡŽΡ‰Π΅Π³ΠΎΡΡ Ρ‚ΠΎΡ‡Π΅Ρ‡Π½ΠΎΠ³ΠΎ Ρ‚Π΅Π»Π° (Π² этом контСкстС послСднюю всСгда Π½Π°Π·Ρ‹Π²Π°ΡŽΡ‚ Π»ΠΈΠ½Π΅ΠΉΠ½ΠΎΠΉ ΡΠΊΠΎΡ€ΠΎΡΡ‚ΡŒΡŽ). УмноТая ΠΏΡ€Π°Π²ΡƒΡŽ ΠΈ Π»Π΅Π²ΡƒΡŽ Ρ‡Π°ΡΡ‚ΡŒ Ρ„ΠΎΡ€ΠΌΡƒΠ»Ρ‹ (7.2) Π½Π° радиус окруТности ΠΈ учитывая, Ρ‡Ρ‚ΠΎ – это Π΄Π»ΠΈΠ½Π° ΠΏΡƒΡ‚ΠΈ, ΠΏΡ€ΠΎΠΉΠ΄Π΅Π½Π½ΠΎΠ³ΠΎ Π·Π° ΠΏΠ΅Ρ€ΠΈΠΎΠ΄, ΠΏΠΎΠ»ΡƒΡ‡ΠΈΠΌ

(7.3)

ΠšΠΎΠ½Π΅Ρ‡Π½ΠΎ, для Π½Π΅Ρ‚ΠΎΡ‡Π΅Ρ‡Π½ΠΎΠ³ΠΎ Π²Ρ€Π°Ρ‰Π°ΡŽΡ‰Π΅Π³ΠΎΡΡ Ρ‚Π΅Π»Π° нСльзя ввСсти понятиС Π»ΠΈΠ½Π΅ΠΉΠ½ΠΎΠΉ скорости, ΠΏΠΎΡΠΊΠΎΠ»ΡŒΠΊΡƒ Ρƒ Ρ€Π°Π·Π½Ρ‹Ρ… Ρ‚ΠΎΡ‡Π΅ΠΊ этого Ρ‚Π΅Π»Π° Π»ΠΈΠ½Π΅ΠΉΠ½Ρ‹Π΅ скорости Π±ΡƒΠ΄ΡƒΡ‚ Ρ€Π°Π·Π½Ρ‹ΠΌΠΈ.

ΠžΡ‡Π΅Π²ΠΈΠ΄Π½ΠΎ, ΠΏΡ€ΠΈ Π²Ρ€Π°Ρ‰Π°Ρ‚Π΅Π»ΡŒΠ½ΠΎΠΌ Π΄Π²ΠΈΠΆΠ΅Π½ΠΈΠΈ Ρ‚Π΅Π»ΠΎ всСгда ΠΈΠΌΠ΅Π΅Ρ‚ ускорСниС. Π”Π΅ΠΉΡΡ‚Π²ΠΈΡ‚Π΅Π»ΡŒΠ½ΠΎ, согласно ΠΎΠΏΡ€Π΅Π΄Π΅Π»Π΅Π½ΠΈΡŽ (2.1) ускорСниС Ρ‚Π΅Π»Π° Ρ€Π°Π²Π½ΠΎ Π½ΡƒΠ»ΡŽ, Ссли Π½Π΅ мСняСтся Π²Π΅ΠΊΡ‚ΠΎΡ€ скорости этого Ρ‚Π΅Π»Π° (Ρ‚.Π΅. ΠΊΠ°ΠΊ Π²Π΅Π»ΠΈΡ‡ΠΈΠ½Π° скорости, Ρ‚Π°ΠΊ ΠΈ Π΅Π΅ Π½Π°ΠΏΡ€Π°Π²Π»Π΅Π½ΠΈΠ΅). ΠŸΡ€ΠΈ Π²Ρ€Π°Ρ‰Π°Ρ‚Π΅Π»ΡŒΠ½ΠΎΠΌ Π΄Π²ΠΈΠΆΠ΅Π½ΠΈΠΈ Π½Π°ΠΏΡ€Π°Π²Π»Π΅Π½ΠΈΠ΅ скорости ΠΎΠ±ΡΠ·Π°Ρ‚Π΅Π»ΡŒΠ½ΠΎ мСняСтся. МоТно Π΄ΠΎΠΊΠ°Π·Π°Ρ‚ΡŒ, Ρ‡Ρ‚ΠΎ ΠΏΡ€ΠΈ Π²Ρ€Π°Ρ‰Π°Ρ‚Π΅Π»ΡŒΠ½ΠΎΠΌ Π΄Π²ΠΈΠΆΠ΅Π½ΠΈΠΈ Ρ‚ΠΎΡ‡Π΅Ρ‡Π½ΠΎΠ³ΠΎ Ρ‚Π΅Π»Π° с постоянной ΠΏΠΎ Π²Π΅Π»ΠΈΡ‡ΠΈΠ½Π΅ Π»ΠΈΠ½Π΅ΠΉΠ½ΠΎΠΉ ΡΠΊΠΎΡ€ΠΎΡΡ‚ΡŒΡŽ Π²Π΅ΠΊΡ‚ΠΎΡ€ Π΅Π³ΠΎ ускорСния Π² любой ΠΌΠΎΠΌΠ΅Π½Ρ‚ Π½Π°ΠΏΡ€Π°Π²Π»Π΅Π½ ΠΎΡ‚ Ρ‚Π΅Π»Π° ΠΊ Ρ†Π΅Π½Ρ‚Ρ€Ρƒ Ρ‚Ρ€Π°Π΅ΠΊΡ‚ΠΎΡ€ΠΈΠΈ Ρ‚Π΅Π»Π°, Π° Π΅Π³ΠΎ Π²Π΅Π»ΠΈΡ‡ΠΈΠ½Π° Ρ€Π°Π²Π½Π°

(7.4)

УскорСниС (7.3) принято Π½Π°Π·Ρ‹Π²Π°Ρ‚ΡŒ Ρ†Π΅Π½Ρ‚Ρ€ΠΎΡΡ‚Ρ€Π΅ΠΌΠΈΡ‚Π΅Π»ΡŒΠ½Ρ‹ΠΌ. Если ΠΈΡΠΏΠΎΠ»ΡŒΠ·ΠΎΠ²Π°Ρ‚ΡŒ связь Π»ΠΈΠ½Π΅ΠΉΠ½ΠΎΠΉ ΠΈ ΡƒΠ³Π»ΠΎΠ²ΠΎΠΉ скорости Ρ‚Π΅Π»Π° ΠΏΡ€ΠΈ Π²Ρ€Π°Ρ‰Π°Ρ‚Π΅Π»ΡŒΠ½ΠΎΠΌ Π΄Π²ΠΈΠΆΠ΅Π½ΠΈΠΈ (7.3), Ρ‚ΠΎ Ρ„ΠΎΡ€ΠΌΡƒΠ»Ρƒ для Ρ†Π΅Π½Ρ‚Ρ€ΠΎΡΡ‚Ρ€Π΅ΠΌΠΈΡ‚Π΅Π»ΡŒΠ½ΠΎΠ³ΠΎ ускорСния ΠΌΠΎΠΆΠ½ΠΎ Π·Π°ΠΏΠΈΡΠ°Ρ‚ΡŒ ΠΈ Π² Ρ‚Π°ΠΊΠΈΡ… Ρ„ΠΎΡ€ΠΌΠ°Ρ…

(7.5)

Богласно Π²Ρ‚ΠΎΡ€ΠΎΠΌΡƒ Π·Π°ΠΊΠΎΠ½Ρƒ ΠΡŒΡŽΡ‚ΠΎΠ½Π° ускорСния ΡΠΎΠΎΠ±Ρ‰Π°ΡŽΡ‚ΡΡ Ρ‚Π΅Π»Π°ΠΌ силами. ΠŸΠΎΡΡ‚ΠΎΠΌΡƒ Ссли Ρ‚Π΅Π»ΠΎ ΡΠΎΠ²Π΅Ρ€ΡˆΠ°Π΅Ρ‚ Π΄Π²ΠΈΠΆΠ΅Π½ΠΈΠ΅ ΠΏΠΎ окруТности радиуса с постоянной ΠΏΠΎ Π²Π΅Π»ΠΈΡ‡ΠΈΠ½Π΅ ΡΠΊΠΎΡ€ΠΎΡΡ‚ΡŒΡŽ (ΠΈ соотвСтствСнно ΡƒΠ³Π»ΠΎΠ²ΠΎΠΉ ΡΠΊΠΎΡ€ΠΎΡΡ‚ΡŒΡŽ ), Π½Π° Π½Π΅Π³ΠΎ Π΄ΠΎΠ»ΠΆΠ½Π° Π΄Π΅ΠΉΡΡ‚Π²ΠΎΠ²Π°Ρ‚ΡŒ сила, направлСнная ΠΊ Ρ†Π΅Π½Ρ‚Ρ€Ρƒ окруТности ΠΈ равная ΠΏΠΎ Π²Π΅Π»ΠΈΡ‡ΠΈΠ½Π΅

(7.6)

Π‘ΠΈΠ»Ρƒ (7.6) принято Π½Π°Π·Ρ‹Π²Π°Ρ‚ΡŒ Ρ†Π΅Π½Ρ‚Ρ€ΠΎΡΡ‚Ρ€Π΅ΠΌΠΈΡ‚Π΅Π»ΡŒΠ½ΠΎΠΉ. ΠžΡ‚ΠΌΠ΅Ρ‚ΠΈΠΌ, Ρ‡Ρ‚ΠΎ Ρ‚Π΅Ρ€ΠΌΠΈΠ½ Β«Ρ†Π΅Π½Ρ‚Ρ€ΠΎΡΡ‚Ρ€Π΅ΠΌΠΈΡ‚Π΅Π»ΡŒΠ½Π°ΡΒ» связан Π½Π΅ с ΠΏΡ€ΠΈΡ€ΠΎΠ΄ΠΎΠΉ этой силы, Π° с Ρ‚Π΅ΠΌ, ΠΊΠ°ΠΊ ΠΎΠ½Π° дСйствуСт: Π² Ρ€Π°Π·Π½Ρ‹Ρ… ситуациях Ρ†Π΅Π½Ρ‚Ρ€ΠΎΡΡ‚Ρ€Π΅ΠΌΠΈΡ‚Π΅Π»ΡŒΠ½ΠΎΠΉ силой ΠΌΠΎΠΆΠ΅Ρ‚ Π±Ρ‹Ρ‚ΡŒ ΠΈ сила тяТСсти, ΠΈ сила трСния, ΠΈ сила Ρ€Π΅Π°ΠΊΡ†ΠΈΠΈ, ΠΈ Π΄Ρ€ΡƒΠ³ΠΈΠ΅ силы ΠΈΠ»ΠΈ ΠΈΡ… ΠΊΠΎΠΌΠ±ΠΈΠ½Π°Ρ†ΠΈΠΈ.

ΠŸΠ΅Ρ€Π΅Ρ‡ΠΈΡΠ»Π΅Π½Π½Ρ‹Ρ… Π·Π°ΠΊΠΎΠ½ΠΎΠ² ΠΈ ΠΎΠΏΡ€Π΅Π΄Π΅Π»Π΅Π½ΠΈΠΉ достаточно для Ρ€Π΅ΡˆΠ΅Π½ΠΈΡ Π»ΡŽΠ±Ρ‹Ρ… Π·Π°Π΄Π°Ρ‡ Π•Π“Π­ Π½Π° Π²Ρ€Π°Ρ‰Π°Ρ‚Π΅Π»ΡŒΠ½ΠΎΠ΅ Π΄Π²ΠΈΠΆΠ΅Π½ΠΈΠ΅. Рассмотрим ΠΈΡ… ΠΏΡ€ΠΈΠΌΠ΅Π½Π΅Π½ΠΈΠ΅ ΠΊ Ρ€Π΅ΡˆΠ΅Π½ΠΈΡŽ Π·Π°Π΄Π°Ρ‡, ΠΏΡ€ΠΈΠ²Π΅Π΄Π΅Π½Π½Ρ‹Ρ… Π² ΠΏΠ΅Ρ€Π²ΠΎΠΉ части.

Если ΠΏΠ΅Ρ€ΠΈΠΎΠ΄ вращСния Ρ‚Π΅Π»Π° Π·Π°Π΄Π°Π½, Ρ‚ΠΎ Π΅Π³ΠΎ угловая ΡΠΊΠΎΡ€ΠΎΡΡ‚ΡŒ ΠΌΠΎΠΆΠ΅Ρ‚ Π±Ρ‹Ρ‚ΡŒ ΠΎΠ΄Π½ΠΎΠ·Π½Π°Ρ‡Π½ΠΎ ΠΎΠΏΡ€Π΅Π΄Π΅Π»Π΅Π½Π° нСзависимо ΠΎΡ‚ Ρ€Π°Π·ΠΌΠ΅Ρ€ΠΎΠ² Ρ‚Π΅Π»Π° ΠΈΠ»ΠΈ радиуса ΠΎΡ€Π±ΠΈΡ‚Ρ‹ для Ρ‚ΠΎΡ‡Π΅Ρ‡Π½ΠΎΠ³ΠΎ Ρ‚Π΅Π»Π°. Π’ частности, сСкундная стрСлка Π»ΡŽΠ±Ρ‹Ρ… часов поворачиваСтся Π½Π° ΡƒΠ³ΠΎΠ» Π·Π° ΠΎΠ΄Π½Ρƒ ΠΌΠΈΠ½ΡƒΡ‚Ρƒ (ΠΊΠΎΠ½Π΅Ρ‡Π½ΠΎ, ΠΏΡ€ΠΈ условии, Ρ‡Ρ‚ΠΎ ΠΎΠ½ΠΈ ΠΈΠ΄ΡƒΡ‚ Β«ΠΏΡ€Π°Π²ΠΈΠ»ΡŒΠ½ΠΎΒ»). ΠŸΠΎΡΡ‚ΠΎΠΌΡƒ угловая ΡΠΊΠΎΡ€ΠΎΡΡ‚ΡŒ сСкундных стрСлок Π»ΡŽΠ±Ρ‹Ρ… часов Ρ€Π°Π²Π½Π° Ρ€Π°Π΄/ΠΌΠΈΠ½ (

Π·Π°Π΄Π°Ρ‡Π° 7.1.1 – ΠΎΡ‚Π²Π΅Ρ‚ 2).

Для нахоТдСния Π»ΠΈΠ½Π΅ΠΉΠ½ΠΎΠΉ скорости ΠΊΠΎΠ½Ρ†Π° сСкундной стрСлки часов (Π·Π°Π΄Π°Ρ‡Π° 7.1.2) ΠΈΡΠΏΠΎΠ»ΡŒΠ·ΡƒΠ΅ΠΌ связь ΡƒΠ³Π»ΠΎΠ²ΠΎΠΉ ΠΈ Π»ΠΈΠ½Π΅ΠΉΠ½ΠΎΠΉ скоростСй (7.5). ИмССм

(ΠΏΡ€Π°Π²ΠΈΠ»ΡŒΠ½Ρ‹ΠΉ ΠΎΡ‚Π²Π΅Ρ‚ – 2).

ΠŸΡ€ΠΈΠΌΠ΅Π½ΡΡ ΠΎΠΏΡ€Π΅Π΄Π΅Π»Π΅Π½ΠΈΠ΅ ΡƒΠ³Π»ΠΎΠ²ΠΎΠΉ скорости ΠΊ колСсу (Π·Π°Π΄Π°Ρ‡Π° 7.1.3), ΠΏΠΎΠ»ΡƒΡ‡Π°Π΅ΠΌ

(ΠΏΡ€Π°Π²ΠΈΠ»ΡŒΠ½Ρ‹ΠΉ ΠΎΡ‚Π²Π΅Ρ‚ 1).

Из Ρ„ΠΎΡ€ΠΌΡƒΠ»Ρ‹ (7.2) ΠΈΠΌΠ΅Π΅ΠΌ

(Π·Π°Π΄Π°Ρ‡Π° 7.1.4 – ΠΏΡ€Π°Π²ΠΈΠ»ΡŒΠ½Ρ‹ΠΉ ΠΎΡ‚Π²Π΅Ρ‚

4).

Π˜ΡΠΏΠΎΠ»ΡŒΠ·ΡƒΡ извСстноС расстояниС ΠΎΡ‚ ΠΏΠ΅Ρ€Π²ΠΎΠΉ Ρ‚ΠΎΡ‡ΠΊΠΈ Π΄ΠΎ оси вращСния ΠΈ Π΅Π΅ Ρ†Π΅Π½Ρ‚Ρ€ΠΎΡΡ‚Ρ€Π΅ΠΌΠΈΡ‚Π΅Π»ΡŒΠ½ΠΎΠ΅ ускорСниС (Π·Π°Π΄Π°Ρ‡Π° 7.1.5), ΠΈΠ· Ρ„ΠΎΡ€ΠΌΡƒΠ»Ρ‹ (7.5) Π½Π°Ρ…ΠΎΠ΄ΠΈΠΌ ΠΊΠ²Π°Π΄Ρ€Π°Ρ‚ ΡƒΠ³Π»ΠΎΠ²ΠΎΠΉ скорости диска

А Ρ‚Π΅ΠΏΠ΅Ρ€ΡŒ ΠΏΠΎ Ρ„ΠΎΡ€ΠΌΡƒΠ»Π΅ (7. 5) для Π²Ρ‚ΠΎΡ€ΠΎΠΉ Ρ‚ΠΎΡ‡ΠΊΠΈ ΠΏΠΎΠ»ΡƒΡ‡Π°Π΅ΠΌ

(ΠΎΡ‚Π²Π΅Ρ‚ 2).

ΠŸΠΎΡΠΊΠΎΠ»ΡŒΠΊΡƒ ΡΠΊΠΎΡ€ΠΎΡΡ‚ΡŒ автомобиля Π² Π·Π°Π΄Π°Ρ‡Π΅ 7.1.6 Π½Π΅ мСняСтся Π² процСссС двиТСния для сравнСния Ρ†Π΅Π½Ρ‚Ρ€ΠΎΡΡ‚Ρ€Π΅ΠΌΠΈΡ‚Π΅Π»ΡŒΠ½Ρ‹Ρ… ускорСний автомобиля Π² Ρ€Π°Π·Π½Ρ‹Ρ… Ρ‚ΠΎΡ‡ΠΊΠ°Ρ… Ρ‚Ρ€Π°Π΅ΠΊΡ‚ΠΎΡ€ΠΈΠΈ слСдуСт ΠΈΡΠΏΠΎΠ»ΡŒΠ·ΠΎΠ²Π°Ρ‚ΡŒ Ρ„ΠΎΡ€ΠΌΡƒΠ»Ρƒ (7.4), ΠΈΠ· ΠΊΠΎΡ‚ΠΎΡ€ΠΎΠΉ Π½Π°Ρ…ΠΎΠ΄ΠΈΠΌ, Ρ‡Ρ‚ΠΎ ускорСниС Ρ‚Π΅ΠΌ большС, Ρ‡Π΅ΠΌ мСньшС радиус Ρ‚Ρ€Π°Π΅ΠΊΡ‚ΠΎΡ€ΠΈΠΈ (ΠΏΡ€Π°Π²ΠΈΠ»ΡŒΠ½Ρ‹ΠΉ ΠΎΡ‚Π²Π΅Ρ‚ –

3).

УскорСниС ΠΌΠ°Π»ΡŒΡ‡ΠΈΠΊΠ° ΠΈΠ· Π·Π°Π΄Π°Ρ‡ΠΈ 7.1.7 Π±ΡƒΠ΄Π΅Ρ‚ Ρ€Π°Π²Π½ΠΎ Π½ΡƒΠ»ΡŽ, Ссли Π΅Π³ΠΎ ΡΠΊΠΎΡ€ΠΎΡΡ‚ΡŒ ΠΎΡ‚Π½ΠΎΡΠΈΡ‚Π΅Π»ΡŒΠ½ΠΎ Π·Π΅ΠΌΠ»ΠΈ Π±ΡƒΠ΄Π΅Ρ‚ Ρ€Π°Π²Π½Π° Π½ΡƒΠ»ΡŽ. ΠŸΠΎΡΡ‚ΠΎΠΌΡƒ ΠΏΡ€ΠΈ Π΄Π²ΠΈΠΆΠ΅Π½ΠΈΠΈ ΠΌΠ°Π»ΡŒΡ‡ΠΈΠΊΠ° ΠΏΡ€ΠΎΡ‚ΠΈΠ² двиТСния карусСли, Π΅Π³ΠΎ ΡΠΊΠΎΡ€ΠΎΡΡ‚ΡŒ ΠΎΡ‚Π½ΠΎΡΠΈΡ‚Π΅Π»ΡŒΠ½ΠΎ карусСли Ρ€Π°Π²Π½Π° скорости карусСли ΠΎΡ‚Π½ΠΎΡΠΈΡ‚Π΅Π»ΡŒΠ½ΠΎ Π·Π΅ΠΌΠ»ΠΈ . Если ΠΌΠ°Π»ΡŒΡ‡ΠΈΠΊ ΠΏΠΎΠΉΠ΄Π΅Ρ‚ Π² Π΄Ρ€ΡƒΠ³ΡƒΡŽ сторону с Ρ‚ΠΎΠΉ ΠΆΠ΅ ΡΠΊΠΎΡ€ΠΎΡΡ‚ΡŒΡŽ ΠΎΡ‚Π½ΠΎΡΠΈΡ‚Π΅Π»ΡŒΠ½ΠΎ карусСли, Π΅Π³ΠΎ ΡΠΊΠΎΡ€ΠΎΡΡ‚ΡŒ ΠΎΡ‚Π½ΠΎΡΠΈΡ‚Π΅Π»ΡŒΠ½ΠΎ Π·Π΅ΠΌΠ»ΠΈ Π±ΡƒΠ΄Π΅Ρ‚ Ρ€Π°Π²Π½Π° . ΠŸΠΎΡΡ‚ΠΎΠΌΡƒ Ρ†Π΅Π½Ρ‚Ρ€ΠΎΡΡ‚Ρ€Π΅ΠΌΠΈΡ‚Π΅Π»ΡŒΠ½ΠΎΠ΅ ускорСниС ΠΌΠ°Π»ΡŒΡ‡ΠΈΠΊΠ° Π±ΡƒΠ΄Π΅Ρ‚ Ρ€Π°Π²Π½ΠΎ

(ΠΎΡ‚Π²Π΅Ρ‚ 4).

Π’Π΅Π»ΠΎ, находящССся Π½Π° повСрхности Π²Ρ€Π°Ρ‰Π°ΡŽΡ‰Π΅Π³ΠΎΡΡ диска ΠΈ Π²Ρ€Π°Ρ‰Π°ΡŽΡ‰Π΅Π΅ΡΡ вмСстС с Π½ΠΈΠΌ (Π·Π°Π΄Π°Ρ‡Π° 7.1.8), участвуСт Π² ΡΠ»Π΅Π΄ΡƒΡŽΡ‰ΠΈΡ… взаимодСйствиях. Π’ΠΎ-ΠΏΠ΅Ρ€Π²Ρ‹Ρ…, Ρ‚Π΅Π»ΠΎ притягиваСтся ΠΊ Π·Π΅ΠΌΠ»Π΅ (сила тяТСсти), ΠΈ Π½Π° Π½Π΅Π³ΠΎ дСйствуСт ΠΏΠΎΠ²Π΅Ρ€Ρ…Π½ΠΎΡΡ‚ΡŒ диска (сила Π½ΠΎΡ€ΠΌΠ°Π»ΡŒΠ½ΠΎΠΉ Ρ€Π΅Π°ΠΊΡ†ΠΈΠΈ ΠΈ трСния), ΠΏΡ€ΠΈΡ‡Π΅ΠΌ сила трСния Π² ΠΊΠ°ΠΆΠ΄Ρ‹ΠΉ ΠΌΠΎΠΌΠ΅Π½Ρ‚ Π²Ρ€Π΅ΠΌΠ΅Π½ΠΈ Π½Π°ΠΏΡ€Π°Π²Π»Π΅Π½Π° ΠΊ оси вращСния (см. рисунок). Π”Π΅ΠΉΡΡ‚Π²ΠΈΡ‚Π΅Π»ΡŒΠ½ΠΎ, Π² отсутствии силы трСния Ρ‚Π΅Π»ΠΎ Π»ΠΈΠ±ΠΎ Π±ΡƒΠ΄Π΅Ρ‚ ΠΎΡΡ‚Π°Π²Π°Ρ‚ΡŒΡΡ Π½Π° мСстС, Π° диск ΠΏΠΎΠ΄ Π½ΠΈΠΌ Π±ΡƒΠ΄Π΅Ρ‚ Π²Ρ€Π°Ρ‰Π°Ρ‚ΡŒΡΡ, Π»ΠΈΠ±ΠΎ (Ссли Ρ‚Π΅Π»ΠΎ ΠΈΠΌΠ΅Π΅Ρ‚ ΡΠΊΠΎΡ€ΠΎΡΡ‚ΡŒ) слСтит с повСрхности диска. ИмСнно сила трСния «заставляСт» Ρ‚Π΅Π»ΠΎ Π²Ρ€Π°Ρ‰Π°Ρ‚ΡŒΡΡ вмСстС с диском. ΠŸΠΎΡΡ‚ΠΎΠΌΡƒ сила трСния слуТит Π² Π΄Π°Π½Π½ΠΎΠΉ Π·Π°Π΄Π°Ρ‡Π΅ Ρ†Π΅Π½-Ρ‚Ρ€ΠΎΡΡ‚Ρ€Π΅ΠΌΠΈΡ‚Π΅Π»ΡŒΠ½ΠΎΠΉ силой. ΠžΡΡ‚Π°Π»ΡŒΠ½Ρ‹Π΅ пСрСчислСния, Π΄Π°Π½Π½Ρ‹Π΅ Π² условии: Β«Π½Π° Ρ‚Π΅Π»ΠΎ Π΄Π΅ΠΉΡΡ‚Π²ΡƒΡŽΡ‚ силы тяТСсти, трСния, Ρ€Π΅Π°ΠΊΡ†ΠΈΠΈ ΠΎΠΏΠΎΡ€Ρ‹, Ρ†Π΅Π½Ρ‚Ρ€ΠΎΡΡ‚Ρ€Π΅ΠΌΠΈΡ‚Π΅Π»ΡŒΠ½Π°Ρ (ΠΈΠ»ΠΈ цСнтробСТная)Β» ΡΠ²Π»ΡΡŽΡ‚ΡΡ Π½Π΅ΠΏΡ€Π°Π²ΠΈΠ»ΡŒΠ½Ρ‹ΠΌΠΈ, ΠΏΠΎΡΠΊΠΎΠ»ΡŒΠΊΡƒ Π² Π½ΠΈΡ… ΡΠΌΠ΅ΡˆΠΈΠ²Π°ΡŽΡ‚ΡΡ характСристики сил Ρ€Π°Π·Π½Ρ‹Ρ… Ρ‚ΠΈΠΏΠΎΠ² – ΠΏΠ΅Ρ€Π²Ρ‹Π΅ Ρ‚Ρ€ΠΈ ΠΊΠ°ΡΠ°ΡŽΡ‚ΡΡ ΠΏΡ€ΠΈΡ€ΠΎΠ΄Ρ‹ взаимодСйствий, Π²Ρ‚ΠΎΡ€Ρ‹Π΅ – Ρ€Π΅Π·ΡƒΠ»ΡŒΡ‚Π°Ρ‚ дСйствия. ΠŸΠΎΡΡ‚ΠΎΠΌΡƒ ΠΏΡ€Π°Π²ΠΈΠ»ΡŒΠ½Ρ‹ΠΉ ΠΎΡ‚Π²Π΅Ρ‚ Π½Π° вопрос Π·Π°Π΄Π°Ρ‡ΠΈ –

1. ΠšΡ€ΠΎΠΌΠ΅ Ρ‚ΠΎΠ³ΠΎ, ΠΎΡ‚ΠΌΠ΅Ρ‚ΠΈΠΌ, Ρ‡Ρ‚ΠΎ цСнтробСТная сила Π²ΠΎΠ·Π½ΠΈΠΊΠ°Π΅Ρ‚ Ρ‚ΠΎΠ»ΡŒΠΊΠΎ Π² Π½Π΅ΠΈΠ½Π΅Ρ€Ρ†ΠΈΠ°Π»ΡŒΠ½Ρ‹Ρ… систСмах отсчСта ΠΈ Π² школьном курсС Ρ„ΠΈΠ·ΠΈΠΊΠΈ Π½Π΅ рассматриваСтся (поэтому Π»ΡƒΡ‡ΡˆΠ΅ этим понятиСм Π²ΠΎΠΎΠ±Ρ‰Π΅ Π½Π΅ ΠΏΠΎΠ»ΡŒΠ·ΠΎΠ²Π°Ρ‚ΡŒΡΡ).

ΠŸΠΎΡΠΊΠΎΠ»ΡŒΠΊΡƒ Ρ‚Π΅Π»ΠΎ Π² Π·Π°Π΄Π°Ρ‡Π΅ 7.1.9 вращаСтся с постоянной ΠΏΠΎ Π²Π΅Π»ΠΈΡ‡ΠΈΠ½Π΅ ΡΠΊΠΎΡ€ΠΎΡΡ‚ΡŒΡŽ ΠΏΠΎ окруТности, Ρ‚ΠΎ Π΅Π³ΠΎ ускорСниС Π½Π°ΠΏΡ€Π°Π²Π»Π΅Π½ΠΎ ΠΊ Ρ†Π΅Π½Ρ‚Ρ€Ρƒ окруТности, ΠΈ, ΡΠ»Π΅Π΄ΠΎΠ²Π°Ρ‚Π΅Π»ΡŒΠ½ΠΎ, согласно Π²Ρ‚ΠΎΡ€ΠΎΠΌΡƒ Π·Π°ΠΊΠΎΠ½Ρƒ ΠΡŒΡŽΡ‚ΠΎΠ½Π°, Ρ‚ΡƒΠ΄Π° ΠΆΠ΅ Π½Π°ΠΏΡ€Π°Π²Π»Π΅Π½Π° ΠΈ Ρ€Π΅Π·ΡƒΠ»ΡŒΡ‚ΠΈΡ€ΡƒΡŽΡ‰Π°Ρ сила, Π΄Π΅ΠΉΡΡ‚Π²ΡƒΡŽΡ‰Π°Ρ Π½Π° Ρ‚Π΅Π»ΠΎ (ΠΎΡ‚Π²Π΅Ρ‚ 2).

ΠŸΡ€ΠΈΠΌΠ΅Π½ΡΡ ΠΊ Π΄Π°Π½Π½ΠΎΠΌΡƒ Π² Π·Π°Π΄Π°Ρ‡Π΅ 7.1.10 Ρ‚Π΅Π»Ρƒ Π²Ρ‚ΠΎΡ€ΠΎΠΉ Π·Π°ΠΊΠΎΠ½ ΠΡŒΡŽΡ‚ΠΎΠ½Π° ΠΈ учитывая, Ρ‡Ρ‚ΠΎ Π΅Π³ΠΎ ускорСниС Ρ€Π°Π²Π½ΠΎ ΠΌ/с2, ΠΏΠΎΠ»ΡƒΡ‡ΠΈΠΌ для Ρ€Π°Π²Π½ΠΎΠ΄Π΅ΠΉΡΡ‚Π²ΡƒΡŽΡ‰Π΅ΠΉ =2 Н (ΠΎΡ‚Π²Π΅Ρ‚ 2).

Π˜ΡΠΏΠΎΠ»ΡŒΠ·ΡƒΡ Ρ„ΠΎΡ€ΠΌΡƒΠ»Ρƒ для Ρ†Π΅Π½Ρ‚Ρ€ΠΎΡΡ‚Ρ€Π΅ΠΌΠΈΡ‚Π΅Π»ΡŒΠ½ΠΎΠ³ΠΎ ускорСния , Π½Π°Ρ…ΠΎΠ΄ΠΈΠΌ ΠΎΡ‚Π½ΠΎΡˆΠ΅Π½ΠΈΠ΅ ускорСний ΠΌΠ°Ρ‚Π΅Ρ€ΠΈΠ°Π»ΡŒΠ½Ρ‹Ρ… Ρ‚ΠΎΡ‡Π΅ΠΊ ΠΈΠ· Π·Π°Π΄Π°Ρ‡ΠΈ 7.2.1

(ΠΎΡ‚Π²Π΅Ρ‚ 1).

Для сравнСния Ρ†Π΅Π½Ρ‚Ρ€ΠΎΡΡ‚Ρ€Π΅ΠΌΠΈΡ‚Π΅Π»ΡŒΠ½Ρ‹Ρ… ускорСний ΠΌΠ°Ρ‚Π΅Ρ€ΠΈΠ°Π»ΡŒΠ½Ρ‹Ρ… Ρ‚ΠΎΡ‡Π΅ΠΊ Π² Π·Π°Π΄Π°Ρ‡Π΅ 7. 2.2 ΡƒΠ΄ΠΎΠ±Π½ΠΎ ΠΈΡΠΏΠΎΠ»ΡŒΠ·ΠΎΠ²Π°Ρ‚ΡŒ Ρ„ΠΎΡ€ΠΌΡƒΠ»Ρƒ , ΠΏΠΎΡΠΊΠΎΠ»ΡŒΠΊΡƒ Π² этой Π·Π°Π΄Π°Ρ‡Π΅ ΠΎΠ΄ΠΈΠ½Π°ΠΊΠΎΠ²Ρ‹ ΡƒΠ³Π»ΠΎΠ²Ρ‹Π΅ скорости Ρ‚ΠΎΡ‡Π΅ΠΊ. ΠŸΠΎΠ»ΡƒΡ‡Π°Π΅ΠΌ

(ΠΎΡ‚Π²Π΅Ρ‚ 3).

Для сравнСния Ρ†Π΅Π½Ρ‚Ρ€ΠΎΡΡ‚Ρ€Π΅ΠΌΠΈΡ‚Π΅Π»ΡŒΠ½Ρ‹Ρ… ускорСний Ρ‚Π΅Π» Π² Π·Π°Π΄Π°Ρ‡Π΅ 7.2.3 Π²Ρ‹Ρ€Π°Π·ΠΈΠΌ ускорСниС Ρ‡Π΅Ρ€Π΅Π· радиус окруТности ΠΈ ΠΏΠ΅Ρ€ΠΈΠΎΠ΄. Π˜ΡΠΏΠΎΠ»ΡŒΠ·ΡƒΡ Ρ„ΠΎΡ€ΠΌΡƒΠ»Ρƒ (7.2) для ΠΏΠ΅Ρ€ΠΈΠΎΠ΄Π° ΠΈ (7.5) для Ρ†Π΅Π½Ρ‚Ρ€ΠΎΡΡ‚Ρ€Π΅ΠΌΠΈΡ‚Π΅Π»ΡŒΠ½ΠΎΠ³ΠΎ ускорСния, ΠΏΠΎΠ»ΡƒΡ‡ΠΈΠΌ

(7.5)

ΠŸΠΎΡΡ‚ΠΎΠΌΡƒ

(ΠΎΡ‚Π²Π΅Ρ‚ 1).

Π˜ΡΠΏΠΎΠ»ΡŒΠ·ΡƒΡ связь ΡƒΠ³Π»ΠΎΠ²ΠΎΠΉ ΠΈ Π»ΠΈΠ½Π΅ΠΉΠ½ΠΎΠΉ скорости, Π½Π°Ρ…ΠΎΠ΄ΠΈΠΌ скорости ΠΊΠΎΠ½Ρ†ΠΎΠ² часовой ΠΈ ΠΌΠΈΠ½ΡƒΡ‚Π½ΠΎΠΉ стрСлки (Π·Π°Π΄Π°Ρ‡Π° 7.2.4)

Π³Π΄Π΅ ΠΈ – ΡƒΠ³Π»ΠΎΠ²Ρ‹Π΅ скорости часовой ΠΈ ΠΌΠΈΠ½ΡƒΡ‚Π½ΠΎΠΉ стрСлки соотвСтствСнно (Π² Ρ€Π°Π΄/час), ΠΈ – Π΄Π»ΠΈΠ½Ρ‹ часовой ΠΈ ΠΌΠΈΠ½ΡƒΡ‚Π½ΠΎΠΉ стрСлок. Учитывая, Ρ‡Ρ‚ΠΎ , ΠΏΠΎΠ»ΡƒΡ‡Π°Π΅ΠΌ

(ΠΎΡ‚Π²Π΅Ρ‚ 2).

Π’Π΅Π»Ρƒ, Π²Ρ€Π°Ρ‰Π°ΡŽΡ‰Π΅ΠΌΡƒΡΡ вмСстС с диском Π½Π° Π΅Π³ΠΎ Π³ΠΎΡ€ΠΈΠ·ΠΎΠ½Ρ‚Π°Π»ΡŒΠ½ΠΎΠΉ повСрхности (Π·Π°Π΄Π°Ρ‡Π° 7.2.5), Ρ†Π΅Π½Ρ‚Ρ€ΠΎΡΡ‚Ρ€Π΅ΠΌΠΈΡ‚Π΅Π»ΡŒΠ½ΠΎΠ΅ ускорСниС сообщаСтся силой трСния

ΠŸΠΎΡΡ‚ΠΎΠΌΡƒ ΠΏΡ€ΠΈ ΡƒΠ²Π΅Π»ΠΈΡ‡Π΅Π½ΠΈΠΈ ΡƒΠ³Π»ΠΎΠ²ΠΎΠΉ скорости вращСния диска возрастаСт ΠΈ сила трСния ΠΌΠ΅ΠΆΠ΄Ρƒ Ρ‚Π΅Π»ΠΎΠΌ ΠΈ диском. ΠŸΡ€ΠΈ Π½Π΅ΠΊΠΎΡ‚ΠΎΡ€ΠΎΠΉ ΡƒΠ³Π»ΠΎΠ²ΠΎΠΉ скорости сила трСния достигнСт максимально Π²ΠΎΠ·ΠΌΠΎΠΆΠ½ΠΎΠ³ΠΎ для Π½Π΅Π΅ значСния . Если Π΅Ρ‰Π΅ ΡƒΠ²Π΅Π»ΠΈΡ‡ΠΈΡ‚ΡŒ ΡƒΠ³Π»ΠΎΠ²ΡƒΡŽ ΡΠΊΠΎΡ€ΠΎΡΡ‚ΡŒ диска, сила трСния ΡƒΠΆΠ΅ Π½Π΅ смоТСт ΡƒΠ΄Π΅Ρ€ΠΆΠ°Ρ‚ΡŒ Ρ‚Π΅Π»ΠΎ Π½Π° дискС: Ρ‚Π΅Π»ΠΎ Π½Π°Ρ‡Π½Π΅Ρ‚ ΡΠΊΠΎΠ»ΡŒΠ·ΠΈΡ‚ΡŒ ΠΏΠΎ повСрхности ΠΈ слСтит с повСрхности диска. ΠŸΠΎΡΡ‚ΠΎΠΌΡƒ значСния ΡƒΠ³Π»ΠΎΠ²ΠΎΠΉ скорости, ΠΏΡ€ΠΈ ΠΊΠΎΡ‚ΠΎΡ€ΠΎΠΉ Ρ‚Π΅Π»ΠΎ ΠΌΠΎΠΆΠ΅Ρ‚ Π²Ρ€Π°Ρ‰Π°Ρ‚ΡŒΡΡ вмСстС с диском, находится ΠΈΠ· нСравСнства

(ΠΎΡ‚Π²Π΅Ρ‚ 4).

Π’ Π·Π°Π΄Π°Ρ‡Π΅ 7. 2.6 Ρ†Π΅Π½Ρ‚Ρ€ΠΎΡΡ‚Ρ€Π΅ΠΌΠΈΡ‚Π΅Π»ΡŒΠ½ΠΎΠΉ силой являСтся сила натяТСния Π½ΠΈΡ‚ΠΈ. ΠŸΠΎΡΡ‚ΠΎΠΌΡƒ ΠΈΠ· Π²Ρ‚ΠΎΡ€ΠΎΠ³ΠΎ Π·Π°ΠΊΠΎΠ½Π° ΠΡŒΡŽΡ‚ΠΎΠ½Π° с ΡƒΡ‡Π΅Ρ‚ΠΎΠΌ Ρ„ΠΎΡ€ΠΌΡƒΠ»Ρ‹ (7.5) для Ρ†Π΅Π½Ρ‚Ρ€ΠΎΡΡ‚Ρ€Π΅ΠΌΠΈΡ‚Π΅Π»ΡŒΠ½ΠΎΠ³ΠΎ ускорСния ΠΈΠΌΠ΅Π΅ΠΌ

(ΠΎΡ‚Π²Π΅Ρ‚ 3).

Π’ Π·Π°Π΄Π°Ρ‡Π΅ 7.2.7 Π½ΡƒΠΆΠ½ΠΎ ΠΈΡΠΏΠΎΠ»ΡŒΠ·ΠΎΠ²Π°Ρ‚ΡŒ Π²Ρ‚ΠΎΡ€ΠΎΠΉ Π·Π°ΠΊΠΎΠ½ ΠΡŒΡŽΡ‚ΠΎΠ½Π° для ΠΊΠ°ΠΆΠ΄ΠΎΠ³ΠΎ Ρ‚Π΅Π»Π°. Π‘ΠΈΠ»Ρ‹, Π΄Π΅ΠΉΡΡ‚Π²ΡƒΡŽΡ‰ΠΈΠ΅ Π½Π° Ρ‚Π΅Π»Π°, ΠΏΠΎΠΊΠ°Π·Π°Π½Ρ‹ Π½Π° рисункС. ΠŸΡ€ΠΎΠ΅ΠΊΡ†ΠΈΡ Π²Ρ‚ΠΎΡ€ΠΎΠ³ΠΎ Π·Π°ΠΊΠΎΠ½Π° ΠΡŒΡŽΡ‚ΠΎΠ½Π° для дальнСго Ρ‚Π΅Π»Π° Π½Π° ΠΊΠΎΠΎΡ€Π΄ΠΈΠ½Π°Ρ‚Π½ΡƒΡŽ ось, Π½Π°ΠΏΡ€Π°Π²Π»Π΅Π½Π½ΡƒΡŽ ΠΊ Ρ†Π΅Π½Ρ‚Ρ€Ρƒ диска, Π΄Π°Π΅Ρ‚

(1)

На Π±Π»ΠΈΠΆΠ½Π΅Π΅ Ρ‚Π΅Π»ΠΎ Π΄Π΅ΠΉΡΡ‚Π²ΡƒΡŽΡ‚ силы натяТСния ΠΈ Π΄Π²ΡƒΡ… Π½ΠΈΡ‚Π΅ΠΉ (см. рисунок). ΠŸΠΎΡΡ‚ΠΎΠΌΡƒ для Π½Π΅Π³ΠΎ ΠΈΠ· Π²Ρ‚ΠΎΡ€ΠΎΠ³ΠΎ Π·Π°ΠΊΠΎΠ½Π° ΠΡŒΡŽΡ‚ΠΎΠ½Π° ΠΈΠΌΠ΅Π΅ΠΌ

ΠŸΠΎΠ΄ΡΡ‚Π°Π²Π»ΡΡ Π² эту Ρ„ΠΎΡ€ΠΌΡƒΠ»Ρƒ силу ΠΈΠ· Ρ„ΠΎΡ€ΠΌΡƒΠ»Ρ‹ (1), Π½Π°Ρ…ΠΎΠ΄ΠΈΠΌ (ΠΎΡ‚Π²Π΅Ρ‚ 2).

Π’ Π·Π°Π΄Π°Ρ‡Π΅ 7. 2.8 Π½Π΅ΠΎΠ±Ρ…ΠΎΠ΄ΠΈΠΌΠΎ ΠΈΡΠΏΠΎΠ»ΡŒΠ·ΠΎΠ²Π°Ρ‚ΡŒ Ρ‚ΠΎ ΠΎΠ±ΡΡ‚ΠΎΡΡ‚Π΅Π»ΡŒΡΡ‚Π²ΠΎ, Ρ‡Ρ‚ΠΎ угловая ΡΠΊΠΎΡ€ΠΎΡΡ‚ΡŒ всСх Ρ‚ΠΎΡ‡Π΅ΠΊ стСрТня ΠΎΠ΄ΠΈΠ½Π°ΠΊΠΎΠ²Π°. ΠžΠ±ΠΎΠ·Π½Π°Ρ‡Π°Ρ расстояния ΠΎΡ‚ оси вращСния Π΄ΠΎ ΠΊΠΎΠ½Ρ†ΠΎΠ² стСрТня ΠΊΠ°ΠΊ ΠΈ , ΠΈΠΌΠ΅Π΅ΠΌ

Π³Π΄Π΅ = 1 ΠΌ/с ΠΈ = 2 ΠΌ/с – Π»ΠΈΠ½Π΅ΠΉΠ½Ρ‹Π΅ скорости ΠΊΠΎΠ½Ρ†ΠΎΠ² стСрТня, ΠΌ – Π΅Π³ΠΎ Π΄Π»ΠΈΠ½Π°. РСшая эту систСму ΡƒΡ€Π°Π²Π½Π΅Π½ΠΈΠΉ, Π½Π°ΠΉΠ΄Π΅ΠΌ расстояния ΠΈ , Π° Π·Π°Ρ‚Π΅ΠΌ ΠΈ ΡƒΠ³Π»ΠΎΠ²ΡƒΡŽ ΡΠΊΠΎΡ€ΠΎΡΡ‚ΡŒ стСрТня . Π’ Ρ€Π΅Π·ΡƒΠ»ΡŒΡ‚Π°Ρ‚Π΅ ΠΏΠΎΠ»ΡƒΡ‡ΠΈΠΌ

(ΠΎΡ‚Π²Π΅Ρ‚ 3).

Π‘Ρ€Π΅Π΄Π½Π΅Π΅ ускорСниС Ρ‚Π΅Π»Π° Π·Π° Π½Π΅ΠΊΠΎΡ‚ΠΎΡ€Ρ‹ΠΉ ΠΈΠ½Ρ‚Π΅Ρ€Π²Π°Π» Π²Ρ€Π΅ΠΌΠ΅Π½ΠΈ (Π½Π΅ ΠΎΠ±ΡΠ·Π°Ρ‚Π΅Π»ΡŒΠ½ΠΎ ΠΌΠ°Π»Ρ‹ΠΉ) опрСдСляСтся ΠΏΠΎ Ρ„ΠΎΡ€ΠΌΡƒΠ»Π΅ (2.1):

Π³Π΄Π΅ ΠΈ – скорости Ρ‚Π΅Π»Π° Π² ΠΊΠΎΠ½Ρ†Π΅ ΠΈ Π½Π°Ρ‡Π°Π»Π΅ ΠΈΠ½Ρ‚Π΅Ρ€Π²Π°Π»Π° Π²Ρ€Π΅ΠΌΠ΅Π½ΠΈ . Π—Π° ΠΏΠΎΠ»ΠΎΠ²ΠΈΠ½Ρƒ ΠΏΠ΅Ρ€ΠΈΠΎΠ΄Π° Π²Π΅ΠΊΡ‚ΠΎΡ€ скорости поворачиваСтся Π½Π° 180Β°, поэтому Π²Π΅Π»ΠΈΡ‡ΠΈΠ½Π° разности Ρ€Π°Π²Π½Π° . ΠŸΠΎΡΡ‚ΠΎΠΌΡƒ срСднСС ускорСниС Ρ‚Π΅Π»Π° Π·Π° ΠΏΠΎΠ»ΠΎΠ²ΠΈΠ½Ρƒ ΠΏΠ΅Ρ€ΠΈΠΎΠ΄Π° Ρ€Π°Π²Π½ΠΎ

(Π·Π°Π΄Π°Ρ‡Π° 7. 2.9 – ΠΎΡ‚Π²Π΅Ρ‚ 1).

ΠžΡ‡Π΅Π²ΠΈΠ΄Π½ΠΎ, ΠΏΡ€ΠΈ Π·ΡƒΠ±Ρ‡Π°Ρ‚ΠΎΠΉ ΠΏΠ΅Ρ€Π΅Π΄Π°Ρ‡Π΅ ΡΠΎΠ²ΠΏΠ°Π΄Π°ΡŽΡ‚ Π»ΠΈΠ½Π΅ΠΉΠ½Ρ‹Π΅ скорости Ρ‚ΠΎΡ‡Π΅ΠΊ Π½Π° ΠΎΠ±ΠΎΠ΄Π΅ ΡˆΠ΅ΡΡ‚Π΅Ρ€Π½Π΅ΠΉ. Π”Π΅ΠΉΡΡ‚Π²ΠΈΡ‚Π΅Π»ΡŒΠ½ΠΎ, Ссли Π±Ρ‹ эти скорости Π±Ρ‹Π»ΠΈ Ρ€Π°Π·Π½Ρ‹ΠΌΠΈ, ΠΌΠ΅ΠΆΠ΄Ρƒ повСрхностями ΡˆΠ΅ΡΡ‚Π΅Ρ€Π½Π΅ΠΉ Π±Ρ‹Π»ΠΎ Π±Ρ‹ ΠΏΡ€ΠΎΡΠΊΠ°Π»ΡŒΠ·Ρ‹Π²Π°Π½ΠΈΠ΅, ΠΊΠΎΡ‚ΠΎΡ€ΠΎΠΌΡƒ ΠΏΡ€Π΅ΠΏΡΡ‚ΡΡ‚Π²ΡƒΡŽΡ‚ Π·ΡƒΠ±Ρ†Ρ‹ ΡˆΠ΅ΡΡ‚Π΅Ρ€Π½Π΅ΠΉ (Π·Π°Π΄Π°Ρ‡Π° 7.2.10 – ΠΎΡ‚Π²Π΅Ρ‚ 2).

ЀизичСскиС основы ΠΌΠ΅Ρ…Π°Π½ΠΈΠΊΠΈ

ΠŸΡ€Π΅Π΄ΡΡ‚Π°Π²ΠΈΠΌ сСбС ΠΌΠ°Ρ‚Π΅Ρ€ΠΈΠ°Π»ΡŒΠ½ΡƒΡŽ Ρ‚ΠΎΡ‡ΠΊΡƒ, Π΄Π²ΠΈΠΆΡƒΡ‰ΡƒΡŽΡΡ ΠΏΠΎ Π½Π΅ΠΊΠΎΡ‚ΠΎΡ€ΠΎΠΉ ΠΊΡ€ΠΈΠ²ΠΎΠ»ΠΈΠ½Π΅ΠΉΠ½ΠΎΠΉ Ρ‚Ρ€Π°Π΅ΠΊΡ‚ΠΎΡ€ΠΈΠΈ . Π—Π°ΠΏΠΈΡˆΠ΅ΠΌ ΡΠΊΠΎΡ€ΠΎΡΡ‚ΡŒ Π² Π²ΠΈΠ΄Π΅

ΠΈ Π·Π°ΠΌΠ΅Ρ‚ΠΈΠΌ, Ρ‡Ρ‚ΠΎ Π²Π΅ΠΊΡ‚ΠΎΡ€

β€” это Π΅Π΄ΠΈΠ½ΠΈΡ‡Π½Ρ‹ΠΉ Π²Π΅ΠΊΡ‚ΠΎΡ€, ΠΊΠ°ΡΠ°Ρ‚Π΅Π»ΡŒΠ½Ρ‹ΠΉ ΠΊ Ρ‚Ρ€Π°Π΅ΠΊΡ‚ΠΎΡ€ΠΈΠΈ ΠΈ ΡΠΎΠ²ΠΏΠ°Π΄Π°ΡŽΡ‰ΠΈΠΉ ΠΏΠΎ Π½Π°ΠΏΡ€Π°Π²Π»Π΅Π½ΠΈΡŽ с Π²Π΅ΠΊΡ‚ΠΎΡ€ΠΎΠΌ скорости. ΠŸΡ€ΠΎΠ΄ΠΈΡ„Ρ„Π΅Ρ€Π΅Π½Ρ†ΠΈΡ€ΡƒΠ΅ΠΌ Π²Π΅ΠΊΡ‚ΠΎΡ€ скорости, записанный Π² Π΄Π°Π½Π½ΠΎΠΌ прСдставлСнии, ΠΈ ΠΏΠΎΠ»ΡƒΡ‡ΠΈΠΌ

ΠœΡ‹ прСдставили ускорСниС Π² Π²ΠΈΠ΄Π΅ Π΄Π²ΡƒΡ… слагаСмых. Π—Π°ΠΌΠ΅Ρ‚ΠΈΠΌ ΠΏΡ€Π΅ΠΆΠ΄Π΅ всСго, Ρ‡Ρ‚ΠΎ слагаСмыС ΠΎΡ€Ρ‚ΠΎΠ³ΠΎΠ½Π°Π»ΡŒΠ½Ρ‹ Π΄Ρ€ΡƒΠ³ Π΄Ρ€ΡƒΠ³Ρƒ. Π”Π΅ΠΉΡΡ‚Π²ΠΈΡ‚Π΅Π»ΡŒΠ½ΠΎ, ΠΏΠΎΡΠΊΠΎΠ»ΡŒΠΊΡƒ Π²Π΅ΠΊΡ‚ΠΎΡ€ β€” Π΅Π΄ΠΈΠ½ΠΈΡ‡Π½Ρ‹ΠΉ, Ρ‚ΠΎ

ДиффСрСнцируя это скалярноС ΠΏΡ€ΠΎΠΈΠ·Π²Π΅Π΄Π΅Π½ΠΈΠ΅, ΠΏΠΎΠ»ΡƒΡ‡Π°Π΅ΠΌ

Ρ‚ΠΎ Π΅ΡΡ‚ΡŒ

ΠΏΠΎ свойству скалярного произвСдСния.

Π’Π°ΠΊΠΈΠΌ ΠΎΠ±Ρ€Π°Π·ΠΎΠΌ, ΠΌΡ‹ Ρ€Π°Π·Π»ΠΎΠΆΠΈΠ»ΠΈ ускорСниС Π½Π° сумму Π΄Π²ΡƒΡ… Π²Π·Π°ΠΈΠΌΠ½ΠΎ ΠΎΡ€Ρ‚ΠΎΠ³ΠΎΠ½Π°Π»ΡŒΠ½Ρ‹Ρ… ΡΠΎΡΡ‚Π°Π²Π»ΡΡŽΡ‰ΠΈΡ…, ΠΎΠ±ΠΎΠ·Π½Π°Ρ‡Π΅ΠΌ ΠΈΡ… ΠΈ :

ΠžΠ±ΡΡƒΠ΄ΠΈΠΌ физичСский смысл ΠΊΠ°ΠΆΠ΄ΠΎΠ³ΠΎ слагаСмого. Π‘Π»Π°Π³Π°Π΅ΠΌΠΎΠ΅

β€” это Ρ‚Π°Π½Π³Π΅Π½Ρ†ΠΈΠ°Π»ΡŒΠ½ΠΎΠ΅ ускорСниС, ΠΊΠΎΡ‚ΠΎΡ€ΠΎΠ΅ Ρ…Π°Ρ€Π°ΠΊΡ‚Π΅Ρ€ΠΈΠ·ΡƒΠ΅Ρ‚ быстроту измСнСния модуля скорости. Π­Ρ‚Π° Ρ‡Π°ΡΡ‚ΡŒ ΠΏΠΎΠ»Π½ΠΎΠ³ΠΎ ускорСния Π½Π°ΠΏΡ€Π°Π²Π»Π΅Π½Π° Π»ΠΈΠ±ΠΎ ΠΏΠΎ скорости, ΠΊΠΎΠ³Π΄Π° производная dv/dt > 0, Ρ‚ΠΎ Π΅ΡΡ‚ΡŒ Π΄Π²ΠΈΠΆΠ΅Π½ΠΈΠ΅ ускорСнноС, Π»ΠΈΠ±ΠΎ Π² сторону ΠΏΡ€ΠΎΡ‚ΠΈΠ²ΠΎΠΏΠΎΠ»ΠΎΠΆΠ½ΡƒΡŽ скорости, ΠΊΠΎΠ³Π΄Π° эта производная dv/dt < 0, Ρ‚ΠΎ Π΅ΡΡ‚ΡŒ Π΄Π²ΠΈΠΆΠ΅Π½ΠΈΠ΅ Π·Π°ΠΌΠ΅Π΄Π»Π΅Π½Π½ΠΎΠ΅. Если Π΄Π²ΠΈΠΆΠ΅Π½ΠΈΠ΅ Ρ€Π°Π²Π½ΠΎΠΌΠ΅Ρ€Π½ΠΎΠ΅ dv/dt = 0, Ρ‚ΠΎ Π΅ΡΡ‚ΡŒ ΡΠΊΠΎΡ€ΠΎΡΡ‚ΡŒ, Ссли ΠΈ мСняСтся, Ρ‚ΠΎ лишь ΠΏΠΎ Π½Π°ΠΏΡ€Π°Π²Π»Π΅Π½ΠΈΡŽ, Ρ‚ΠΎ Ρ‚Π°Π½Π³Π΅Π½Ρ†ΠΈΠ°Π»ΡŒΠ½Π°Ρ Ρ‡Π°ΡΡ‚ΡŒ ускорСния Ρ€Π°Π²Π½Π° Π½ΡƒΠ»ΡŽ:

Π‘Π»Π°Π³Π°Π΅ΠΌΠΎΠ΅

Π½Π°ΠΏΡ€Π°Π²Π»Π΅Π½ΠΎ ΠΏΠΎ Π½ΠΎΡ€ΠΌΠ°Π»ΠΈ ΠΊ Ρ‚Ρ€Π°Π΅ΠΊΡ‚ΠΎΡ€ΠΈΠΈ β€” пСрпСндикулярно ΠΊΠ°ΡΠ°Ρ‚Π΅Π»ΡŒΠ½ΠΎΠΉ ΠΊ Ρ‚Ρ€Π°Π΅ΠΊΡ‚ΠΎΡ€ΠΈΠΈ ΠΈ называСтся Π½ΠΎΡ€ΠΌΠ°Π»ΡŒΠ½Ρ‹ΠΌ ускорСниСм. Если Ρ‚Π°Π½Π³Π΅Π½Ρ†ΠΈΠ°Π»ΡŒΠ½ΠΎΠ΅ ускорСниС опрСдСляСт ΡΠΊΠΎΡ€ΠΎΡΡ‚ΡŒ, с ΠΊΠΎΡ‚ΠΎΡ€ΠΎΠΉ мСняСтся ΠΌΠΎΠ΄ΡƒΠ»ΡŒ Π²Π΅ΠΊΡ‚ΠΎΡ€Π° скорости, Ρ‚ΠΎ Π½ΠΎΡ€ΠΌΠ°Π»ΡŒΠ½ΠΎΠ΅ ускорСниС опрСдСляСт ΡΠΊΠΎΡ€ΠΎΡΡ‚ΡŒ, с ΠΊΠΎΡ‚ΠΎΡ€ΠΎΠΉ мСняСтся Π½Π°ΠΏΡ€Π°Π²Π»Π΅Π½ΠΈΠ΅ Π²Π΅ΠΊΡ‚ΠΎΡ€Π° скорости.

Рис. 2.10. К ΠΎΠΏΡ€Π΅Π΄Π΅Π»Π΅Π½ΠΈΡŽ ΠΊΡ€ΠΈΠ²ΠΈΠ·Π½Ρ‹ Ρ‚Ρ€Π°Π΅ΠΊΡ‚ΠΎΡ€ΠΈΠΈ

Рассмотрим «достаточно Π³Π»Π°Π΄ΠΊΡƒΡŽΒ», Π² ΠΎΡΡ‚Π°Π»ΡŒΠ½ΠΎΠΌ ΠΏΡ€ΠΎΠΈΠ·Π²ΠΎΠ»ΡŒΠ½ΡƒΡŽ ΠΏΠ»ΠΎΡΠΊΡƒΡŽ ΠΊΡ€ΠΈΠ²ΠΎΠ»ΠΈΠ½Π΅ΠΉΠ½ΡƒΡŽ Ρ‚Ρ€Π°Π΅ΠΊΡ‚ΠΎΡ€ΠΈΡŽ. ΠŸΠ»ΠΎΡΠΊΡƒΡŽ, Ρ‚ΠΎ Π΅ΡΡ‚ΡŒ всС Ρ‚ΠΎΡ‡ΠΊΠΈ Ρ‚Ρ€Π°Π΅ΠΊΡ‚ΠΎΡ€ΠΈΠΈ Π»Π΅ΠΆΠ°Ρ‚ Π² Π½Π΅ΠΊΠΎΡ‚ΠΎΡ€ΠΎΠΉ плоскости, β€” ΠΈΡΠΊΠ»ΡŽΡ‡ΠΈΡ‚Π΅Π»ΡŒΠ½ΠΎ для упрощСния Π²Ρ‹ΠΊΠ»Π°Π΄ΠΎΠΊ, ΠΏΠΎΠ»ΡƒΡ‡Π°Π΅ΠΌΡ‹ΠΉ Π² Ρ€Π°ΠΌΠΊΠ°Ρ… этого прСдполоТСния, Ρ€Π΅Π·ΡƒΠ»ΡŒΡ‚Π°Ρ‚ годится ΠΈ для любой «достаточно Π³Π»Π°Π΄ΠΊΠΎΠΉΒ» пространствСнной ΠΊΡ€ΠΈΠ²ΠΎΠΉ, Ρ‡ΡŒΠΈ Ρ‚ΠΎΡ‡ΠΊΠΈ ΡƒΠ»ΠΎΠΆΠΈΡ‚ΡŒ Π² ΠΎΠ΄Π½Ρƒ ΠΏΠ»ΠΎΡΠΊΠΎΡΡ‚ΡŒ Π½Π΅Π²ΠΎΠ·ΠΌΠΎΠΆΠ½ΠΎ. ПослСднСС ΠΎΠ±ΡΡ‚ΠΎΡΡ‚Π΅Π»ΡŒΡΡ‚Π²ΠΎ ΠΌΡ‹ здСсь Ρ€Π°ΡΡΠΌΠ°Ρ‚Ρ€ΠΈΠ²Π°Ρ‚ΡŒ Π½Π΅ Π±ΡƒΠ΄Π΅ΠΌ, ΠΎΠ½ΠΎ строго доказываСтся ΠΌΠ΅Ρ‚ΠΎΠ΄Π°ΠΌΠΈ аналитичСской Π³Π΅ΠΎΠΌΠ΅Ρ‚Ρ€ΠΈΠΈ. Π‘Π»ΠΎΠ²Π° «достаточно гладкая» ΠΎΠ·Π½Π°Ρ‡Π°ΡŽΡ‚, Ρ‡Ρ‚ΠΎ кривая описываСтся Π½Π΅ΠΏΡ€Π΅Ρ€Ρ‹Π²Π½ΠΎΠΉ Ρ„ΡƒΠ½ΠΊΡ†ΠΈΠ΅ΠΉ, ΠΈΠΌΠ΅ΡŽΡ‰Π΅ΠΉ Π½Π΅ΠΏΡ€Π΅Ρ€Ρ‹Π²Π½Ρ‹Π΅ ΠΏΠ΅Ρ€Π²ΡƒΡŽ ΠΈ Π²Ρ‚ΠΎΡ€ΡƒΡŽ ΠΏΡ€ΠΎΠΈΠ·Π²ΠΎΠ΄Π½Ρ‹Π΅. Π‘ Ρ‚ΠΎΡ‡ΠΊΠΈ зрСния физичСских ΠΏΡ€ΠΈΠ»ΠΎΠΆΠ΅Π½ΠΈΠΉ, Ρ‚Ρ€Π΅Π±ΠΎΠ²Π°Π½ΠΈΠ΅ сущСствования Π½Π΅ΠΏΡ€Π΅Ρ€Ρ‹Π²Π½Ρ‹Ρ… ΠΏΠ΅Ρ€Π²Ρ‹Ρ… Π΄Π²ΡƒΡ… ΠΏΡ€ΠΎΠΈΠ·Π²ΠΎΠ΄Π½Ρ‹Ρ… фактичСски Π½Π΅ являСтся ΠΎΠ³Ρ€Π°Π½ΠΈΡ‡Π΅Π½ΠΈΠ΅ΠΌ Π½Π° Ρ„ΠΎΡ€ΠΌΡƒ Ρ‚Ρ€Π°Π΅ΠΊΡ‚ΠΎΡ€ΠΈΠΈ, Ρ‚Π°ΠΊ ΠΊΠ°ΠΊ практичСски всСгда Π²Ρ‹ΠΏΠΎΠ»Π½Π΅Π½ΠΎ. ΠŸΡ€ΠΎΡ‰Π΅ говоря, Π½Π° Ρ‚Ρ€Π°Π΅ΠΊΡ‚ΠΎΡ€ΠΈΠΈ Π½Π΅ Π΄ΠΎΠ»ΠΆΠ½ΠΎ Π±Ρ‹Ρ‚ΡŒ “ΡƒΠ³Π»ΠΎΠ²” Ρ‚ΠΈΠΏΠ° ΠΏΠΎΠΊΠ°Π·Π°Π½Π½ΠΎΠ³ΠΎ Π½Π° рисункС 2.11.

Рис. 2.11.

Π’Π°ΠΊΡƒΡŽ Β«Π³Π»Π°Π΄ΠΊΡƒΡŽΒ» ΠΊΡ€ΠΈΠ²ΡƒΡŽ Π½Π° любом Π΅Ρ‘ бСсконСчно ΠΌΠ°Π»ΠΎΠΌ участкС ΠΌΠΎΠΆΠ½ΠΎ Π·Π°ΠΌΠ΅Π½ΠΈΡ‚ΡŒ (рис. 2.12) участком окруТности Π½Π΅ΠΊΠΎΡ‚ΠΎΡ€ΠΎΠ³ΠΎ радиуса. Радиус этой окруТности, Π°ΠΏΠΏΡ€ΠΎΠΊΡΠΈΠΌΠΈΡ€ΡƒΡŽΡ‰Π΅ΠΉ Ρ‚Ρ€Π°Π΅ΠΊΡ‚ΠΎΡ€ΠΈΡŽ Π½Π° Π΅Ρ‘ бСсконСчно ΠΌΠ°Π»ΠΎΠΌ участкС Π² окрСстности Π½Π΅ΠΊΠΎΡ‚ΠΎΡ€ΠΎΠΉ Ρ‚ΠΎΡ‡ΠΊΠΈ, принято Π½Π°Π·Ρ‹Π²Π°Ρ‚ΡŒ радиусом ΠΊΡ€ΠΈΠ²ΠΈΠ·Π½Ρ‹ Ρ‚Ρ€Π°Π΅ΠΊΡ‚ΠΎΡ€ΠΈΠΈ Π² этой Ρ‚ΠΎΡ‡ΠΊΠ΅. Π¦Π΅Π½Ρ‚Ρ€ этой окруТности принято Π½Π°Π·Ρ‹Π²Π°Ρ‚ΡŒ Ρ†Π΅Π½Ρ‚Ρ€ΠΎΠΌ ΠΊΡ€ΠΈΠ²ΠΈΠ·Π½Ρ‹ Ρ‚Ρ€Π°Π΅ΠΊΡ‚ΠΎΡ€ΠΈΠΈ Π² Π΄Π°Π½Π½ΠΎΠΉ Ρ‚ΠΎΡ‡ΠΊΠ΅. ΠšΡ€ΠΈΠ²ΠΈΠ·Π½ΠΎΠΉ Ρ‚Ρ€Π°Π΅ΠΊΡ‚ΠΎΡ€ΠΈΠΈ называСтся Π²Π΅Π»ΠΈΡ‡ΠΈΠ½Π° C = 1/R. ΠŸΠΎΠ΄Ρ‡Π΅Ρ€ΠΊΠ½Π΅ΠΌ, Ρ‡Ρ‚ΠΎ радиус ΠΊΡ€ΠΈΠ²ΠΈΠ·Π½Ρ‹, ΠΊΠ°ΠΊ ΠΈ Ρ†Π΅Π½Ρ‚Ρ€ ΠΊΡ€ΠΈΠ²ΠΈΠ·Π½Ρ‹ Ρ‚Ρ€Π°Π΅ΠΊΡ‚ΠΎΡ€ΠΈΠΈ β€” Π΅Ρ‘ Π»ΠΎΠΊΠ°Π»ΡŒΠ½Ρ‹Π΅ характСристика: ΠΊΠ°ΠΆΠ΄ΠΎΠΉ Ρ‚ΠΎΡ‡ΠΊΠ΅ Ρ‚Ρ€Π°Π΅ΠΊΡ‚ΠΎΡ€ΠΈΠΈ соотвСтствуСт свой радиус ΠΊΡ€ΠΈΠ²ΠΈΠ·Π½Ρ‹ ΠΈ свой Ρ†Π΅Π½Ρ‚Ρ€ ΠΊΡ€ΠΈΠ²ΠΈΠ·Π½Ρ‹. Π˜ΡΠΊΠ»ΡŽΡ‡Π΅Π½ΠΈΡΠΌΠΈ ΡΠ²Π»ΡΡŽΡ‚ΡΡ: 1) ΠΎΠΊΡ€ΡƒΠΆΠ½ΠΎΡΡ‚ΡŒ, Π΅Ρ‘ радиус ΠΊΡ€ΠΈΠ²ΠΈΠ·Π½Ρ‹ Π²ΠΎ всСх Π΅Ρ‘ Ρ‚ΠΎΡ‡ΠΊΠ°Ρ… ΠΎΠ΄ΠΈΠ½ ΠΈ Ρ‚ΠΎΡ‚ ΠΆΠ΅ ΠΈ Ρ€Π°Π²Π΅Π½ радиусу окруТности, Ρ†Π΅Π½Ρ‚Ρ€ ΠΊΡ€ΠΈΠ²ΠΈΠ·Π½Ρ‹ Β«ΠΎΠ΄ΠΈΠ½ Π½Π° всСх» ΠΈ совпадаСт с Ρ†Π΅Π½Ρ‚Ρ€ΠΎΠΌ окруТности, ΠΈ 2) прямая, для любой Ρ‚ΠΎΡ‡ΠΊΠΈ прямой радиус ΠΊΡ€ΠΈΠ²ΠΈΠ·Π½Ρ‹ бСсконСчСн, Π° Ρ†Π΅Π½Ρ‚Ρ€ ΠΊΡ€ΠΈΠ²ΠΈΠ·Π½Ρ‹ находится Π² бСсконСчно ΡƒΠ΄Π°Π»Π΅Π½Π½ΠΎΠΉ ΠΎΡ‚ прямой Ρ‚ΠΎΡ‡ΠΊΠ΅. Π­Ρ‚ΠΎ Π»Π΅Π³ΠΊΠΎ ΠΏΠΎΠ½ΡΡ‚ΡŒ: Π΄Π°Π²Π°ΠΉΡ‚Π΅ ΡƒΠ²Π΅Π»ΠΈΡ‡ΠΈΠ²Π°Ρ‚ΡŒ радиус окруТности, Ρ‡Π΅ΠΌ большС радиус окруТности, Ρ‚Π΅ΠΌ Π±Π»ΠΈΠΆΠ΅ любой Π΅Ρ‘ ΠΊΠΎΠ½Π΅Ρ‡Π½Ρ‹ΠΉ участок ΠΊ участку прямой. На Ρ€Π°Π²Π½ΠΈΠ½Π΅, Π»ΡƒΡ‡ΡˆΠ΅ всСго Π½Π° пляТС, с высоты чСловСчСского роста Π΄ΠΎ Π³ΠΎΡ€ΠΈΠ·ΠΎΠ½Ρ‚Π° Π½Π΅ Π±ΠΎΠ»Π΅Π΅ пяти ΠΊΠΈΠ»ΠΎΠΌΠ΅Ρ‚Ρ€ΠΎΠ², β€” Π² этих ΠΏΡ€Π΅Π΄Π΅Π»Π°Ρ… ЗСмля плоская.

Рис. 2.12. К ΠΎΠΏΡ€Π΅Π΄Π΅Π»Π΅Π½ΠΈΡŽ радиуса ΠΊΡ€ΠΈΠ²ΠΈΠ·Π½Ρ‹ Ρ‚Ρ€Π°Π΅ΠΊΡ‚ΠΎΡ€ΠΈΠΈ

Вычислим ΠΌΠΎΠ΄ΡƒΠ»ΡŒ ΠΏΡ€ΠΎΠΈΠ·Π²ΠΎΠ΄Π½ΠΎΠΉ , входящСй Π² Π²Ρ‹Ρ€Π°ΠΆΠ΅Π½ΠΈΠ΅ для Π½ΠΎΡ€ΠΌΠ°Π»ΡŒΠ½ΠΎΠ³ΠΎ ускорСния. НаправлСн Π²Π΅ΠΊΡ‚ΠΎΡ€ ΠΏΠΎ Π½ΠΎΡ€ΠΌΠ°Π»ΠΈ ΠΊ Ρ‚Ρ€Π°Π΅ΠΊΡ‚ΠΎΡ€ΠΈΠΈ ΠΊ Ρ†Π΅Π½Ρ‚Ρ€Ρƒ ΠΊ Ρ†Π΅Π½Ρ‚Ρ€Ρƒ ΠΊΡ€ΠΈΠ²ΠΈΠ·Π½Ρ‹, Ρ‡Ρ‚ΠΎ поясняСт рис. 2.13.

Рис. 2.13. ГрафичСскоС ΠΎΠΏΡ€Π΅Π΄Π΅Π»Π΅Π½ΠΈΠ΅ радиуса ΠΊΡ€ΠΈΠ²ΠΈΠ·Π½Ρ‹ Ρ‚Ρ€Π°Π΅ΠΊΡ‚ΠΎΡ€ΠΈΠΈ

Для этого ΠΏΡ€Π΅ΠΆΠ΄Π΅ всСго ΠΏΠ΅Ρ€Π΅ΠΉΠ΄Π΅ΠΌ ΠΎΡ‚ диффСрСнцирования ΠΏΠΎ Π²Ρ€Π΅ΠΌΠ΅Π½ΠΈ ΠΊ Π΄ΠΈΡ„Ρ„Π΅Ρ€Π΅Π½Ρ†ΠΈΡ€ΠΎΠ²Π°Π½ΠΈΡŽ ΠΏΠΎ Β«ΠΏΡƒΡ‚ΠΈΒ»: , ΠΈΠΌΠ΅Π΅ΠΌ:

По ΠΎΠΏΡ€Π΅Π΄Π΅Π»Π΅Π½ΠΈΡŽ производная ΠΊΡ€ΠΈΠ²ΠΈΠ·Π½Π΅ ΠΊΡ€ΠΈΠ²ΠΎΠΉ C, Π° Π²Π΅Π»ΠΈΡ‡ΠΈΠ½Π° Π΅ΠΉ обратная Ρ€Π°Π²Π½Π° радиусу ΠΊΡ€ΠΈΠ²ΠΈΠ·Π½Ρ‹ ΠΊΡ€ΠΈΠ²ΠΎΠΉ R. Бобирая всё вмСстС, для Π½ΠΎΡ€ΠΌΠ°Π»ΡŒΠ½ΠΎΠ³ΠΎ ускорСния ΠΎΠΊΠΎΠ½Ρ‡Π°Ρ‚Π΅Π»ΡŒΠ½ΠΎ ΠΏΠΎΠ»ΡƒΡ‡Π°Π΅ΠΌ:

,

Π³Π΄Π΅ Π½ΠΎΡ€ΠΌΠ°Π»ΡŒ пСрпСндикулярна ΠΊ ΠΊΠ°ΡΠ°Ρ‚Π΅Π»ΡŒΠ½ΠΎΠΉ ΠΈ всСгда Π½Π°ΠΏΡ€Π°Π²Π»Π΅Π½Π° ΠΊ Ρ†Π΅Π½Ρ‚Ρ€Ρƒ ΠΊΡ€ΠΈΠ²ΠΈΠ·Π½Ρ‹, см. рис. 11.

ΠŸΡ€ΠΈΠ²Π΅Π΄Π΅ΠΌ Π½Π΅ΠΊΠΎΡ‚ΠΎΡ€ΠΎΠ΅ Π΄ΠΎΠΏΠΎΠ»Π½ΠΈΡ‚Π΅Π»ΡŒΠ½ΠΎΠ΅ пояснСниС ΠΊ рисунку 11. Π’ΠΎΠ·ΡŒΠΌΠ΅ΠΌ Π½Π΅ΠΏΠΎΠ΄Π°Π»Π΅ΠΊΡƒ ΠΎΡ‚ Ρ‚ΠΎΡ‡ΠΊΠΈ 1 Ρ‚ΠΎΡ‡ΠΊΡƒ 2. ΠŸΠΎΡΡ‚Ρ€ΠΎΠΈΠΌ Π² этих Ρ‚ΠΎΡ‡ΠΊΠ°Ρ… ΠΊΠ°ΡΠ°Ρ‚Π΅Π»ΡŒΠ½Ρ‹Π΅ Π΅Π΄ΠΈΠ½ΠΈΡ‡Π½Ρ‹Π΅ Π²Π΅ΠΊΡ‚ΠΎΡ€Ρ‹ 1 ΠΈ 2. ΠŸΠ΅Ρ€ΠΏΠ΅Π½Π΄ΠΈΠΊΡƒΠ»ΡΡ€Ρ‹ ΠΊ этим ΠΊΠ°ΡΠ°Ρ‚Π΅Π»ΡŒΠ½Ρ‹ΠΌ пСрСсСкутся Π² Π½Π΅ΠΊΠΎΡ‚ΠΎΡ€ΠΎΠΉ Ρ‚ΠΎΡ‡ΠΊΠ΅ O2. Π—Π°ΠΌΠ΅Ρ‚ΠΈΠΌ, Ρ‡Ρ‚ΠΎ для ΠΊΡ€ΠΈΠ²ΠΎΠΉ, Π½Π΅ ΡΠ²Π»ΡΡŽΡ‰Π΅ΠΉΡΡ ΠΎΠΊΡ€ΡƒΠΆΠ½ΠΎΡΡ‚ΡŒΡŽ, расстояния R1 ΠΈ R2 Π±ΡƒΠ΄ΡƒΡ‚ Π½Π΅ΠΌΠ½ΠΎΠ³ΠΎ ΠΎΡ‚Π»ΠΈΡ‡Π°Ρ‚ΡŒΡΡ Π΄Ρ€ΡƒΠ³ ΠΎΡ‚ Π΄Ρ€ΡƒΠ³Π°. Если Ρ‚Π΅ΠΏΠ΅Ρ€ΡŒ Ρ‚ΠΎΡ‡ΠΊΡƒ 2 ΠΏΡ€ΠΈΠ±Π»ΠΈΠΆΠ°Ρ‚ΡŒ ΠΊ Ρ‚ΠΎΡ‡ΠΊΠ΅ 1, пСрСсСчСниС пСрпСндикуляров O2 Π±ΡƒΠ΄Π΅Ρ‚ ΠΏΠ΅Ρ€Π΅ΠΌΠ΅Ρ‰Π°Ρ‚ΡŒΡΡ вдоль прямой O21 ΠΈ Π² ΠΏΡ€Π΅Π΄Π΅Π»Π΅ окаТСтся Π² Π½Π΅ΠΊΠΎΡ‚ΠΎΡ€ΠΎΠΉ Ρ‚ΠΎΡ‡ΠΊΠ΅ O1. Расстояния R1 ΠΈ R2 Π±ΡƒΠ΄ΡƒΡ‚ ΡΡ‚Ρ€Π΅ΠΌΠΈΡ‚ΡŒΡΡ ΠΊ ΠΎΠ±Ρ‰Π΅ΠΌΡƒ ΠΏΡ€Π΅Π΄Π΅Π»Ρƒ R, Ρ€Π°Π²Π½ΠΎΠΌΡƒ радиусу ΠΊΡ€ΠΈΠ²ΠΈΠ·Π½Ρ‹, Π° Ρ‚ΠΎΡ‡ΠΊΠ° O1 ΠΈ Π±ΡƒΠ΄Π΅Ρ‚ Ρ†Π΅Π½Ρ‚Ρ€ΠΎΠΌ ΠΊΡ€ΠΈΠ²ΠΈΠ·Π½Ρ‹ для Ρ‚ΠΎΡ‡ΠΊΠΈ 1. Π”Π΅ΠΉΡΡ‚Π²ΠΈΡ‚Π΅Π»ΡŒΠ½ΠΎ, ΠΎΠΊΡ€ΡƒΠΆΠ½ΠΎΡΡ‚ΡŒ радиусом R с Ρ†Π΅Π½Ρ‚Ρ€ΠΎΠΌ Π² 0 ΠΏΡ€ΠΎΡ…ΠΎΠ΄ΠΈΡ‚ Ρ‡Π΅Ρ€Π΅Π· Ρ‚ΠΎΡ‡ΠΊΡƒ 1 ΠΈ касаСтся Ρ‚Ρ€Π°Π΅ΠΊΡ‚ΠΎΡ€ΠΈΠΈ (Ρ‚Π°ΠΊ ΠΊΠ°ΠΊ радиус ΠΎΡ€Ρ‚ΠΎΠ³ΠΎΠ½Π°Π»Π΅Π½ ΠΎΡ€Ρ‚Ρƒ 1). ΠšΡ€ΠΎΠΌΠ΅ Ρ‚ΠΎΠ³ΠΎ, ΠΏΠΎ ΠΏΠΎΡΡ‚Ρ€ΠΎΠ΅Π½ΠΈΡŽ бСсконСчно близкая Ρ‚ΠΎΡ‡ΠΊΠ° 2 Ρ‚Π°ΠΊΠΆΠ΅ Π»Π΅ΠΆΠΈΡ‚ Π½Π° этой окруТности. Π’Π°ΠΊΠΈΠΌ ΠΎΠ±Ρ€Π°Π·ΠΎΠΌ, построСнная ΠΎΠΊΡ€ΡƒΠΆΠ½ΠΎΡΡ‚ΡŒ Π΄Π΅ΠΉΡΡ‚Π²ΠΈΡ‚Π΅Π»ΡŒΠ½ΠΎ «сливаСтся» с Ρ‚Ρ€Π°Π΅ΠΊΡ‚ΠΎΡ€ΠΈΠ΅ΠΉ Π² Ρ‚ΠΎΡ‡ΠΊΠ΅ 1.

Π˜Ρ‚Π°ΠΊ, Π² ΠΎΠ±Ρ‰Π΅ΠΌ случаС ускорСниС ΠΈΠΌΠ΅Π΅Ρ‚ Π΄Π²Π΅ ΡΠΎΡΡ‚Π°Π²Π»ΡΡŽΡ‰ΠΈΠ΅ β€” Ρ‚Π°Π½Π³Π΅Π½Ρ†ΠΈΠ°Π»ΡŒΠ½ΡƒΡŽ

Π½Π°ΠΏΡ€Π°Π²Π»Π΅Π½Π½ΡƒΡŽ вдоль ΠΊΠ°ΡΠ°Ρ‚Π΅Π»ΡŒΠ½ΠΎΠΉ ΠΈ ΠΎΠΏΡ€Π΅Π΄Π΅Π»ΡΡŽΡ‰ΡƒΡŽ ΡΠΊΠΎΡ€ΠΎΡΡ‚ΡŒ измСнСния модуля Π²Π΅ΠΊΡ‚ΠΎΡ€Π° скорости Π½ΠΎΡ€ΠΌΠ°Π»ΡŒΠ½ΡƒΡŽ

Π½Π°ΠΏΡ€Π°Π²Π»Π΅Π½Π½ΡƒΡŽ пСрпСндикулярно скорости ΠΊ Ρ†Π΅Π½Ρ‚Ρ€Ρƒ ΠΊΡ€ΠΈΠ²ΠΈΠ·Π½Ρ‹ Ρ‚Ρ€Π°Π΅ΠΊΡ‚ΠΎΡ€ΠΈΠΈ ΠΈ ΠΏΡ€ΠΎΠΏΠΎΡ€Ρ†ΠΈΠΎΠ½Π°Π»ΡŒΠ½ΡƒΡŽ ΡƒΠ³Π»ΠΎΠ²ΠΎΠΉ скорости вращСния Π²Π΅ΠΊΡ‚ΠΎΡ€Π° скорости ΠΏΡ€ΠΈ Π΄Π²ΠΈΠΆΠ΅Π½ΠΈΠΈ частицы вдоль ΠΊΡ€ΠΈΠ²ΠΎΠ»ΠΈΠ½Π΅ΠΉΠ½ΠΎΠΉ Ρ‚Ρ€Π°Π΅ΠΊΡ‚ΠΎΡ€ΠΈΠΈ (рис. 2.14).

Рис. 2.14. Π’Π°Π½Π³Π΅Π½Ρ†ΠΈΠ°Π»ΡŒΠ½ΠΎΠ΅ ΠΈ Π½ΠΎΡ€ΠΌΠ°Π»ΡŒΠ½ΠΎΠ΅ ускорСния ΠΏΡ€ΠΈ ускорСнном ΠΊΡ€ΠΈΠ²ΠΎΠ»ΠΈΠ½Π΅ΠΉΠ½ΠΎΠΌ Π΄Π²ΠΈΠΆΠ΅Π½ΠΈΠΈ.

Π”Π΅ΠΉΡΡ‚Π²ΠΈΡ‚Π΅Π»ΡŒΠ½ΠΎ , Π³Π΄Π΅ ΠΈ Π΅ΡΡ‚ΡŒ угловая ΡΠΊΠΎΡ€ΠΎΡΡ‚ΡŒ вращСния Π²Π΅ΠΊΡ‚ΠΎΡ€Π° скорости .

ПолноС ускорСниС

опрСдСляСтся ΠΏΠΎ ΠΏΡ€Π°Π²ΠΈΠ»Ρƒ ΠΏΠ°Ρ€Π°Π»Π»Π΅Π»ΠΎΠ³Ρ€Π°ΠΌΠΌΠ°. ΠœΠΎΠ΄ΡƒΠ»ΡŒ ΠΏΠΎΠ»Π½ΠΎΠ³ΠΎ ускорСния Π² соотвСтствии с Ρ‚Π΅ΠΎΡ€Π΅ΠΌΠΎΠΉ ΠŸΠΈΡ„Π°Π³ΠΎΡ€Π° Ρ€Π°Π²Π΅Π½

Π’Ρ‹ΠΏΠΈΡˆΠ΅ΠΌ Π±Π΅Π· Π²Ρ‹Π²ΠΎΠ΄Π° Ρ„ΠΎΡ€ΠΌΡƒΠ»Ρ‹, ΡΠ²ΡΠ·Ρ‹Π²Π°ΡŽΡ‰ΠΈΠ΅ радиус ΠΊΡ€ΠΈΠ²ΠΈΠ·Π½Ρ‹ плоской Ρ‚Ρ€Π°Π΅ΠΊΡ‚ΠΎΡ€ΠΈΠΈ с ΠΊΠΎΠΎΡ€Π΄ΠΈΠ½Π°Ρ‚Π°ΠΌΠΈ Ρ‚Ρ€Π°Π΅ΠΊΡ‚ΠΎΡ€ΠΈΠΈ. Если извСстна Π·Π°Π²ΠΈΡΠΈΠΌΠΎΡΡ‚ΡŒ y = y(x), Ρ‚ΠΎ

Если ΠΆΠ΅ траСктория Π·Π°Π΄Π°Π½Π° Π² парамСтричСском Π²ΠΈΠ΄Π΅, x = x(t), y = y(t), Ρ‚ΠΎ

ΠŸΡ€ΠΈΠΌΠ΅Ρ€ ΠΊΡ€ΠΈΠ²ΠΎΠ»ΠΈΠ½Π΅ΠΉΠ½ΠΎΠ³ΠΎ двиТСния с постоянным ускорСниСм (Ρ‚Π΅Π»ΠΎ, Π±Ρ€ΠΎΡˆΠ΅Π½Π½ΠΎΠ΅ ΠΏΠΎΠ΄ ΡƒΠ³Π»ΠΎΠΌ ΠΊ Π³ΠΎΡ€ΠΈΠ·ΠΎΠ½Ρ‚Ρƒ) ΠΏΡ€ΠΈΠ²Π΅Π΄Π΅Π½ Π½Π° ΡΠ»Π΅Π΄ΡƒΡŽΡ‰Π΅ΠΌ рисункС:

Рис. 2.15. Π”Π²ΠΈΠΆΠ΅Π½ΠΈΠ΅ Ρ‚Π΅Π»Π°, Π±Ρ€ΠΎΡˆΠ΅Π½Π½ΠΎΠ³ΠΎ ΠΏΠΎΠ΄ ΡƒΠ³Π»ΠΎΠΌ ΠΊ Π³ΠΎΡ€ΠΈΠ·ΠΎΠ½Ρ‚Ρƒ

РасчСт скорости ΠΈ ускорСний Ρ‚ΠΎΡ‡ΠΊΠΈ Π² Π·Π°Π΄Π°Π½Π½Ρ‹ΠΉ ΠΌΠΎΠΌΠ΅Π½Ρ‚ Π²Ρ€Π΅ΠΌΠ΅Π½ΠΈ

ΠŸΡ€ΠΈΠΌΠ΅Ρ€ Ρ€Π΅ΡˆΠ΅Π½ΠΈΡ Π·Π°Π΄Π°Ρ‡ΠΈ ΠΏΠΎ ΠΎΠΏΡ€Π΅Π΄Π΅Π»Π΅Π½ΠΈΡŽ Π² Π·Π°Π΄Π°Π½Π½Ρ‹ΠΉ ΠΌΠΎΠΌΠ΅Π½Ρ‚ Π²Ρ€Π΅ΠΌΠ΅Π½ΠΈ скорости, ΠΏΠΎΠ»Π½ΠΎΠ³ΠΎ, ΠΊΠ°ΡΠ°Ρ‚Π΅Π»ΡŒΠ½ΠΎΠ³ΠΎ, Π½ΠΎΡ€ΠΌΠ°Π»ΡŒΠ½ΠΎΠ³ΠΎ ускорСний, радиуса ΠΊΡ€ΠΈΠ²ΠΈΠ·Π½Ρ‹ ΠΈ Π²ΠΈΠ΄Π° Ρ‚Ρ€Π°Π΅ΠΊΡ‚ΠΎΡ€ΠΈΠΈ Ρ‚ΠΎΡ‡ΠΊΠΈ ΠΏΠΎ извСстным уравнСниям Π΅Ρ‘ двиТСния Π² ΠΊΠΎΠΎΡ€Π΄ΠΈΠ½Π°Ρ‚Π½ΠΎΠΉ Ρ„ΠΎΡ€ΠΌΠ΅.

Π—Π°Π΄Π°Ρ‡Π°

Π”Π°Π½Ρ‹ уравнСния двиТСния Ρ‚ΠΎΡ‡ΠΊΠΈ M:

x=(3cos2Ο€t/3)+3 см;
y=(2sin2Ο€t/3)-2 см.

ΠžΠΏΡ€Π΅Π΄Π΅Π»ΠΈΡ‚ΡŒ Π²ΠΈΠ΄ Ρ‚Ρ€Π°Π΅ΠΊΡ‚ΠΎΡ€ΠΈΠΈ ΠΈ Π² ΠΌΠΎΠΌΠ΅Π½Ρ‚ Π²Ρ€Π΅ΠΌΠ΅Π½ΠΈ t=1Β c Π½Π°ΠΉΡ‚ΠΈ ΡΠΊΠΎΡ€ΠΎΡΡ‚ΡŒ Ρ‚ΠΎΡ‡ΠΊΠΈ, ΠΏΠΎΠ»Π½ΠΎΠ΅, ΠΊΠ°ΡΠ°Ρ‚Π΅Π»ΡŒΠ½ΠΎΠ΅, Π½ΠΎΡ€ΠΌΠ°Π»ΡŒΠ½ΠΎΠ΅ ускорСния ΠΈ радиус ΠΊΡ€ΠΈΠ²ΠΈΠ·Π½Ρ‹ Ρ‚Ρ€Π°Π΅ΠΊΡ‚ΠΎΡ€ΠΈΠΈ Π² Π΄Π°Π½Π½ΠΎΠΉ Ρ‚ΠΎΡ‡ΠΊΠ΅.

Π”Ρ€ΡƒΠ³ΠΈΠ΅ ΠΏΡ€ΠΈΠΌΠ΅Ρ€Ρ‹ Ρ€Π΅ΡˆΠ΅Π½ΠΈΠΉ >
ΠŸΠΎΠΌΠΎΡ‰ΡŒ с Ρ€Π΅ΡˆΠ΅Π½ΠΈΠ΅ΠΌ Π·Π°Π΄Π°Ρ‡ >

РСшСниС

ΠšΠΎΠΎΡ€Π΄ΠΈΠ½Π°Ρ‚Π½Ρ‹ΠΉ способ задания двиТСния – это траСктория двиТСния Ρ‚ΠΎΡ‡ΠΊΠΈ Π² парамСтричСской Ρ„ΠΎΡ€ΠΌΠ΅.

Π˜ΡΠΊΠ»ΡŽΡ‡ΠΈΠΌ ΠΏΠ°Ρ€Π°ΠΌΠ΅Ρ‚Ρ€ t:

cos2Ο€t/3=(x-3)/3,
sin2Ο€t/3=(y+2)/2,
(cos2Ο€t/3)2+(sin2Ο€t/3)2=1=((x-3)2/32)+((y+2)2/22),
((x-3)2/32)+((y+2)2/22)=1

ΠΏΠΎΠ»ΡƒΡ‡ΠΈΠ»ΠΈ эллипс с полуосями 3 см ΠΈ 2 см (рисунок 1.7).

Наш Π²ΠΈΠ΄Π΅ΠΎΡƒΡ€ΠΎΠΊ ΠΏΠΎ Ρ‚Π΅ΠΌΠ΅:

Π’ ΠΌΠΎΠΌΠ΅Π½Ρ‚ Π²Ρ€Π΅ΠΌΠ΅Π½ΠΈ t=1Β c ΠΊΠΎΠΎΡ€Π΄ΠΈΠ½Π°Ρ‚Ρ‹ Ρ‚ΠΎΡ‡ΠΊΠΈ:

x=(3cos2Ο€βˆ™1/3)+3=3cos120o+3=3βˆ™(-1/2)+3=1,5 см;
y=(2sin2Ο€βˆ™1/3)-2=2sin120o-2=2βˆ™(31/2/2)-2=-0,27 см.

Рисунок 1.7

Π”Π²ΠΈΠΆΠ΅Π½ΠΈΠ΅ начинаСтся ΠΈΠ· Ρ‚ΠΎΡ‡ΠΊΠΈ A:

xt=0=(3cos2Ο€βˆ™0/3)+3=3+3=6 см;
yt=0=(2sin2Ο€βˆ™0/3)-2=0-2=-2 см.

Учитывая Π³Ρ€Π°Ρ„ΠΈΠΊΠΈ измСнСния Ρ„ΡƒΠ½ΠΊΡ†ΠΈΠΉ синуса ΠΈ косинуса, ΠΌΠΎΠΆΠ½ΠΎ ΡƒΡ‚Π²Π΅Ρ€ΠΆΠ΄Π°Ρ‚ΡŒ, Ρ‡Ρ‚ΠΎ Ρ‚ΠΎΡ‡ΠΊΠ° M двиТСтся ΠΏΠΎ эллипсу ΠΈΠ· Ρ‚ΠΎΡ‡ΠΊΠΈ A ΠΏΡ€ΠΎΡ‚ΠΈΠ² Ρ…ΠΎΠ΄Π° часовой стрСлки.

Π‘ΠΊΠΎΡ€ΠΎΡΡ‚ΡŒ Ρ‚ΠΎΡ‡ΠΊΠΈ:

Vx=dx/dt=d((3cos2Ο€t/3)+3)/dt=

=3βˆ™(-sin2Ο€t/3)βˆ™2Ο€/3+0=-2Ο€sin2Ο€t/3 см/с;
Vy=dy/dt=d((2sin2Ο€t/3)-2)/dt=

=2βˆ™(cos2Ο€t/3)βˆ™2Ο€/3-0=(4/3)Ο€cos2Ο€t/3 см/с.

Π’ ΠΌΠΎΠΌΠ΅Π½Ρ‚ Π²Ρ€Π΅ΠΌΠ΅Π½ΠΈ t=1:

НаправлСниС Π²Π΅ΠΊΡ‚ΠΎΡ€Π° скорости опрСдСляСтся Π½Π°ΠΏΡ€Π°Π²Π»ΡΡŽΡ‰ΠΈΠΌΠΈ косинусами:

Π’Π°ΠΊΠΈΠΌ ΠΎΠ±Ρ€Π°Π·ΠΎΠΌ, Π²Π΅ΠΊΡ‚ΠΎΡ€ скорости ΠΎΠΏΡ€Π΅Π΄Π΅Π»Π΅Π½ ΠΈ ΠΏΠΎ Π²Π΅Π»ΠΈΡ‡ΠΈΠ½Π΅ ΠΈ ΠΏΠΎ Π½Π°ΠΏΡ€Π°Π²Π»Π΅Π½ΠΈΡŽ (рисунок 1.8).

УскорСниС Ρ‚ΠΎΡ‡ΠΊΠΈ:

Для t=1Β c:

ПолноС ускорСниС:


Рисунок 1.8

НаправлСниС Π²Π΅ΠΊΡ‚ΠΎΡ€Π° ускорСния:

Π Π΅Π·ΡƒΠ»ΡŒΡ‚Π°Ρ‚Ρ‹ расчСтов ΠΏΠΎΠΊΠ°Π·Π°Π½Ρ‹ Π½Π° рисункС 1.8.

ΠšΠ°ΡΠ°Ρ‚Π΅Π»ΡŒΠ½ΠΎΠ΅ ускорСниС опрСдСляСтся ΠΏΠΎ Ρ„ΠΎΡ€ΠΌΡƒΠ»Π΅ (1.11):

ΠΠΎΡ€ΠΌΠ°Π»ΡŒΠ½ΠΎΠ΅ ускорСниС ΠΌΠΎΠΆΠ½ΠΎ ΠΎΠΏΡ€Π΅Π΄Π΅Π»ΠΈΡ‚ΡŒ Π»ΠΈΠ±ΠΎ ΠΈΠ· Ρ„ΠΎΡ€ΠΌΡƒΠ»Ρ‹ (1.5), Π»ΠΈΠ±ΠΎ ΠΈΠ· Ρ„ΠΎΡ€ΠΌΡƒΠ»Ρ‹ (1.12). По Ρ„ΠΎΡ€ΠΌΡƒΠ»Π΅ (1.12) ΠΏΠΎΠ»ΡƒΡ‡ΠΈΠΌ:

Π Π΅Π·ΡƒΠ»ΡŒΡ‚Π°Ρ‚ ΠΌΠΎΠΆΠ΅Ρ‚ Π±Ρ‹Ρ‚ΡŒ ΠΏΡ€ΠΎΠ²Π΅Ρ€Π΅Π½ (см. Π²Ρ‹ΡˆΠ΅ расчСт):

Радиус ΠΊΡ€ΠΈΠ²ΠΈΠ·Π½Ρ‹ Ρ‚Ρ€Π°Π΅ΠΊΡ‚ΠΎΡ€ΠΈΠΈ Π² Ρ‚ΠΎΡ‡ΠΊΠ΅ M:

ρ=V2/an=5,832/9,44=3,60 см.

Π”Ρ€ΡƒΠ³ΠΈΠ΅ ΠΏΡ€ΠΈΠΌΠ΅Ρ€Ρ‹ Ρ€Π΅ΡˆΠ΅Π½ΠΈΡ Π·Π°Π΄Π°Ρ‡Β >>

. ΠžΠΏΡ€Π΅Π΄Π΅Π»ΠΈΡ‚Π΅ Π½ΠΎΡ€ΠΌΠ°Π»ΡŒΠ½ΠΎΠ΅ ускорСниС ΠΈ радиус ΠΊΡ€ΠΈΠ²ΠΈΠ·Π½Ρ‹ Ρ‚Ρ€Π°Π΅ΠΊΡ‚ΠΎΡ€ΠΈΠΈ Ρ‚Π΅Π»Π°, двиТущСйся с ускорСниСм

НомСра 79 Π± ΠΈ 89 Π± поТалуйста

НомСра 59 Π± ΠΈ 69 Π± поТалуйста

ΠŸΠΎΠΌΠΎΠ³ΠΈΡ‚Π΅ с физикойВСлосипСдист ΠΈΒ Π°Π²Ρ‚ΠΎΠΌΠΎΠ±ΠΈΠ»ΡŒ ΠΏΠΎΠ΄ΡŠΠ΅Π·ΠΆΠ°ΡŽΡ‚ по пСрпСндикулярным Π΄ΠΎΡ€ΠΎΠ³Π°ΠΌ к пСрСкрёстку. Когда вСлосипСдист ΠΏΡ€ΠΎΠ΅Π·ΠΆΠ°Π» пСрСкрёсток, Π°Π²Ρ‚ΠΎΠΌΠΎΠ±ΠΈΠ»ΡŒ … Π΅Ρ‰Ρ‘ Π½Π΅Β Π΄ΠΎΠ΅Ρ…Π°Π» до пСрСкрёстка 100Β ΠΌ. КакоС врСмя послС этого ΠΏΡ€ΠΎΠΉΠ΄Ρ‘Ρ‚ Π΄ΠΎΒ ΠΌΠΎΠΌΠ΅Π½Ρ‚Π°, ΠΊΠΎΠ³Π΄Π° вСлосипСдист ΠΈΒ Π°Π²Ρ‚ΠΎΠΌΠΎΠ±ΠΈΠ»ΡŒ окаТутся на минимальном расстоянии, Ссли ΠΈΡ… скорости постоянны ΠΈΒ Ρ€Π°Π²Π½Ρ‹Β V1=7Β ΠΌ/с у вСлосипСдиста ΠΈΒ V2=24Β ΠΌ/с у автомобиля? ΠžΡ‚Π²Π΅Ρ‚ Π·Π°ΠΏΠΈΡˆΠΈΡ‚Π΅ в сСкундах, ΠΎΠΊΡ€ΡƒΠ³Π»ΠΈΠ² до сотых.​

Π”Π²Π° Ρ‡Π΅Π»ΠΎΠ²Π΅ΠΊΠ° стоят Π½Π° расстоянии h2 ΠΈ h3 ΠΎΡ‚ стСнки ΠΈ l β€”Π΄Ρ€ΡƒΠ³ ΠΎΡ‚ Π΄Ρ€ΡƒΠ³Π°. Один ΠΈΠ· Π½ΠΈΡ… сказал слово, Π΄Ρ€ΡƒΠ³ΠΎΠΉ ΡƒΡΠ»Ρ‹ΡˆΠ°Π» ΠΊΠΎΠ½Π΅Ρ† слова, совпавшСС с Π½Π°Ρ‡Π°Π»ΠΎΠΌ эха э … Ρ‚ΠΎΠ³ΠΎ ΠΆΠ΅ слова. Π‘ΠΊΠΎΡ€ΠΎΡΡ‚ΡŒ Π·Π²ΡƒΠΊΠ° c. ΠžΠΏΡ€Π΅Π΄Π΅Π»ΠΈΡ‚Π΅ Π΄Π»ΠΈΡ‚Π΅Π»ΡŒΠ½ΠΎΡΡ‚ΡŒ звучания слова.

ΠŸΡ€ΠΈ Ρ‚Π΅ΠΌΠΏΠ΅Ρ€Π°Ρ‚ΡƒΡ€Π΅ 10 Β°Π‘ ΠΎΡ‚Π½ΠΎΡΠΈΡ‚Π΅Π»ΡŒΠ½Π°Ρ Π²Π»Π°ΠΆΠ½ΠΎΡΡ‚ΡŒ Π²ΠΎΠ·Π΄ΡƒΡ…Π° Π±Ρ‹Π»Π° Ρ€Π°Π²Π½Π° 72 %. Π˜ΡΠΏΠΎΠ»ΡŒΠ·ΡƒΡ Π΄Π°Π½Π½Ρ‹Π΅ Ρ‚Π°Π±Π»ΠΈΡ†Ρ‹, ΠΎΠΏΡ€Π΅Π΄Π΅Π»ΠΈ, Π½Π° сколько ΡƒΠΌΠ΅Π½ΡŒΡˆΠΈΠ»Π°ΡΡŒ ΠΈΠ»ΠΈ ΡƒΠ²Π΅Π»ΠΈΡ‡ΠΈΠ»Π°ΡΡŒ ΠΎΡ‚Π½ΠΎ … ΡΠΈΡ‚Π΅Π»ΡŒΠ½Π°Ρ Π²Π»Π°ΠΆΠ½ΠΎΡΡ‚ΡŒ Π²ΠΎΠ·Π΄ΡƒΡ…Π°, Ссли Ρ‚Π΅ΠΌΠΏΠ΅Ρ€Π°Ρ‚ΡƒΡ€Ρƒ повысили Π΄ΠΎ 20 Β°Π‘.ΠžΡ‚Π²Π΅Ρ‚ (ΠΎΠΊΡ€ΡƒΠ³Π»ΠΈ Π΄ΠΎ дСсятых): ΠΎΡ‚Π½ΠΎΡΠΈΡ‚Π΅Π»ΡŒΠ½Π°Ρ Π²Π»Π°ΠΆΠ½ΠΎΡΡ‚ΡŒ Π²ΠΎΠ·Π΄ΡƒΡ…Π° Π½Π° %.t, Β°Π‘ p, кПа ρ0, Π³ΠΌ310 1,23 9,411 1,33 1012 1,4 10,713 1,49 11,414 1,6 12,115 1,71 12,816 1,81 13,617 1,93 14,518 2,07 15,419 2,2 16,320 2,33 17,3​

ΠžΠΏΡ€Π΅Π΄Π΅Π»ΠΈ массу ΠΏΠ°Ρ€ΠΎΠ² Π²ΠΎΠ΄Ρ‹ Π² Π²ΠΎΠ·Π΄ΡƒΡ…Π΅ Π°ΠΊΡ‚ΠΎΠ²ΠΎΠ³ΠΎ Π·Π°Π»Π° ΠΎΠ±ΡŠΡ‘ΠΌΠΎΠΌ 52 ΠΌΒ³ ΠΏΡ€ΠΈ Ρ‚Π΅ΠΌΠΏΠ΅Ρ€Π°Ρ‚ΡƒΡ€Π΅ Π²ΠΎΠ·Π΄ΡƒΡ…Π° 24 Β°Π‘ ΠΈ ΠΎΡ‚Π½ΠΎΡΠΈΡ‚Π΅Π»ΡŒΠ½ΠΎΠΉ влаТности 62 %. ΠŸΠ»ΠΎΡ‚Π½ΠΎΡΡ‚ΡŒ насыщСнного ΠΏΠ° … Ρ€Π° Ρ€Π°Π²Π½Π° 17,3 Π³ΠΌ3. ΠžΡ‚Π²Π΅Ρ‚ (ΠΎΠΊΡ€ΡƒΠ³Π»ΠΈ Π΄ΠΎ дСсятых): Π³. ​

8 класс. ΠΏΠΎΡ‡Π΅ΠΌΡƒ Π² Ρ€Π΅ΡˆΠ΅Π½ΠΈΠΈ Π·Π°Π΄Π°Ρ‡ΠΈ Π΄Π²Π° Π²Ρ€Π΅ΠΌΠ΅Π½ΠΈ ΠΎΠ΄ΠΈΠ½Π°ΠΊΠΎΠ²Ρ‹Π΅?сайт: http://fizportal.ru/kinematics-problem-1-1-6Π·Π°Π΄Π°Ρ‡Π°: БпортсмСны Π±Π΅Π³ΡƒΡ‚ ΠΊΠΎΠ»ΠΎΠ½Π½ΠΎΠΉ Π΄Π»ΠΈΠ½Ρ‹ l со … ΡΠΊΠΎΡ€ΠΎΡΡ‚ΡŒΡŽ v. НавстрСчу Π±Π΅ΠΆΠΈΡ‚ Ρ‚Ρ€Π΅Π½Π΅Ρ€ со ΡΠΊΠΎΡ€ΠΎΡΡ‚ΡŒΡŽ uΡ€Π΅ΡˆΠ΅Π½ΠΈΠ΅:ΠœΡ‹ΡΠ»Π΅Π½Π½ΠΎ остановим ΠΊΠΎΠ»ΠΎΠ½Ρƒ, Ρ‚ΠΎΠ³Π΄Π° Ρ‚Ρ€Π΅Π½Π΅Ρ€ ΠΊΡ€ΠΎΠΌΠ΅ своСй скорости Π±ΡƒΠ΄Π΅Ρ‚ ΠΈΠΌΠ΅Ρ‚ΡŒ ΡΠΊΠΎΡ€ΠΎΡΡ‚ΡŒ ΠΊΠΎΠ»ΠΎΠ½Π½Ρ‹, Π½Π°ΠΏΡ€Π°Π²Π»Π΅Π½Π½ΡƒΡŽ Π² ΠΏΡ€ΠΎΡ‚ΠΈΠ²ΠΎΠΏΠΎΠ»ΠΎΠΆΠ½ΡƒΡŽ сторону. Π‘ этой ΠΎΡ‚Π½ΠΎΡΠΈΡ‚Π΅Π»ΡŒΠ½ΠΎΠΉ ΡΠΊΠΎΡ€ΠΎΡΡ‚ΡŒΡŽ v+u ΠΎΠ½ Π² Ρ‚Π΅Ρ‡Π΅Π½ΠΈΠ΅ Π²Ρ€Π΅ΠΌΠ΅Π½ΠΈt=lv+u,ΠΏΡ€ΠΎΠ±Π΅ΠΆΠΈΡ‚ вдоль ΠΊΠΎΠ»ΠΎΠ½Π½Ρ‹ ΠΈ сравняСтся с хвостом. Π“ΠΎΠ»ΠΎΠ²Π° ΠΊΠΎΠ»ΠΎΠ½Ρ‹, Ρ€Π°Π·Π²Π΅Ρ€Π½ΡƒΠ²ΡˆΠΈΡΡŒ, Π±ΡƒΠ΄Π΅Ρ‚ Π΄Π²ΠΈΠ³Π°Ρ‚ΡŒΡΡ с ΠΎΡ‚Π½ΠΎΡΠΈΡ‚Π΅Π»ΡŒΠ½ΠΎΠΉ ΡΠΊΠΎΡ€ΠΎΡΡ‚ΡŒΡŽ vβˆ’u ΠΈ Ρ‡Π΅Ρ€Π΅Π· врСмяt=l/vβˆ’uΠ³Π΄Π΅ l/ – Π΄Π»ΠΈΠ½Π° Π½ΠΎΠ²ΠΎΠΉ Β«ΠΊΠΎΠ»ΠΎΠ½Ρ‹Β» послС ΠΎΠ±Π³ΠΎΠ½Π°. Π’ΠΎΠ³Π΄Π°l/vβˆ’u=lv+uΠΈ Π΄Π»ΠΈΠ½Π° Π½ΠΎΠ²ΠΎΠΉ ΠΊΠΎΠ»ΠΎΠ½Π½Ρ‹l/=vβˆ’uv+ul.ΠžΡ‚Π²Π΅Ρ‚: ΠΊΠΎΠ³Π΄Π° всС спортсмСны Ρ€Π°Π·Π²Π΅Ρ€Π½ΡƒΡ‚ΡŒΡΡ Π΄Π»ΠΈΠ½Π° Π½ΠΎΠ²ΠΎΠΉ ΠΊΠΎΠ»ΠΎΠ½Ρ‹ Π±ΡƒΠ΄Π΅Ρ‚ Ρ€Π°Π²Π½Π° l/=vβˆ’uv+ul

Π›Ρ–Ρ‚Π°ΠΊ ΠΏΡ–Π΄Π½Ρ–ΠΌΠ°Ρ”Ρ‚ΡŒΡΡ Π²Π³ΠΎΡ€Ρƒ Π· ΡˆΠ²ΠΈΠ΄ΠΊΡ–ΡΡ‚ΡŽ v = 700 ΠΊΠΌ/Π³ΠΎΠ΄ Ρ– Π·Π° t = 4 Ρ…Π² досягає висоти h = 4 ΠΊΠΌ. Π— якою ΡˆΠ²ΠΈΠ΄ΠΊΡ–ΡΡ‚ΡŽ Ρ€ΡƒΡ…Π°Ρ”Ρ‚ΡŒΡΡ ΠΏΠΎ ΠΏΠΎΠ²Π΅Ρ€Ρ…Π½Ρ– Π—Π΅ΠΌΠ»Ρ– Ρ‚Ρ–Π½ΡŒ Π²Ρ–Π΄ Π»Ρ–Ρ‚ … Π°ΠΊΠ°, якщо Π‘ΠΎΠ½Ρ†Π΅ Π² Π·Π΅Π½Ρ–Ρ‚Ρ–?

ΠŸΠΎΠΌΠΎΠ³ΠΈΡ‚Π΅ Ρ€Π΅ΡˆΠΈΡ‚ΡŒ Π·Π°Π΄Π°Ρ‡Ρƒ ΠΏΠΎ ΠΊΠΈΠ½Π΅ΠΌΠ°Ρ‚ΠΈΠΊΠ΅: Из Π³ΠΎΡ€ΠΎΠ΄Π° А Π²Ρ‹Π΅Ρ…Π°Π»ΠΈ с ΠΎΠ΄ΠΈΠ½Π°ΠΊΠΎΠ²Ρ‹ΠΌΠΈ скоростями Π΄Π²Π° автомобиля, Π²Ρ‚ΠΎΡ€ΠΎΠΉ Ρ‡Π΅Ρ€Π΅Π· 12 ΠΌΠΈΠ½ΡƒΡ‚ послС ΠΏΠ΅Ρ€Π²ΠΎΠ³ΠΎ. Они ΠΏΠΎΠΎΡ‡Π΅Ρ€Π΅Π΄Π½ΠΎ … , с ΠΈΠ½Ρ‚Π΅Ρ€Π²Π°Π»ΠΎΠΌ Π² 14 ΠΌΠΈΠ½ΡƒΡ‚, ΠΎΠ±ΠΎΠ³Π½Π°Π»ΠΈ ΠΎΠ΄Π½ΠΎΠ³ΠΎ ΠΈ Ρ‚ΠΎΠ³ΠΎ ΠΆΠ΅ вСлосипСдиста. Π’ΠΎ сколько Ρ€Π°Π· ΡΠΊΠΎΡ€ΠΎΡΡ‚ΡŒ Π°Π²Ρ‚ΠΎΠΌΠΎΠ±ΠΈΠ»Π΅ΠΉ большС скорости вСлосипСдиста? (ΠšΠΈΡ€ΠΈΠΊ – с/Ρ€ ΠΈ ΠΊ/Ρ€, 9 класс)

На Π³Π»Π°Π΄ΠΊΠΎΠΉ Π³ΠΎΡ€ΠΈΠ·ΠΎΠ½Ρ‚Π°Π»ΡŒΠ½ΠΎΠΉ повСрхности покоится ΡƒΠ³ΠΎΠ»ΠΎΠΊ массы M, ΠΊΠΎΡ‚ΠΎΡ€Ρ‹ΠΉ с ΠΏΠΎΠΌΠΎΡ‰ΡŒΡŽ Π»Ρ‘Π³ΠΊΠΎΠΉ Π½ΠΈΡ‚ΠΈ ΠΈ Π΄Π²ΡƒΡ… Π±Π»ΠΎΠΊΠΎΠ² соСдинён со стСнкой ΠΈ бруском массы m (см. … рисунок). Брусок касаСтся Π²Π½ΡƒΡ‚- Ρ€Π΅Π½Π½Π΅ΠΉ повСрхности ΡƒΠ³ΠΎΠ»ΠΊΠ°. Нити, ΠΏΠ΅Ρ€Π΅ΠΊΠΈΠ½ΡƒΡ‚Ρ‹Π΅ Ρ‡Π΅Ρ€Π΅Π· Π±Π»ΠΎΠΊ, ΠΏΡ€ΠΈΠΊΡ€Π΅ΠΏ- Π»Ρ‘Π½Π½Ρ‹ΠΉ ΠΊ стСнС, натянуты Π³ΠΎΡ€ΠΈΠ·ΠΎΠ½Ρ‚Π°Π»ΡŒΠ½ΠΎ. Π’Π½Π°Ρ‡Π°Π»Π΅ систСму ΡƒΠ΄Π΅Ρ€ΠΆΠΈΠ²Π°ΡŽΡ‚ Π² состоянии покоя, Π° Π·Π°Ρ‚Π΅ΠΌ отпуска- ΡŽΡ‚. НайдитС ускорСниС a ΡƒΠ³ΠΎΠ»ΠΊΠ°. Π‘Π»ΠΎΠΊΠΈ Π»Ρ‘Π³ΠΊΠΈΠ΅. Π’Ρ€Π΅Π½ΠΈΠ΅ Π² систСмС отсутствуСт.

Радиус ΠΊΡ€ΠΈΠ²ΠΈΠ·Π½Ρ‹ Ρ‚Ρ€Π°Π΅ΠΊΡ‚ΠΎΡ€ΠΈΠΈ

Π’ этой ΡΡ‚Π°Ρ‚ΡŒΠ΅ ΠΏΡ€ΠΈΠ²Π΅Π΄Π΅Π½Ρ‹ Π΄Π²Π΅ Π·Π°Π΄Π°Ρ‡ΠΈ, ΠΊΠΎΡ‚ΠΎΡ€Ρ‹Π΅ ΠΏΠΎΠΌΠΎΠ³ΡƒΡ‚ Π²Π°ΠΌ Π½Π°ΡƒΡ‡ΠΈΡ‚ΡŒΡΡ ΠΎΠΏΡ€Π΅Π΄Π΅Π»ΡΡ‚ΡŒ радиус ΠΊΡ€ΠΈΠ²ΠΈΠ·Π½Ρ‹ Ρ‚Ρ€Π°Π΅ΠΊΡ‚ΠΎΡ€ΠΈΠΈ ΠΏΡ€ΠΈ Π΄Π²ΠΈΠΆΠ΅Π½ΠΈΠΈ Ρ‚Π΅Π»Π° ΠΏΠΎΠ΄ ΡƒΠ³Π»ΠΎΠΌ ΠΊ Π³ΠΎΡ€ΠΈΠ·ΠΎΠ½Ρ‚Ρƒ. КаТдая ΠΈΠ· Β Π·Π°Π΄Π°Ρ‡ прСдставляСт собой Ρ†Π΅Π»Ρ‹ΠΉ Π½Π°Π±ΠΎΡ€, поэтому нСясностСй Π½Π΅ Π΄ΠΎΠ»ΠΆΠ½ΠΎ ΠΎΡΡ‚Π°Ρ‚ΡŒΡΡ.

Π—Π°Π΄Π°Ρ‡Π° 1. Π’Π΅Π»ΠΎ Π±Ρ€ΠΎΡˆΠ΅Π½ΠΎ со ΡΠΊΠΎΡ€ΠΎΡΡ‚ΡŒΡŽ 10 ΠΌ/с ΠΏΠΎΠ΄ ΡƒΠ³Π»ΠΎΠΌ ΠΊ Π³ΠΎΡ€ΠΈΠ·ΠΎΠ½Ρ‚Ρƒ. Найти радиусы ΠΊΡ€ΠΈΠ²ΠΈΠ·Π½Ρ‹ Ρ‚Ρ€Π°Π΅ΠΊΡ‚ΠΎΡ€ΠΈΠΈ Ρ‚Π΅Π»Π° Π² Π½Π°Ρ‡Π°Π»ΡŒΠ½Ρ‹ΠΉ ΠΌΠΎΠΌΠ΅Π½Ρ‚ Π΅Π³ΠΎ двиТСния, спустя врСмя 0,5 с ΠΈ Π² Ρ‚ΠΎΡ‡ΠΊΠ΅ Π½Π°ΠΈΠ²Ρ‹ΡΡˆΠ΅Π³ΠΎ подъСма Ρ‚Π΅Π»Π° Π½Π°Π΄ ΠΏΠΎΠ²Π΅Ρ€Ρ…Π½ΠΎΡΡ‚ΡŒΡŽ Π·Π΅ΠΌΠ»ΠΈ.
Как извСстно, радиус ΠΊΡ€ΠΈΠ²ΠΈΠ·Π½Ρ‹ Ρ‚Ρ€Π°Π΅ΠΊΡ‚ΠΎΡ€ΠΈΠΈ связан с Π½ΠΎΡ€ΠΌΠ°Π»ΡŒΠ½Ρ‹ΠΌ ускорСниСм ΠΈ ΡΠΊΠΎΡ€ΠΎΡΡ‚ΡŒΡŽ Ρ„ΠΎΡ€ΠΌΡƒΠ»ΠΎΠΉ:

Β  Β 

ΠžΡ‚ΠΊΡƒΠ΄Π° :

Β  Β 

Π’ΠΎ Π΅ΡΡ‚ΡŒ, Ρ‡Ρ‚ΠΎΠ±Ρ‹ Π½Π°ΠΉΡ‚ΠΈ радиус ΠΊΡ€ΠΈΠ²ΠΈΠ·Π½Ρ‹ Ρ‚Ρ€Π°Π΅ΠΊΡ‚ΠΎΡ€ΠΈΠΈ Π² любой Ρ‚ΠΎΡ‡ΠΊΠ΅, Π½Π΅ΠΎΠ±Ρ…ΠΎΠ΄ΠΈΠΌΠΎ лишь Π·Π½Π°Ρ‚ΡŒ ΡΠΊΠΎΡ€ΠΎΡΡ‚ΡŒ ΠΈ Π½ΠΎΡ€ΠΌΠ°Π»ΡŒΠ½ΠΎΠ΅ ускорСниС, Ρ‚ΠΎ Π΅ΡΡ‚ΡŒ ускорСниС, пСрпСндикулярноС Π²Π΅ΠΊΡ‚ΠΎΡ€Ρƒ скорости. Рассмотрим всС Π·Π°Π΄Π°Π½Π½Ρ‹Π΅ Ρ‚ΠΎΡ‡ΠΊΠΈ ΠΈ ΠΎΠΏΡ€Π΅Π΄Π΅Π»ΠΈΠΌ Π² Π½ΠΈΡ… скорости ΠΈ Π½ΡƒΠΆΠ½Ρ‹Π΅ ΡΠΎΡΡ‚Π°Π²Π»ΡΡŽΡ‰ΠΈΠ΅ ускорСния.

К Π·Π°Π΄Π°Ρ‡Π΅ 1

Π‘Π°ΠΌΠΎΠ΅ простоС – это ΠΎΠΏΡ€Π΅Π΄Π΅Π»Π΅Π½ΠΈΠ΅ этих Π²Π΅Π»ΠΈΡ‡ΠΈΠ½ Π² Ρ‚ΠΎΡ‡ΠΊΠ΅ Π½Π°ΠΈΠ²Ρ‹ΡΡˆΠ΅Π³ΠΎ подъСма. Π”Π΅ΠΉΡΡ‚Π²ΠΈΡ‚Π΅Π»ΡŒΠ½ΠΎ, Π²Π΅Ρ€Ρ‚ΠΈΠΊΠ°Π»ΡŒΠ½Π°Ρ ΡΠΎΡΡ‚Π°Π²Π»ΡΡŽΡ‰Π°Ρ скорости здСсь Ρ€Π°Π²Π½Π° Π½ΡƒΠ»ΡŽ, поэтому ΡΠΊΠΎΡ€ΠΎΡΡ‚ΡŒ Ρ‚Π΅Π»Π° Π² Π΄Π°Π½Π½ΠΎΠΉ Ρ‚ΠΎΡ‡ΠΊΠ΅ Ρ€Π°Π²Π½Π° Π³ΠΎΡ€ΠΈΠ·ΠΎΠ½Ρ‚Π°Π»ΡŒΠ½ΠΎΠΉ ΡΠΎΡΡ‚Π°Π²Π»ΡΡŽΡ‰Π΅ΠΉ, Π° ускорСниС, Π½ΠΎΡ€ΠΌΠ°Π»ΡŒΠ½ΠΎΠ΅ ΠΊ Π²Π΅ΠΊΡ‚ΠΎΡ€Ρƒ этой скорости – это ускорСниС свободного падСния, поэтому

Β  Β 

Вторая ΠΏΠΎ простотС расчСта – Ρ‚ΠΎΡ‡ΠΊΠ° Π½Π°Ρ‡Π°Π»Π° двиТСния. Π‘ΠΊΠΎΡ€ΠΎΡΡ‚ΡŒ Π² Π½Π΅ΠΉ Π½Π°ΠΌ ΡƒΠΆΠ΅ извСстна, ΠΎΡΡ‚Π°Π»ΠΎΡΡŒ с ускорСниСм Ρ€Π°Π·ΠΎΠ±Ρ€Π°Ρ‚ΡŒΡΡ. УскорСниС свободного падСния Ρ€Π°Π·Π»ΠΎΠΆΠΈΠΌ Π½Π° Π΄Π²Π΅ ΡΠΎΡΡ‚Π°Π²Π»ΡΡŽΡ‰ΠΈΠ΅: ΠΈ . ΠŸΠ΅Ρ€Π²Π°Ρ – пСрпСндикулярна скорости, ΠΎΠ½Π°-Ρ‚ΠΎ Π½Π°ΠΌ ΠΈ Π½ΡƒΠΆΠ½Π°. ΠžΠΏΡ€Π΅Π΄Π΅Π»ΡΠ΅ΠΌ радиус:

Β  Β 

НаконСц, Ρ‚ΠΎΡ‡ΠΊΠ°, Π² ΠΊΠΎΡ‚ΠΎΡ€ΠΎΠΉ Ρ‚Π΅Π»ΠΎ окаТСтся Ρ‡Π΅Ρ€Π΅Π· ΠΏΠΎΠ»-сСкунды.
НашС Ρ‚Π΅Π»ΠΎ Π±ΡƒΠ΄Π΅Ρ‚ Π»Π΅Ρ‚Π΅Ρ‚ΡŒ ΠΏΠΎ Π³ΠΎΡ€ΠΈΠ·ΠΎΠ½Ρ‚Π°Π»ΠΈ с постоянной ΡΠΊΠΎΡ€ΠΎΡΡ‚ΡŒΡŽ, Ρ€Π°Π²Π½ΠΎΠΉ . По Π²Π΅Ρ€Ρ‚ΠΈΠΊΠ°Π»ΠΈ Ρ‚Π΅Π»ΠΎ Π±ΡƒΠ΄Π΅Ρ‚ Π΄Π²ΠΈΠ³Π°Ρ‚ΡŒΡΡ Ρ€Π°Π²Π½ΠΎΠ·Π°ΠΌΠ΅Π΄Π»Π΅Π½Π½ΠΎ Π΄ΠΎ сСрСдины Ρ‚Ρ€Π°Π΅ΠΊΡ‚ΠΎΡ€ΠΈΠΈ (Π½Π°ΠΈΠ²Ρ‹ΡΡˆΠ΅ΠΉ Ρ‚ΠΎΡ‡ΠΊΠΈ), Π° Π·Π°Ρ‚Π΅ΠΌ равноускорСнно. ΠžΠΏΡ€Π΅Π΄Π΅Π»ΠΈΠΌ, успССт Π»ΠΈ Ρ‚Π΅Π»ΠΎ Π΄ΠΎΠ±Ρ€Π°Ρ‚ΡŒΡΡ Π΄ΠΎ апогСя:

Β  Β 

Β  Β 

Β  Β 

ΠŸΡ€ΠΎΡΡ‚ΠΎΠΉ ΠΏΡ€ΠΈΠΊΠΈΠ΄ΠΎΡ‡Π½Ρ‹ΠΉ расчСт ΠΏΠΎΠΊΠ°Π·Ρ‹Π²Π°Π΅Ρ‚, Ρ‡Ρ‚ΠΎ нуТная Π½Π°ΠΌ Ρ‚ΠΎΡ‡ΠΊΠ° находится Π½Π° ΠΏΠ΅Ρ€Π²ΠΎΠΉ ΠΏΠΎΠ»ΠΎΠ²ΠΈΠ½Π΅ Ρ‚Ρ€Π°Π΅ΠΊΡ‚ΠΎΡ€ΠΈΠΈ, Π³Π΄Π΅ Ρ‚Π΅Π»ΠΎ Π΅Ρ‰Π΅ двигаСтся Π²Π²Π΅Ρ€Ρ…. Π’ΠΎΠ³Π΄Π° Π΅Π³ΠΎ ΡΠΊΠΎΡ€ΠΎΡΡ‚ΡŒ ΠΏΠΎ оси :

Β  Β 

ΠžΠΏΡ€Π΅Π΄Π΅Π»ΠΈΠΌ ΠΏΠΎΠ»Π½ΡƒΡŽ ΡΠΊΠΎΡ€ΠΎΡΡ‚ΡŒ Ρ‚Π΅Π»Π° Π² ΠΌΠΎΠΌΠ΅Π½Ρ‚ Π²Ρ€Π΅ΠΌΠ΅Π½ΠΈ :

Β  Β 

Π£Π³ΠΎΠ» Π½Π°ΠΊΠ»ΠΎΠ½Π° Π²Π΅ΠΊΡ‚ΠΎΡ€Π° скорости ΠΊ Π³ΠΎΡ€ΠΈΠ·ΠΎΠ½Ρ‚Ρƒ Π² этот ΠΌΠΎΠΌΠ΅Π½Ρ‚ Ρ€Π°Π²Π΅Π½:

Β  Β 

А ΠΌΠΎΠΆΠ½ΠΎ Π±Ρ‹Π»ΠΎ сразу ΠΈ косинус Π½Π°ΠΉΡ‚ΠΈ:

Β  Β 

Π’ΠΎΠ³Π΄Π° искомый радиус ΠΊΡ€ΠΈΠ²ΠΈΠ·Π½Ρ‹ Ρ‚Ρ€Π°Π΅ΠΊΡ‚ΠΎΡ€ΠΈΠΈ Ρ€Π°Π²Π΅Π½:

Β  Β 

ΠžΡ‚Π²Π΅Ρ‚: ΠΌ, ΠΌ, ΠΌ.

Β 

Π—Π°Π΄Π°Ρ‡Π° 2. Под ΠΊΠ°ΠΊΠΈΠΌ ΡƒΠ³Π»ΠΎΠΌ ΠΊ Π³ΠΎΡ€ΠΈΠ·ΠΎΠ½Ρ‚Ρƒ Π½ΡƒΠΆΠ½ΠΎ Π±Ρ€ΠΎΡΠΈΡ‚ΡŒ ΡˆΠ°Ρ€ΠΈΠΊ, Ρ‡Ρ‚ΠΎΠ±Ρ‹ Π°) радиус ΠΊΡ€ΠΈΠ²ΠΈΠ·Π½Ρ‹ Ρ‚Ρ€Π°Π΅ΠΊΡ‚ΠΎΡ€ΠΈΠΈ Π² Π½Π°Ρ‡Π°Π»ΡŒΠ½Ρ‹ΠΉ ΠΌΠΎΠΌΠ΅Π½Ρ‚ Π²Ρ€Π΅ΠΌΠ΅Π½ΠΈ Π±Ρ‹Π» Π² 8 Ρ€Π°Π· большС, Ρ‡Π΅ΠΌ Π² Π²Π΅Ρ€ΡˆΠΈΠ½Π΅; Π±) Ρ†Π΅Π½Ρ‚Ρ€ ΠΊΡ€ΠΈΠ²ΠΈΠ·Π½Ρ‹ Π²Π΅Ρ€ΡˆΠΈΠ½Ρ‹ Ρ‚Ρ€Π°Π΅ΠΊΡ‚ΠΎΡ€ΠΈΠΈ находился Π±Ρ‹ Π½Π° повСрхности Π·Π΅ΠΌΠ»ΠΈ?
Π—Π°ΠΏΠΈΡˆΠ΅ΠΌ условиС Π·Π°Π΄Π°Ρ‡ΠΈ Ρ‚Π°ΠΊ: Π°) , Π±).
Π°)Как ΠΈ Π² ΠΏΡ€Π΅Π΄Ρ‹Π΄ΡƒΡ‰Π΅ΠΉ Π·Π°Π΄Π°Ρ‡Π΅, опрСдСляСм радиус ΠΊΡ€ΠΈΠ²ΠΈΠ·Π½Ρ‹ Ρ‚Ρ€Π°Π΅ΠΊΡ‚ΠΎΡ€ΠΈΠΈ Π² Ρ‚ΠΎΡ‡ΠΊΠ΅ броска. Π‘ΠΊΠΎΡ€ΠΎΡΡ‚ΡŒ Π½Π°ΠΌ извСстна, Π° Π½ΠΎΡ€ΠΌΠ°Π»ΡŒΠ½Ρ‹ΠΌ ускорСниСм Π±ΡƒΠ΄Π΅Ρ‚ проСкция ускорСния свободного падСния:

Β  Β 

ΠžΠΏΡ€Π΅Π΄Π΅Π»ΠΈΠΌ Ρ‚Π΅ΠΏΠ΅Ρ€ΡŒ радиус ΠΊΡ€ΠΈΠ²ΠΈΠ·Π½Ρ‹ Π² Π²Π΅Ρ€ΡˆΠΈΠ½Π΅:

Β  Β 

По ΡƒΡΠ»ΠΎΠ²ΠΈΡŽ :

Β  Β 

Β  Β 

Β  Β 

Β  Β 

Β  Β 

Π±) ΠœΡ‹ ΡƒΠΆΠ΅ ΠΎΠΏΡ€Π΅Π΄Π΅Π»ΠΈΠ»ΠΈ , ΠΎΡΡ‚Π°Π»Π°ΡΡŒ максимальная высота подъСма.

Β  Β 

ВрСмя опрСдСляСм ΠΈΠ· условия равСнства Π½ΡƒΠ»ΡŽ Π²Π΅Ρ€Ρ‚ΠΈΠΊΠ°Π»ΡŒΠ½ΠΎΠΉ ΡΠΎΡΡ‚Π°Π²Π»ΡΡŽΡ‰Π΅ΠΉ скорости Ρ‚Π°ΠΊ ΠΆΠ΅, ΠΊΠ°ΠΊ ΠΌΡ‹ это Π΄Π΅Π»Π°Π»ΠΈ Π² ΠΏΡ€Π΅Π΄Ρ‹Π΄ΡƒΡ‰Π΅ΠΉ Π·Π°Π΄Π°Ρ‡Π΅:

Β  Β 

Β  Β 

Β  Β 

ΠŸΡ€ΠΈΡ€Π°Π²Π½ΠΈΠ²Π°Π΅ΠΌ ΠΈ :

Β  Β 

ΠžΡ‚ΠΊΡƒΠ΄Π° .

Β  Β 

Β  Β 

Β  Β 

ΠžΡ‚Π²Π΅Ρ‚: Π°) , Π±) .

Π”Π²ΠΈΠΆΠ΅Π½ΠΈΠ΅ ΠΏΠΎ окруТности | LAMPA

НайдСм ΡƒΠ³Π»ΠΎΠ²ΡƒΡŽ ΡΠΊΠΎΡ€ΠΎΡΡ‚ΡŒ. Π˜Π·Π²Π΅ΡΡ‚Π½ΠΎ, Ρ‡Ρ‚ΠΎ Ο‰=Ο†t\omega=\frac{\varphi}{t}Ο‰=tφ​. Π’ качСствС ΡƒΠ³Π»Π° Ο†\varphiΟ† ΠΌΠΎΠΆΠ½ΠΎ Π²Π·ΡΡ‚ΡŒ ΠΏΠΎΠ»Π½Ρ‹ΠΉ ΠΎΠ±ΠΎΡ€ΠΎΡ‚, Ρ‚ΠΎ Π΅ΡΡ‚ΡŒ ΡƒΠ³ΠΎΠ» 2Ο€2\pi2Ο€ Ρ€Π°Π΄ΠΈΠ°Π½, Π° Π² качСствС Π²Ρ€Π΅ΠΌΠ΅Π½ΠΈ β€” врСмя ΠΎΠ΄Π½ΠΎΠ³ΠΎ ΠΏΠΎΠ»Π½ΠΎΠ³ΠΎ ΠΎΠ±ΠΎΡ€ΠΎΡ‚Π°, Ρ‚ΠΎ Π΅ΡΡ‚ΡŒ ΠΏΠ΅Ρ€ΠΈΠΎΠ΄ TTT. ΠŸΠΎΡΡ‚ΠΎΠΌΡƒ

Ο‰=2Ο€T,\omega=\frac{2\pi}{T}{,}Ο‰=T2π​,Ο‰=2Ο€T=2Ο€β‹…1T=2πν.\omega=\frac{2\pi}{T}=2\pi\cdot\frac{1}{T}=2\pi\nu{.}Ο‰=T2π​=2Ο€β‹…T1​=2πν.

Π­Ρ‚ΠΈ Ρ„ΠΎΡ€ΠΌΡƒΠ»Ρ‹ ΠΌΡ‹ Ρ‚ΠΎΠΆΠ΅ Ρ€Π΅ΠΊΠΎΠΌΠ΅Π½Π΄ΡƒΠ΅ΠΌ Π·Π°ΠΏΠΎΠΌΠ½ΠΈΡ‚ΡŒ. Π­Ρ‚ΠΎ Π±ΡƒΠ΄Π΅Ρ‚ ΠΏΠΎΠ»Π΅Π·Π½ΠΎ.

Π•Π΄ΠΈΠ½ΠΈΡ†Π° измСрСния ΡƒΠ³Π»ΠΎΠ²ΠΎΠΉ скорости [Ο‰]=радс[\omega]=\frac{\text{Ρ€Π°Π΄}}{\text{с}}[Ο‰]=срад​.

ΠžΠΊΠ°Π·Ρ‹Π²Π°Π΅Ρ‚ΡΡ, Ρ‡Ρ‚ΠΎ линСйная ΡΠΊΠΎΡ€ΠΎΡΡ‚ΡŒ VVV ΠΈ угловая ΡΠΊΠΎΡ€ΠΎΡΡ‚ΡŒ Ο‰\omegaΟ‰ связаны Π΄Ρ€ΡƒΠ³ с Π΄Ρ€ΡƒΠ³ΠΎΠΌ. Рассмотрим ΠΏΡ€ΠΈΠΌΠ΅Ρ€ ΠΈΠ· ΠΆΠΈΠ·Π½ΠΈ. На дСтских ΠΏΠ»ΠΎΡ‰Π°Π΄ΠΊΠ°Ρ… навСрняка всС Π²ΠΈΠ΄Π΅Π»ΠΈ ΠΊΠ°Ρ€ΡƒΡΠ΅Π»ΡŒ. ΠŸΡ€Π΅Π΄ΡΡ‚Π°Π²ΡŒΡ‚Π΅, Ρ‡Ρ‚ΠΎ ΠΊΠ°Ρ€ΡƒΡΠ΅Π»ΡŒ вращаСтся. Π’Ρ‹ сами сидитС Π½Π° сидСньи этой карусСли, Π° ваш Π΄Ρ€ΡƒΠ³ Π½Π΅ стал ΡΠΈΠ΄Π΅Ρ‚ΡŒ Π½Π° сидСньи, Π° “ΠΏΡ€ΠΎΠ»Π΅Π·” ΠΏΠΎΠ±Π»ΠΈΠΆΠ΅ ΠΊ Ρ†Π΅Π½Ρ‚Ρ€Ρƒ карусСли.

ΠŸΠΎΡΠΊΠΎΠ»ΡŒΠΊΡƒ ΠΊΠ°ΠΆΠ΄Ρ‹ΠΉ ΠΈΠ· вас поворачиваСтся Π²ΠΎΠΊΡ€ΡƒΠ³ карусСли Π½Π° ΠΎΠ΄ΠΈΠ½ ΠΈ Ρ‚ΠΎΡ‚ ΠΆΠ΅ ΡƒΠ³ΠΎΠ» Π·Π° Ρ‚ΠΎ ΠΆΠ΅ врСмя, Ρ‚ΠΎ ΡƒΠ³Π»ΠΎΠ²Ρ‹Π΅ скорости Ρƒ вас Ρ€Π°Π²Π½Ρ‹: Ο‰Π²Ρ‹=Ο‰Π΄Ρ€ΡƒΠ³\omega_{Π²Ρ‹}=\omega_{Π΄Ρ€ΡƒΠ³}ωвы​=ωдруг​. Но Π²ΠΎΡ‚ Π»ΠΈΠ½Π΅ΠΉΠ½Ρ‹Π΅ скорости Ρƒ вас Π½Π΅ Ρ€Π°Π²Π½Ρ‹: VΠ²Ρ‹β‰ VΠ΄Ρ€ΡƒΠ³V_{Π²Ρ‹}\neq V_{Π΄Ρ€ΡƒΠ³}Vвы​≠Vдруг​. Π­Ρ‚ΠΎ Π½Π°ΠΌ подсказываСт наш ΠΆΠΈΠ·Π½Π΅Π½Π½Ρ‹ΠΉ ΠΎΠΏΡ‹Ρ‚. Π’ΠΎΡ‚, ΠΊΡ‚ΠΎ сидит ΠΏΠΎΠ±Π»ΠΈΠΆΠ΅, двигаСтся ΠΌΠ΅Π΄Π»Π΅Π½Π½Π΅Π΅.

Π§Π΅ΠΌ Π±Π»ΠΈΠΆΠ΅ ΠΊ Ρ†Π΅Π½Ρ‚Ρ€Ρƒ находится Ρ‚Π΅Π»ΠΎ β€” Ρ‚Π΅ΠΌ мСньшС Π΅Π³ΠΎ линСйная ΡΠΊΠΎΡ€ΠΎΡΡ‚ΡŒ VVV. И Π½Π°ΠΎΠ±ΠΎΡ€ΠΎΡ‚: Ρ‡Π΅ΠΌ дальшС ΠΎΡ‚ Ρ†Π΅Π½Ρ‚Ρ€Π° (Ρ‡Π΅ΠΌ большС расстояниС ΠΎΡ‚ Ρ†Π΅Π½Ρ‚Ρ€Π°), Ρ‚Π΅ΠΌ большС ΡΠΊΠΎΡ€ΠΎΡΡ‚ΡŒ VVV.

ЛинСйная ΡΠΊΠΎΡ€ΠΎΡΡ‚ΡŒ VVV Ρ‚Π°ΠΊΠΆΠ΅ Π±ΡƒΠ΄Π΅Ρ‚ большС ΠΈ Π² Ρ‚ΠΎΠΌ случаС, Ссли Π±ΡƒΠ΄Π΅Ρ‚ большС быстрота ΠΏΠΎΠ²ΠΎΡ€ΠΎΡ‚Π° Π²ΠΎΠΊΡ€ΡƒΠ³ оси, Ρ‚ΠΎ Π΅ΡΡ‚ΡŒ угловая ΡΠΊΠΎΡ€ΠΎΡΡ‚ΡŒ Ο‰\omegaΟ‰.

По-простому: Ρ‡Π΅ΠΌ дальшС сидишь ΠΎΡ‚ оси (Ρ‡Π΅ΠΌ большС RRR) ΠΈ Ρ‡Π΅ΠΌ быстрСС вращаСтся Ρ‚Π΅Π»ΠΎ (Ρ‡Π΅ΠΌ большС Ο‰\omegaΟ‰), Ρ‚Π΅ΠΌ большС линСйная ΡΠΊΠΎΡ€ΠΎΡΡ‚ΡŒ VVV.

Π›ΠΈΠ½Π΅ΠΉΠ½ΡƒΡŽ ΡΠΊΠΎΡ€ΠΎΡΡ‚ΡŒ VVV ΠΌΠΎΠΆΠ½ΠΎ ΠΏΠΎΠΉΡ‚ΠΈ ΠΏΠΎ Ρ„ΠΎΡ€ΠΌΡƒΠ»Π΅:

V=Ο‰β‹…R.V=\omega\cdot R{.}V=Ο‰β‹…R.

Π­Ρ‚Ρƒ Ρ„ΠΎΡ€ΠΌΡƒΠ»Ρƒ ΠΌΠΎΠΆΠ½ΠΎ вывСсти строго. Π’ΠΎΠ·ΡŒΠΌΠ΅ΠΌ ΡƒΠΆΠ΅ извСстныС Π½Π°ΠΌ Ρ„ΠΎΡ€ΠΌΡƒΠ»Ρ‹:

V=2πR⋅νV=2\pi R\cdot \nuV=2πR⋅ν и ω=2π⋅ν\omega=2\pi\cdot \nuω=2π⋅ν.

Из Π½ΠΈΡ… Π²ΠΈΠ΄Π½ΠΎ, Ρ‡Ρ‚ΠΎ Π² ΠΏΠ΅Ρ€Π²ΠΎΠΉ Ρ„ΠΎΡ€ΠΌΡƒΠ»Π΅ вмСсто 2πν2\pi\nu2πν ΠΌΠΎΠΆΠ½ΠΎ ΠΏΠΎΠ΄ΡΡ‚Π°Π²ΠΈΡ‚ΡŒ Ο‰\omegaΟ‰:

V=2Ο€Rβ‹…Ξ½=2πνR=(2πν)β‹…R=Ο‰β‹…RV=2\pi R\cdot \nu=2\pi\nu R=(2\pi\nu)\cdot R=\omega\cdot RV=2Ο€Rβ‹…Ξ½=2πνR=(2πν)β‹…R=Ο‰β‹…R.

ΠœΡ‹ ΠΏΠΎΠ»ΡƒΡ‡ΠΈΠ»ΠΈ Ρ„ΠΎΡ€ΠΌΡƒΠ»Ρƒ V=Ο‰β‹…RV=\omega\cdot RV=Ο‰β‹…R.

РасчСт RCF-RPM on-line

РасчСт RCF-RPM on-line

Он-Π»Π°ΠΉΠ½ ΠΊΠ°Π»ΡŒΠΊΡƒΠ»ΡΡ‚ΠΎΡ€ ΠΌΠΎΠΆΠ΅Ρ‚ Π±Ρ‹Ρ‚ΡŒ использован для:

  • расчСта ΠΏΠ°Ρ€Π°ΠΌΠ΅Ρ‚Ρ€Π° RPM (ΠΎΠ±ΠΎΡ€ΠΎΡ‚Ρ‹ Π² ΠΌΠΈΠ½ΡƒΡ‚Ρƒ) ΠΏΡ€ΠΈ Ρ€Π°Π±ΠΎΡ‚Π΅ ΠΏΠΎ ΠΏΡ€ΠΎΡ‚ΠΎΠΊΠΎΠ»Π°ΠΌ ΠΊ нашим Π½Π°Π±ΠΎΡ€Π°ΠΌ ΠΈΠ»ΠΈ ΠΌΠ΅Ρ‚ΠΎΠ΄ΠΈΠΊΠ°ΠΌ, ΠΏΡ€ΠΈΠ²Π΅Π΄Π΅Π½Π½Ρ‹ΠΌΒ Π²Β ΡΡ‚Π°Ρ‚ΡŒΡΡ…;
  • указания ΡƒΠ½ΠΈΠ²Π΅Ρ€ΡΠ°Π»ΡŒΠ½ΠΎΠΉ Π²Π΅Π»ΠΈΡ‡ΠΈΠ½Ρ‹ цСнтрифугирования RCF (g) Π² своих публикациях.

ΠžΡ‚Π»ΠΈΡ‡ΠΈΠ΅

RCF ΠΎΡ‚ RPM

Π’ ΡΡ‚Π°Ρ‚ΡŒΡΡ… рСкомСндуСтся ΡƒΠΊΠ°Π·Ρ‹Π²Π°Ρ‚ΡŒ ΡƒΠ½ΠΈΠ²Π΅Ρ€ΡΠ°Π»ΡŒΠ½ΡƒΡŽ Π²Π΅Π»ΠΈΡ‡ΠΈΠ½Ρƒ – ΠΎΡ‚Π½ΠΎΡΠΈΡ‚Π΅Π»ΡŒΠ½ΠΎΠ΅ ускорСниС Ρ†Π΅Π½Ρ‚Ρ€ΠΈΡ„ΡƒΠ³ΠΈ (RCF, Relative Centrifugal Force), которая измСряСтся Π² g. Π­Ρ‚ΠΎ Π΄Π°Π΅Ρ‚ Π²ΠΎΠ·ΠΌΠΎΠΆΠ½ΠΎΡΡ‚ΡŒ воспроизвСсти ΠΌΠ΅Ρ‚ΠΎΠ΄ΠΈΠΊΡƒ Π² любой Π»Π°Π±ΠΎΡ€Π°Ρ‚ΠΎΡ€ΠΈΠΈ. Если ΡƒΡΡ‚Π°Π½ΠΎΠ²ΠΈΡ‚ΡŒ ΠΎΠ΄Π½ΠΎ ΠΈ Ρ‚ΠΎ ΠΆΠ΅ Π·Π½Π°Ρ‡Π΅Π½ΠΈΠ΅ RCF Π½Π° Ρ€Π°Π·Π½Ρ‹Ρ… Ρ†Π΅Π½Ρ‚Ρ€ΠΈΡ„ΡƒΠ³Π°Ρ…, ΠΎΠ½ΠΈ Π±ΡƒΠ΄ΡƒΡ‚ ΠΎΡΠ°ΠΆΠ΄Π°Ρ‚ΡŒ ΠΎΠ±Ρ€Π°Π·Π΅Ρ† с ΠΎΠ΄ΠΈΠ½Π°ΠΊΠΎΠ²ΠΎΠΉ ΡΡ„Ρ„Π΅ΠΊΡ‚ΠΈΠ²Π½ΠΎΡΡ‚ΡŒΡŽ.

НСкоторыС ΠΌΠΎΠ΄Π΅Π»ΠΈ Ρ†Π΅Π½Ρ‚Ρ€ΠΈΡ„ΡƒΠ³ Π½Π΅ ΠΏΠΎΠ·Π²ΠΎΠ»ΡΡŽΡ‚ Π·Π°Π΄Π°Ρ‚ΡŒ ускорСниС RCF, Π½Π° Π½ΠΈΡ… Π²ΠΎΠ·ΠΌΠΎΠΆΠ½ΠΎ ΡƒΡΡ‚Π°Π½ΠΎΠ²ΠΈΡ‚ΡŒ Ρ‚ΠΎΠ»ΡŒΠΊΠΎ частоту вращСния (RPM, Rotation Per Minute), которая измСряСтся Π² ΠΎΠ±ΠΎΡ€ΠΎΡ‚Π°Ρ… Π² ΠΌΠΈΠ½ΡƒΡ‚Ρƒ. RPM Ρ…Π°Ρ€Π°ΠΊΡ‚Π΅Ρ€ΠΈΠ·ΡƒΠ΅Ρ‚ условия цСнтрифугирования Ρ‚ΠΎΠ»ΡŒΠΊΠΎ Π½Π° Π²Ρ‹Π±Ρ€Π°Π½Π½ΠΎΠΉ ΠΌΠΎΠ΄Π΅Π»ΠΈ Ρ†Π΅Π½Ρ‚Ρ€ΠΈΡ„ΡƒΠ³ΠΈ: Ссли ΡƒΡΡ‚Π°Π½ΠΎΠ²ΠΈΡ‚ΡŒ ΠΎΠ΄Π½ΠΎ ΠΈ Ρ‚ΠΎ ΠΆΠ΅ Π·Π½Π°Ρ‡Π΅Π½ΠΈΠ΅ RPM Π½Π° Ρ†Π΅Π½Ρ‚Ρ€ΠΈΡ„ΡƒΠ³Π°Ρ… с Ρ€Π°Π·Π½Ρ‹ΠΌΠΈ Ρ€ΠΎΡ‚ΠΎΡ€Π°ΠΌΠΈ, ΠΎΠ½ΠΈ Π±ΡƒΠ΄ΡƒΡ‚ ΠΎΡΠ°ΠΆΠ΄Π°Ρ‚ΡŒ ΠΎΠ±Ρ€Π°Π·Π΅Ρ† с Ρ€Π°Π·Π½ΠΎΠΉ ΡΡ„Ρ„Π΅ΠΊΡ‚ΠΈΠ²Π½ΠΎΡΡ‚ΡŒΡŽ.

RCF, RPM ΠΈ радиус Ρ€ΠΎΡ‚ΠΎΡ€Π° Ρ†Π΅Π½Ρ‚Ρ€ΠΈΡ„ΡƒΠ³ΠΈ связаны Ρ„ΠΎΡ€ΠΌΡƒΠ»ΠΎΠΉ:

, Π³Π΄Π΅:

RPM – частота вращСния Π² ΠΎΠ±ΠΎΡ€ΠΎΡ‚Π°Ρ… Π² ΠΌΠΈΠ½ΡƒΡ‚Ρƒ,
RCF – ΠΎΡ‚Π½ΠΎΡΠΈΡ‚Π΅Π»ΡŒΠ½ΠΎΠ΅ ускорСниС Ρ†Π΅Π½Ρ‚Ρ€ΠΈΡ„ΡƒΠ³ΠΈ Π² g,
r – радиус Ρ€ΠΎΡ‚ΠΎΡ€Π° Π² сантимСтрах.

Из этой Ρ„ΠΎΡ€ΠΌΡƒΠ»Ρ‹ слСдуСт Π΄Π²Π° Π²Ρ‹Π²ΠΎΠ΄Π°:

  • Π§Π΅ΠΌ большС радиус Ρ€ΠΎΡ‚ΠΎΡ€Π°, Ρ‚Π΅ΠΌ мСньшС Π½ΡƒΠΆΠ½ΠΎ ΠΎΠ±ΠΎΡ€ΠΎΡ‚ΠΎΠ² Π² ΠΌΠΈΠ½ΡƒΡ‚Ρƒ, Ρ‡Ρ‚ΠΎΠ±Ρ‹ ΠΏΠΎΠ΄Π΄Π΅Ρ€ΠΆΠΈΠ²Π°Ρ‚ΡŒ Ρ‚ΠΎ ΠΆΠ΅ ΠΎΡ‚Π½ΠΎΡΠΈΡ‚Π΅Π»ΡŒΠ½ΠΎΠ΅ ускорСниС. Π˜Π½Ρ„ΠΎΡ€ΠΌΠ°Ρ†ΠΈΡŽ ΠΎ Ρ€ΠΎΡ‚ΠΎΡ€Π΅ ΡƒΠΊΠ°Π·Ρ‹Π²Π°ΡŽΡ‚ Π² руководствС ΠΏΠΎ эксплуатации Ρ†Π΅Π½Ρ‚Ρ€ΠΈΡ„ΡƒΠ³ΠΈ.
  • Π›ΡŽΠ±ΠΎΠ΅ ΠΈΠ·ΠΌΠ΅Π½Π΅Π½ΠΈΠ΅ частоты вращСния сильно влияСт Π½Π° ΡΡ„Ρ„Π΅ΠΊΡ‚ΠΈΠ²Π½ΠΎΡΡ‚ΡŒ цСнтрифугирования, ΠΏΠΎΡΠΊΠΎΠ»ΡŒΠΊΡƒ RCF прямо ΠΏΡ€ΠΎΠΏΠΎΡ€Ρ†ΠΈΠΎΠ½Π°Π»ΡŒΠ½ΠΎ ΠΊΠ²Π°Π΄Ρ€Π°Ρ‚Ρƒ RPM.

ΠŸΠ΅Ρ€Π΅Π²Π΅ΡΡ‚ΠΈ

g Π² ΠΎΠ±/ΠΌΠΈΠ½ ΠΈΠ»ΠΈ Π½Π°ΠΎΠ±ΠΎΡ€ΠΎΡ‚

Радиус Ρ€ΠΎΡ‚ΠΎΡ€Π° (см):

Π’Π²Π΅Π΄ΠΈΡ‚Π΅ радиус Ρ€ΠΎΡ‚ΠΎΡ€Π° Ρ†Π΅Π½Ρ‚Ρ€ΠΈΡ„ΡƒΠ³ΠΈ Π² сантимСтрах.

RCF (g):

ΠŸΡ€ΠΈ Π·Π°ΠΏΠΎΠ»Π½Π΅Π½ΠΈΠΈ поля RCF (g) ΠΈΠ»ΠΈ RPM (ΠΎΠ±/ΠΌΠΈΠ½),
Π²Ρ‚ΠΎΡ€ΠΎΠ΅ Π·Π½Π°Ρ‡Π΅Π½ΠΈΠ΅ рассчитаСтся автоматичСски.

Π£Π³Π»ΠΎΠ²ΠΎΠ΅ ускорСниС | Π€ΠΈΠ·ΠΈΠΊΠ°

Π¦Π΅Π»ΠΈ обучСния

К ΠΊΠΎΠ½Ρ†Ρƒ этого Ρ€Π°Π·Π΄Π΅Π»Π° Π²Ρ‹ смоТСтС:

  • ΠžΠΏΠΈΡˆΠΈΡ‚Π΅ Ρ€Π°Π²Π½ΠΎΠΌΠ΅Ρ€Π½ΠΎΠ΅ ΠΊΡ€ΡƒΠ³ΠΎΠ²ΠΎΠ΅ Π΄Π²ΠΈΠΆΠ΅Π½ΠΈΠ΅.
  • ΠžΠ±ΡŠΡΡΠ½ΠΈΡ‚Π΅ Π½Π΅Ρ€Π°Π²Π½ΠΎΠΌΠ΅Ρ€Π½ΠΎΠ΅ ΠΊΡ€ΡƒΠ³ΠΎΠ²ΠΎΠ΅ Π΄Π²ΠΈΠΆΠ΅Π½ΠΈΠ΅.
  • Π’Ρ‹Ρ‡ΠΈΡΠ»ΠΈΡ‚ΡŒ ΡƒΠ³Π»ΠΎΠ²ΠΎΠ΅ ускорСниС ΠΎΠ±ΡŠΠ΅ΠΊΡ‚Π°.
  • ΠžΠ±Ρ€Π°Ρ‚ΠΈΡ‚Π΅ Π²Π½ΠΈΠΌΠ°Π½ΠΈΠ΅ Π½Π° связь ΠΌΠ΅ΠΆΠ΄Ρƒ Π»ΠΈΠ½Π΅ΠΉΠ½Ρ‹ΠΌ ΠΈ ΡƒΠ³Π»ΠΎΠ²Ρ‹ΠΌ ускорСниСм.
Π Π°Π²Π½ΠΎΠΌΠ΅Ρ€Π½ΠΎΠ΅ ΠΊΡ€ΡƒΠ³ΠΎΠ²ΠΎΠ΅ Π΄Π²ΠΈΠΆΠ΅Π½ΠΈΠ΅ ΠΈ гравитация ΠΎΠ±ΡΡƒΠΆΠ΄Π°ΡŽΡ‚ΡΡ Ρ‚ΠΎΠ»ΡŒΠΊΠΎ Ρ€Π°Π²Π½ΠΎΠΌΠ΅Ρ€Π½ΠΎΠ΅ ΠΊΡ€ΡƒΠ³ΠΎΠ²ΠΎΠ΅ Π΄Π²ΠΈΠΆΠ΅Π½ΠΈΠ΅, Ρ‚ΠΎ Π΅ΡΡ‚ΡŒ Π΄Π²ΠΈΠΆΠ΅Π½ΠΈΠ΅ ΠΏΠΎ ΠΊΡ€ΡƒΠ³Ρƒ с постоянной ΡΠΊΠΎΡ€ΠΎΡΡ‚ΡŒΡŽ ΠΈ, ΡΠ»Π΅Π΄ΠΎΠ²Π°Ρ‚Π΅Π»ΡŒΠ½ΠΎ, с постоянной ΡƒΠ³Π»ΠΎΠ²ΠΎΠΉ ΡΠΊΠΎΡ€ΠΎΡΡ‚ΡŒΡŽ.Напомним, угловая ΡΠΊΠΎΡ€ΠΎΡΡ‚ΡŒ Ο‰ ΠΎΠΏΡ€Π΅Π΄Π΅Π»ΡΠ»Π°ΡΡŒ ΠΊΠ°ΠΊ ΡΠΊΠΎΡ€ΠΎΡΡ‚ΡŒ измСнСния ΡƒΠ³Π»Π° ΞΈ Π²ΠΎ Π²Ρ€Π΅ΠΌΠ΅Π½ΠΈ:

[латСкс] \ omega = \ frac {\ Delta \ theta} {\ Delta t} \\ [/ latex]

, Π³Π΄Π΅ ΞΈ – это ΡƒΠ³ΠΎΠ» ΠΏΠΎΠ²ΠΎΡ€ΠΎΡ‚Π°, ΠΊΠ°ΠΊ ΠΏΠΎΠΊΠ°Π·Π°Π½ΠΎ Π½Π° РисункС 1. ΠžΡ‚Π½ΠΎΡˆΠ΅Π½ΠΈΠ΅ ΠΌΠ΅ΠΆΠ΄Ρƒ ΡƒΠ³Π»ΠΎΠ²ΠΎΠΉ ΡΠΊΠΎΡ€ΠΎΡΡ‚ΡŒΡŽ Ο‰ ΠΈ Π»ΠΈΠ½Π΅ΠΉΠ½ΠΎΠΉ ΡΠΊΠΎΡ€ΠΎΡΡ‚ΡŒΡŽ v Ρ‚Π°ΠΊΠΆΠ΅ Π±Ρ‹Π»ΠΎ ΠΎΠΏΡ€Π΅Π΄Π΅Π»Π΅Π½ΠΎ Π² Π£Π³ΠΎΠ» ΠΏΠΎΠ²ΠΎΡ€ΠΎΡ‚Π° ΠΈ Угловая ΡΠΊΠΎΡ€ΠΎΡΡ‚ΡŒ ΠΊΠ°ΠΊ

v = rω

ΠΈΠ»ΠΈ

[латСкс] \ omega = \ frac {v} {r} \\ [/ латСкс]

, Π³Π΄Π΅ r – радиус ΠΊΡ€ΠΈΠ²ΠΈΠ·Π½Ρ‹, Ρ‚Π°ΠΊΠΆΠ΅ Π²ΠΈΠ΄Π½ΠΎ Π½Π° рисункС 1.Богласно соглашСнию ΠΎ Π·Π½Π°ΠΊΠ°Ρ…, Π½Π°ΠΏΡ€Π°Π²Π»Π΅Π½ΠΈΠ΅ ΠΏΡ€ΠΎΡ‚ΠΈΠ² часовой стрСлки считаСтся ΠΏΠΎΠ»ΠΎΠΆΠΈΡ‚Π΅Π»ΡŒΠ½Ρ‹ΠΌ Π½Π°ΠΏΡ€Π°Π²Π»Π΅Π½ΠΈΠ΅ΠΌ, Π° Π½Π°ΠΏΡ€Π°Π²Π»Π΅Π½ΠΈΠ΅ ΠΏΠΎ часовой стрСлкС – ΠΎΡ‚Ρ€ΠΈΡ†Π°Ρ‚Π΅Π»ΡŒΠ½Ρ‹ΠΌ

.

Рис. 1. На этом рисункС ΠΏΠΎΠΊΠ°Π·Π°Π½ΠΎ Ρ€Π°Π²Π½ΠΎΠΌΠ΅Ρ€Π½ΠΎΠ΅ ΠΊΡ€ΡƒΠ³ΠΎΠ²ΠΎΠ΅ Π΄Π²ΠΈΠΆΠ΅Π½ΠΈΠ΅ ΠΈ Π½Π΅ΠΊΠΎΡ‚ΠΎΡ€Ρ‹Π΅ ΠΈΠ· Π΅Π³ΠΎ ΠΎΠΏΡ€Π΅Π΄Π΅Π»Π΅Π½Π½Ρ‹Ρ… Π²Π΅Π»ΠΈΡ‡ΠΈΠ½.

Угловая ΡΠΊΠΎΡ€ΠΎΡΡ‚ΡŒ нСпостоянна, ΠΊΠΎΠ³Π΄Π° фигуристка тянСт Π½Π° Ρ€ΡƒΠΊΠ°Ρ…, ΠΊΠΎΠ³Π΄Π° Ρ€Π΅Π±Π΅Π½ΠΎΠΊ запускаСт ΠΊΠ°Ρ€ΡƒΡΠ΅Π»ΡŒ ΠΈΠ· состояния покоя ΠΈΠ»ΠΈ ΠΊΠΎΠ³Π΄Π° ТСсткий диск ΠΊΠΎΠΌΠΏΡŒΡŽΡ‚Π΅Ρ€Π° останавливаСтся, ΠΊΠΎΠ³Π΄Π° ΠΎΠ½ Π²Ρ‹ΠΊΠ»ΡŽΡ‡Π΅Π½. Π’ΠΎ всСх этих случаях имССтся ΡƒΠ³Π»ΠΎΠ²ΠΎΠ΅ ускорСниС , ΠΏΡ€ΠΈ ΠΊΠΎΡ‚ΠΎΡ€ΠΎΠΌ измСняСтся Ο‰ .Π§Π΅ΠΌ быстрСС происходит ΠΈΠ·ΠΌΠ΅Π½Π΅Π½ΠΈΠ΅, Ρ‚Π΅ΠΌ большС ΡƒΠ³Π»ΠΎΠ²ΠΎΠ΅ ускорСниС. Π£Π³Π»ΠΎΠ²ΠΎΠ΅ ускорСниС Ξ± опрСдСляСтся ΠΊΠ°ΠΊ ΡΠΊΠΎΡ€ΠΎΡΡ‚ΡŒ измСнСния ΡƒΠ³Π»ΠΎΠ²ΠΎΠΉ скорости. Π’ Ρ„ΠΎΡ€ΠΌΠ΅ уравнСния ΡƒΠ³Π»ΠΎΠ²ΠΎΠ΅ ускорСниС выраТаСтся ΡΠ»Π΅Π΄ΡƒΡŽΡ‰ΠΈΠΌ ΠΎΠ±Ρ€Π°Π·ΠΎΠΌ:

[латСкс] \ alpha = \ frac {\ Delta \ omega} {\ Delta t} \\ [/ latex],

, Π³Π΄Π΅ Ξ” Ο‰ – это ΠΈΠ·ΠΌΠ΅Π½Π΅Π½ΠΈΠ΅ ΡƒΠ³Π»ΠΎΠ²ΠΎΠΉ скорости , Π° Ξ” t – ΠΈΠ·ΠΌΠ΅Π½Π΅Π½ΠΈΠ΅ Π²ΠΎ Π²Ρ€Π΅ΠΌΠ΅Π½ΠΈ. Π•Π΄ΠΈΠ½ΠΈΡ†Ρ‹ ΡƒΠ³Π»ΠΎΠ²ΠΎΠ³ΠΎ ускорСния: (Ρ€Π°Π΄ / с) / с, ΠΈΠ»ΠΈ Ρ€Π°Π΄ / с 2 . Если Ο‰ увСличиваСтся, Ρ‚ΠΎ Ξ± ΠΏΠΎΠ»ΠΎΠΆΠΈΡ‚Π΅Π»ΡŒΠ½ΠΎ.Если Ο‰ ΡƒΠΌΠ΅Π½ΡŒΡˆΠ°Π΅Ρ‚ΡΡ, Ρ‚ΠΎ Ξ± ΠΎΡ‚Ρ€ΠΈΡ†Π°Ρ‚Π΅Π»ΡŒΠ½ΠΎ.

ΠŸΡ€ΠΈΠΌΠ΅Ρ€ 1. РасчСт ΡƒΠ³Π»ΠΎΠ²ΠΎΠ³ΠΎ ускорСния ΠΈ замСдлСния вСлосипСдного колСса

ΠŸΡ€Π΅Π΄ΠΏΠΎΠ»ΠΎΠΆΠΈΠΌ, Ρ‡Ρ‚ΠΎ подросток ΠΊΠ»Π°Π΄Π΅Ρ‚ вСлосипСд Π½Π° спину ΠΈ запускаСт Π²Ρ€Π°Ρ‰Π΅Π½ΠΈΠ΅ Π·Π°Π΄Π½Π΅Π³ΠΎ колСса ΠΎΡ‚ состояния покоя Π΄ΠΎ ΠΊΠΎΠ½Π΅Ρ‡Π½ΠΎΠΉ ΡƒΠ³Π»ΠΎΠ²ΠΎΠΉ скорости 250 ΠΎΠ± / ΠΌΠΈΠ½ Π·Π° 5,00 с. (Π°) РассчитайтС ΡƒΠ³Π»ΠΎΠ²ΠΎΠ΅ ускорСниС Π² Ρ€Π°Π΄ / с 2 . (b) Если Ρ‚Π΅ΠΏΠ΅Ρ€ΡŒ ΠΎΠ½Π° Π½Π°ΠΆΠΈΠΌΠ°Π΅Ρ‚ Π½Π° Ρ‚ΠΎΡ€ΠΌΠΎΠ·Π°, вызывая ΡƒΠ³Π»ΠΎΠ²ΠΎΠ΅ ускорСниС -87,3 Ρ€Π°Π΄ / с 2 , сколько Π²Ρ€Π΅ΠΌΠ΅Π½ΠΈ потрСбуСтся колСсу, Ρ‡Ρ‚ΠΎΠ±Ρ‹ ΠΎΡΡ‚Π°Π½ΠΎΠ²ΠΈΡ‚ΡŒΡΡ?

БтратСгия для (Π°)

Π£Π³Π»ΠΎΠ²ΠΎΠ΅ ускорСниС ΠΌΠΎΠΆΠ½ΠΎ Π½Π°ΠΉΡ‚ΠΈ нСпосрСдствСнно ΠΈΠ· Π΅Π³ΠΎ опрСдСлСния Π² [latex] \ alpha = \ frac {\ Delta \ omega} {\ Delta t} \\ [/ latex], ΠΏΠΎΡΠΊΠΎΠ»ΡŒΠΊΡƒ Π΄Π°Π½Ρ‹ ΠΎΠΊΠΎΠ½Ρ‡Π°Ρ‚Π΅Π»ΡŒΠ½Π°Ρ угловая ΡΠΊΠΎΡ€ΠΎΡΡ‚ΡŒ ΠΈ врСмя.ΠœΡ‹ Π²ΠΈΠ΄ΠΈΠΌ, Ρ‡Ρ‚ΠΎ Ξ” Ο‰ составляСт 250 ΠΎΠ± / ΠΌΠΈΠ½, Π° Ξ” t составляСт 5,00 с.

РСшСниС для (а)

Вводя ΠΈΠ·Π²Π΅ΡΡ‚Π½ΡƒΡŽ ΠΈΠ½Ρ„ΠΎΡ€ΠΌΠ°Ρ†ΠΈΡŽ Π² ΠΎΠΏΡ€Π΅Π΄Π΅Π»Π΅Π½ΠΈΠ΅ ΡƒΠ³Π»ΠΎΠ²ΠΎΠ³ΠΎ ускорСния, ΠΏΠΎΠ»ΡƒΡ‡Π°Π΅ΠΌ

[латСкс] \ begin {array} {lll} \ alpha & = & \ frac {\ Delta \ omega} {\ Delta t} \\ & = & \ frac {\ text {250 ΠΎΠ± / ΠΌΠΈΠ½}} {\ text {5.00 s}} \ text {.} \ end {array} \\ [/ latex]

ΠŸΠΎΡΠΊΠΎΠ»ΡŒΠΊΡƒ Ξ” Ο‰ выраТаСтся Π² ΠΎΠ±ΠΎΡ€ΠΎΡ‚Π°Ρ… Π² ΠΌΠΈΠ½ΡƒΡ‚Ρƒ (ΠΎΠ± / ΠΌΠΈΠ½), ΠΈ Π½Π°ΠΌ Π½ΡƒΠΆΠ½Ρ‹ стандартныС Π΅Π΄ΠΈΠ½ΠΈΡ†Ρ‹ Ρ€Π°Π΄ / с 2 для ΡƒΠ³Π»ΠΎΠ²ΠΎΠ³ΠΎ ускорСния, Π½Π°ΠΌ Π½ΡƒΠΆΠ½ΠΎ ΠΏΡ€Π΅ΠΎΠ±Ρ€Π°Π·ΠΎΠ²Π°Ρ‚ΡŒ Ξ” Ο‰ ΠΈΠ· ΠΎΠ± / ΠΌΠΈΠ½ Π² Ρ€Π°Π΄ / с:

[латСкс] \ begin {array} {c} \ Delta {\ omega} & = & 250 \ frac {\ text {rev}} {\ text {min}} \ cdot \ frac {2 \ pi \ text {rad }} {\ text {rev}} \ cdot \ frac {1 \ text {min}} {60 \ text {sec}} \\ & = & 26.{2} \ text {.} \ End {array} \\ [/ latex]

БтратСгия для (b)

Π’ этой части ΠΌΡ‹ Π·Π½Π°Π΅ΠΌ ΡƒΠ³Π»ΠΎΠ²ΠΎΠ΅ ускорСниС ΠΈ Π½Π°Ρ‡Π°Π»ΡŒΠ½ΡƒΡŽ ΡƒΠ³Π»ΠΎΠ²ΡƒΡŽ ΡΠΊΠΎΡ€ΠΎΡΡ‚ΡŒ. ΠœΡ‹ ΠΌΠΎΠΆΠ΅ΠΌ Π½Π°ΠΉΡ‚ΠΈ врСмя остановки, ΠΈΡΠΏΠΎΠ»ΡŒΠ·ΡƒΡ ΠΎΠΏΡ€Π΅Π΄Π΅Π»Π΅Π½ΠΈΠ΅ ΡƒΠ³Π»ΠΎΠ²ΠΎΠ³ΠΎ ускорСния ΠΈ Ρ€Π΅ΡˆΠΈΠ² для Ξ” t , Ρ‡Ρ‚ΠΎ Π΄Π°Π΅Ρ‚

[латСкс] \ Delta t = \ frac {\ Delta \ omega} {\ alpha} \\ [/ latex].

РСшСниС для (b)

Π—Π΄Π΅ΡΡŒ угловая ΡΠΊΠΎΡ€ΠΎΡΡ‚ΡŒ ΡƒΠΌΠ΅Π½ΡŒΡˆΠ°Π΅Ρ‚ΡΡ с 26,2 Ρ€Π°Π΄ / с (250 ΠΎΠ± / ΠΌΠΈΠ½) Π΄ΠΎ нуля, Ρ‚Π°ΠΊ Ρ‡Ρ‚ΠΎ Ξ” Ο‰ составляСт –26.{2}} \\ & = & \ text {0,300 с.} \ End {array} \\ [/ latex]

ΠžΠ±ΡΡƒΠΆΠ΄Π΅Π½ΠΈΠ΅

ΠžΠ±Ρ€Π°Ρ‚ΠΈΡ‚Π΅ Π²Π½ΠΈΠΌΠ°Π½ΠΈΠ΅, Ρ‡Ρ‚ΠΎ ΡƒΠ³Π»ΠΎΠ²ΠΎΠ΅ ускорСниС, ΠΊΠΎΠ³Π΄Π° Π΄Π΅Π²ΡƒΡˆΠΊΠ° Π²Ρ€Π°Ρ‰Π°Π΅Ρ‚ колСсо, нСбольшоС ΠΈ ΠΏΠΎΠ»ΠΎΠΆΠΈΡ‚Π΅Π»ΡŒΠ½ΠΎΠ΅; для получСния Π·Π°ΠΌΠ΅Ρ‚Π½ΠΎΠΉ ΡƒΠ³Π»ΠΎΠ²ΠΎΠΉ скорости трСбуСтся 5 с. Когда ΠΎΠ½Π° Π½Π°ΠΆΠΈΠΌΠ°Π΅Ρ‚ Π½Π° Ρ‚ΠΎΡ€ΠΌΠΎΠ·, ΡƒΠ³Π»ΠΎΠ²ΠΎΠ΅ ускорСниС Π²Π΅Π»ΠΈΠΊΠΎ ΠΈ ΠΎΡ‚Ρ€ΠΈΡ†Π°Ρ‚Π΅Π»ΡŒΠ½ΠΎ. Угловая ΡΠΊΠΎΡ€ΠΎΡΡ‚ΡŒ быстро стрСмится ΠΊ Π½ΡƒΠ»ΡŽ. Π’ ΠΎΠ±ΠΎΠΈΡ… случаях ΠΎΡ‚Π½ΠΎΡˆΠ΅Π½ΠΈΡ Π°Π½Π°Π»ΠΎΠ³ΠΈΡ‡Π½Ρ‹ Ρ‚ΠΎΠΌΡƒ, Ρ‡Ρ‚ΠΎ происходит с Π»ΠΈΠ½Π΅ΠΉΠ½Ρ‹ΠΌ Π΄Π²ΠΈΠΆΠ΅Π½ΠΈΠ΅ΠΌ. НапримСр, ΠΊΠΎΠ³Π΄Π° Π²Ρ‹ Π²Ρ€Π΅Π·Π°Π΅Ρ‚Π΅ΡΡŒ Π² ΠΊΠΈΡ€ΠΏΠΈΡ‡Π½ΡƒΡŽ стСну, происходит сильноС Π·Π°ΠΌΠ΅Π΄Π»Π΅Π½ΠΈΠ΅ – ΠΈΠ·ΠΌΠ΅Π½Π΅Π½ΠΈΠ΅ скорости Π²Π΅Π»ΠΈΠΊΠΎ Π·Π° ΠΊΠΎΡ€ΠΎΡ‚ΠΊΠΈΠΉ ΠΏΡ€ΠΎΠΌΠ΅ΠΆΡƒΡ‚ΠΎΠΊ Π²Ρ€Π΅ΠΌΠ΅Π½ΠΈ.

Если Π±Ρ‹ вСлосипСд Π² ΠΏΡ€Π΅Π΄Ρ‹Π΄ΡƒΡ‰Π΅ΠΌ ΠΏΡ€ΠΈΠΌΠ΅Ρ€Π΅ Π±Ρ‹Π» Π½Π° колСсах, Π° Π½Π΅ ΠΏΠ΅Ρ€Π΅Π²Π΅Ρ€Π½ΡƒΡ‚, ΠΎΠ½ сначала разогнался Π±Ρ‹ ΠΏΠΎ Π·Π΅ΠΌΠ»Π΅, Π° Π·Π°Ρ‚Π΅ΠΌ остановился Π±Ρ‹. Π­Ρ‚Ρƒ связь ΠΌΠ΅ΠΆΠ΄Ρƒ ΠΊΡ€ΡƒΠ³ΠΎΠ²Ρ‹ΠΌ Π΄Π²ΠΈΠΆΠ΅Π½ΠΈΠ΅ΠΌ ΠΈ Π»ΠΈΠ½Π΅ΠΉΠ½Ρ‹ΠΌ Π΄Π²ΠΈΠΆΠ΅Π½ΠΈΠ΅ΠΌ Π½Π΅ΠΎΠ±Ρ…ΠΎΠ΄ΠΈΠΌΠΎ ΠΈΡΡΠ»Π΅Π΄ΠΎΠ²Π°Ρ‚ΡŒ. НапримСр, Π±Ρ‹Π»ΠΎ Π±Ρ‹ ΠΏΠΎΠ»Π΅Π·Π½ΠΎ Π·Π½Π°Ρ‚ΡŒ, ΠΊΠ°ΠΊ связаны Π»ΠΈΠ½Π΅ΠΉΠ½ΠΎΠ΅ ΠΈ ΡƒΠ³Π»ΠΎΠ²ΠΎΠ΅ ускорСниС. ΠŸΡ€ΠΈ ΠΊΡ€ΡƒΠ³ΠΎΠ²ΠΎΠΌ Π΄Π²ΠΈΠΆΠ΅Π½ΠΈΠΈ Π»ΠΈΠ½Π΅ΠΉΠ½ΠΎΠ΅ ускорСниС составляСт ΠΏΠΎ ΠΊΠ°ΡΠ°Ρ‚Π΅Π»ΡŒΠ½ΠΎΠΉ ΠΊ окруТности Π² ΠΈΠ½Ρ‚Π΅Ρ€Π΅ΡΡƒΡŽΡ‰Π΅ΠΉ Ρ‚ΠΎΡ‡ΠΊΠ΅, ΠΊΠ°ΠΊ ΠΏΠΎΠΊΠ°Π·Π°Π½ΠΎ Π½Π° рисункС 2. Π’Π°ΠΊΠΈΠΌ ΠΎΠ±Ρ€Π°Π·ΠΎΠΌ, Π»ΠΈΠ½Π΅ΠΉΠ½ΠΎΠ΅ ускорСниС называСтся ΠΊΠ°ΡΠ°Ρ‚Π΅Π»ΡŒΠ½Ρ‹ΠΌ ускорСниСм a t .

Рис. 2. ΠŸΡ€ΠΈ ΠΊΡ€ΡƒΠ³ΠΎΠ²ΠΎΠΌ Π΄Π²ΠΈΠΆΠ΅Π½ΠΈΠΈ Π»ΠΈΠ½Π΅ΠΉΠ½ΠΎΠ΅ ускорСниС a Π²ΠΎΠ·Π½ΠΈΠΊΠ°Π΅Ρ‚ ΠΏΠΎ ΠΌΠ΅Ρ€Π΅ измСнСния Π²Π΅Π»ΠΈΡ‡ΠΈΠ½Ρ‹ скорости: a касаСтся двиТСния. Π’ контСкстС ΠΊΡ€ΡƒΠ³ΠΎΠ²ΠΎΠ³ΠΎ двиТСния Π»ΠΈΠ½Π΅ΠΉΠ½ΠΎΠ΅ ускорСниС Ρ‚Π°ΠΊΠΆΠ΅ называСтся Ρ‚Π°Π½Π³Π΅Π½Ρ†ΠΈΠ°Π»ΡŒΠ½Ρ‹ΠΌ ускорСниСм a t .

Π›ΠΈΠ½Π΅ΠΉΠ½ΠΎΠ΅ ΠΈΠ»ΠΈ Ρ‚Π°Π½Π³Π΅Π½Ρ†ΠΈΠ°Π»ΡŒΠ½ΠΎΠ΅ ускорСниС относится ΠΊ измСнСниям Π²Π΅Π»ΠΈΡ‡ΠΈΠ½Ρ‹ скорости, Π½ΠΎ Π½Π΅ Π΅Π΅ направлСния. Из Β«Π Π°Π²Π½ΠΎΠΌΠ΅Ρ€Π½ΠΎΠ³ΠΎ ΠΊΡ€ΡƒΠ³ΠΎΠ²ΠΎΠ³ΠΎ двиТСния ΠΈ Π³Ρ€Π°Π²ΠΈΡ‚Π°Ρ†ΠΈΠΈΒ» ΠΌΡ‹ Π·Π½Π°Π΅ΠΌ, Ρ‡Ρ‚ΠΎ Ρ†Π΅Π½Ρ‚Ρ€ΠΎΡΡ‚Ρ€Π΅ΠΌΠΈΡ‚Π΅Π»ΡŒΠ½ΠΎΠ΅ ускорСниС ΠΏΡ€ΠΈ ΠΊΡ€ΡƒΠ³ΠΎΠ²ΠΎΠΌ Π΄Π²ΠΈΠΆΠ΅Π½ΠΈΠΈ, a c , относится ΠΊ измСнСниям направлСния скорости, Π½ΠΎ Π½Π΅ Π΅Π΅ Π²Π΅Π»ΠΈΡ‡ΠΈΠ½Ρ‹.ΠžΠ±ΡŠΠ΅ΠΊΡ‚, ΡΠΎΠ²Π΅Ρ€ΡˆΠ°ΡŽΡ‰ΠΈΠΉ ΠΊΡ€ΡƒΠ³ΠΎΠ²ΠΎΠ΅ Π΄Π²ΠΈΠΆΠ΅Π½ΠΈΠ΅, испытываСт Ρ†Π΅Π½Ρ‚Ρ€ΠΎΡΡ‚Ρ€Π΅ΠΌΠΈΡ‚Π΅Π»ΡŒΠ½ΠΎΠ΅ ускорСниС, ΠΊΠ°ΠΊ ΠΏΠΎΠΊΠ°Π·Π°Π½ΠΎ Π½Π° рисункС 3. Π’Π°ΠΊΠΈΠΌ ΠΎΠ±Ρ€Π°Π·ΠΎΠΌ, a t ΠΈ a c пСрпСндикулярны ΠΈ нСзависимы Π΄Ρ€ΡƒΠ³ ΠΎΡ‚ Π΄Ρ€ΡƒΠ³Π°. ΠšΠ°ΡΠ°Ρ‚Π΅Π»ΡŒΠ½ΠΎΠ΅ ускорСниС a t Π½Π°ΠΏΡ€ΡΠΌΡƒΡŽ связано с ΡƒΠ³Π»ΠΎΠ²Ρ‹ΠΌ ускорСниСм Ξ± ΠΈ связано с ΡƒΠ²Π΅Π»ΠΈΡ‡Π΅Π½ΠΈΠ΅ΠΌ ΠΈΠ»ΠΈ ΡƒΠΌΠ΅Π½ΡŒΡˆΠ΅Π½ΠΈΠ΅ΠΌ скорости, Π½ΠΎ Π½Π΅ с Π΅Π΅ Π½Π°ΠΏΡ€Π°Π²Π»Π΅Π½ΠΈΠ΅ΠΌ.

Рис. 3. Π¦Π΅Π½Ρ‚Ρ€ΠΎΡΡ‚Ρ€Π΅ΠΌΠΈΡ‚Π΅Π»ΡŒΠ½ΠΎΠ΅ ускорСниС a c Π²ΠΎΠ·Π½ΠΈΠΊΠ°Π΅Ρ‚ ΠΏΡ€ΠΈ ΠΈΠ·ΠΌΠ΅Π½Π΅Π½ΠΈΠΈ направлСния скорости; ΠΎΠ½ пСрпСндикулярСн ΠΊΡ€ΡƒΠ³ΠΎΠ²ΠΎΠΌΡƒ двиТСнию.Π’Π°ΠΊΠΈΠΌ ΠΎΠ±Ρ€Π°Π·ΠΎΠΌ, Ρ†Π΅Π½Ρ‚Ρ€ΠΎΡΡ‚Ρ€Π΅ΠΌΠΈΡ‚Π΅Π»ΡŒΠ½ΠΎΠ΅ ΠΈ Ρ‚Π°Π½Π³Π΅Π½Ρ†ΠΈΠ°Π»ΡŒΠ½ΠΎΠ΅ ускорСния пСрпСндикулярны Π΄Ρ€ΡƒΠ³ Π΄Ρ€ΡƒΠ³Ρƒ.

Π’Π΅ΠΏΠ΅Ρ€ΡŒ ΠΌΡ‹ ΠΌΠΎΠΆΠ΅ΠΌ Π½Π°ΠΉΡ‚ΠΈ Ρ‚ΠΎΡ‡Π½ΠΎΠ΅ ΡΠΎΠΎΡ‚Π½ΠΎΡˆΠ΅Π½ΠΈΠ΅ ΠΌΠ΅ΠΆΠ΄Ρƒ Π»ΠΈΠ½Π΅ΠΉΠ½Ρ‹ΠΌ ускорСниСм a t ΠΈ ΡƒΠ³Π»ΠΎΠ²Ρ‹ΠΌ ускорСниСм Ξ± . ΠŸΠΎΡΠΊΠΎΠ»ΡŒΠΊΡƒ Π»ΠΈΠ½Π΅ΠΉΠ½ΠΎΠ΅ ускорСниС ΠΏΡ€ΠΎΠΏΠΎΡ€Ρ†ΠΈΠΎΠ½Π°Π»ΡŒΠ½ΠΎ измСнСнию Π²Π΅Π»ΠΈΡ‡ΠΈΠ½Ρ‹ скорости, ΠΎΠ½ΠΎ ΠΎΠΏΡ€Π΅Π΄Π΅Π»Π΅Π½ΠΎ (ΠΊΠ°ΠΊ это Π±Ρ‹Π»ΠΎ Π² ΠΎΠ΄Π½ΠΎΠΌΠ΅Ρ€Π½ΠΎΠΉ ΠΊΠΈΠ½Π΅ΠΌΠ°Ρ‚ΠΈΠΊΠ΅) Ρ€Π°Π²Π½Ρ‹ΠΌ

.

[латСкс] {a} _ {\ text {t}} = \ frac {\ Delta v} {\ Delta t} \\ [/ latex].

Для ΠΊΡ€ΡƒΠ³ΠΎΠ²ΠΎΠ³ΠΎ двиТСния ΠΎΠ±Ρ€Π°Ρ‚ΠΈΡ‚Π΅ Π²Π½ΠΈΠΌΠ°Π½ΠΈΠ΅, Ρ‡Ρ‚ΠΎ v = rΟ‰ , Ρ‚Π°ΠΊ Ρ‡Ρ‚ΠΎ

[латСкс] {a} _ {\ text {t}} = \ frac {\ Delta \ left (\ mathrm {r \ omega} \ right)} {\ Delta t} \\ [/ latex].

Радиус r постоянСн для ΠΊΡ€ΡƒΠ³ΠΎΠ²ΠΎΠ³ΠΎ двиТСния, поэтому Ξ” ( rΟ‰ ) = r (Ξ” Ο‰ ). Π’Π°ΠΊΠΈΠΌ ΠΎΠ±Ρ€Π°Π·ΠΎΠΌ,

[латСкс] {a} _ {\ text {t}} = r \ frac {\ Delta \ omega} {\ Delta t} \\ [/ latex].

По ΠΎΠΏΡ€Π΅Π΄Π΅Π»Π΅Π½ΠΈΡŽ, [латСкс] \ alpha = \ frac {\ Delta \ omega} {\ Delta t} \\ [/ latex]. Π’Π°ΠΊΠΈΠΌ ΠΎΠ±Ρ€Π°Π·ΠΎΠΌ,

a t = rΞ± ,

ΠΈΠ»ΠΈ

[латСкс] \ alpha = \ frac {{a} _ {\ text {t}}} {r} \\ [/ latex]

Π­Ρ‚ΠΈ уравнСния ΠΎΠ·Π½Π°Ρ‡Π°ΡŽΡ‚, Ρ‡Ρ‚ΠΎ Π»ΠΈΠ½Π΅ΠΉΠ½ΠΎΠ΅ ускорСниС ΠΈ ΡƒΠ³Π»ΠΎΠ²ΠΎΠ΅ ускорСниС прямо ΠΏΡ€ΠΎΠΏΠΎΡ€Ρ†ΠΈΠΎΠ½Π°Π»ΡŒΠ½Ρ‹.Π§Π΅ΠΌ большС ΡƒΠ³Π»ΠΎΠ²ΠΎΠ΅ ускорСниС, Ρ‚Π΅ΠΌ большС Π»ΠΈΠ½Π΅ΠΉΠ½ΠΎΠ΅ (Ρ‚Π°Π½Π³Π΅Π½Ρ†ΠΈΠ°Π»ΡŒΠ½ΠΎΠ΅) ускорСниС, ΠΈ Π½Π°ΠΎΠ±ΠΎΡ€ΠΎΡ‚. НапримСр, Ρ‡Π΅ΠΌ большС ΡƒΠ³Π»ΠΎΠ²ΠΎΠ΅ ускорСниС Π²Π΅Π΄ΡƒΡ‰ΠΈΡ… колСс автомобиля, Ρ‚Π΅ΠΌ большС ускорСниС автомобиля. Радиус Ρ‚ΠΎΠΆΠ΅ ΠΈΠΌΠ΅Π΅Ρ‚ Π·Π½Π°Ρ‡Π΅Π½ΠΈΠ΅. НапримСр, Ρ‡Π΅ΠΌ мСньшС колСсо, Ρ‚Π΅ΠΌ мСньшС Π΅Π³ΠΎ Π»ΠΈΠ½Π΅ΠΉΠ½ΠΎΠ΅ ускорСниС для Π΄Π°Π½Π½ΠΎΠ³ΠΎ ΡƒΠ³Π»ΠΎΠ²ΠΎΠ³ΠΎ ускорСния Ξ± .

ΠŸΡ€ΠΈΠΌΠ΅Ρ€ 2. РасчСт ΡƒΠ³Π»ΠΎΠ²ΠΎΠ³ΠΎ ускорСния колСса ΠΌΠΎΡ‚ΠΎΡ†ΠΈΠΊΠ»Π°

ΠœΠΎΡ‰Π½Ρ‹ΠΉ ΠΌΠΎΡ‚ΠΎΡ†ΠΈΠΊΠ» ΠΌΠΎΠΆΠ΅Ρ‚ Ρ€Π°Π·Π³ΠΎΠ½ΡΡ‚ΡŒΡΡ ΠΎΡ‚ 0 Π΄ΠΎ 30.0 ΠΌ / с (ΠΎΠΊΠΎΠ»ΠΎ 108 ΠΊΠΌ / Ρ‡) Π·Π° 4,20 с. Каково ΡƒΠ³Π»ΠΎΠ²ΠΎΠ΅ ускорСниС Π΅Π³ΠΎ колСс радиусом 0,320 ΠΌ? (Π‘ΠΌ. Рисунок 4.)

Рис. 4. Π›ΠΈΠ½Π΅ΠΉΠ½ΠΎΠ΅ ускорСниС ΠΌΠΎΡ‚ΠΎΡ†ΠΈΠΊΠ»Π° сопровоТдаСтся ΡƒΠ³Π»ΠΎΠ²Ρ‹ΠΌ ускорСниСм Π΅Π³ΠΎ колСс.

БтратСгия

Нам Π΄Π°Π½Π° информация ΠΎ Π»ΠΈΠ½Π΅ΠΉΠ½Ρ‹Ρ… скоростях ΠΌΠΎΡ‚ΠΎΡ†ΠΈΠΊΠ»Π°. Π’Π°ΠΊΠΈΠΌ ΠΎΠ±Ρ€Π°Π·ΠΎΠΌ, ΠΌΡ‹ ΠΌΠΎΠΆΠ΅ΠΌ Π½Π°ΠΉΡ‚ΠΈ Π΅Π³ΠΎ Π»ΠΈΠ½Π΅ΠΉΠ½ΠΎΠ΅ ускорСниС a t . Π—Π°Ρ‚Π΅ΠΌ Π²Ρ‹Ρ€Π°ΠΆΠ΅Π½ΠΈΠ΅ [latex] \ alpha = \ frac {{a} _ {\ text {t}}} {r} \\ [/ latex] ΠΌΠΎΠΆΠ΅Ρ‚ ΠΈΡΠΏΠΎΠ»ΡŒΠ·ΠΎΠ²Π°Ρ‚ΡŒΡΡ для опрСдСлСния ΡƒΠ³Π»ΠΎΠ²ΠΎΠ³ΠΎ ускорСния.{2} \ end {array} \\ [/ latex].

ΠžΠ±ΡΡƒΠΆΠ΄Π΅Π½ΠΈΠ΅

Π Π°Π΄ΠΈΠ°Π½Ρ‹ Π±Π΅Π·Ρ€Π°Π·ΠΌΠ΅Ρ€Π½Ρ‹ ΠΈ ΠΏΡ€ΠΈΡΡƒΡ‚ΡΡ‚Π²ΡƒΡŽΡ‚ Π² любом ΡΠΎΠΎΡ‚Π½ΠΎΡˆΠ΅Π½ΠΈΠΈ ΠΌΠ΅ΠΆΠ΄Ρƒ ΡƒΠ³Π»ΠΎΠ²Ρ‹ΠΌΠΈ ΠΈ Π»ΠΈΠ½Π΅ΠΉΠ½Ρ‹ΠΌΠΈ Π²Π΅Π»ΠΈΡ‡ΠΈΠ½Π°ΠΌΠΈ.

Π˜Ρ‚Π°ΠΊ, ΠΌΡ‹ ΠΎΠΏΡ€Π΅Π΄Π΅Π»ΠΈΠ»ΠΈ Ρ‚Ρ€ΠΈ Π²Ρ€Π°Ρ‰Π°Ρ‚Π΅Π»ΡŒΠ½Ρ‹Π΅ Π²Π΅Π»ΠΈΡ‡ΠΈΠ½Ρ‹ – ΞΈ, Ο‰ ΠΈ Ξ± . Π­Ρ‚ΠΈ Π²Π΅Π»ΠΈΡ‡ΠΈΠ½Ρ‹ Π°Π½Π°Π»ΠΎΠ³ΠΈΡ‡Π½Ρ‹ трансляционным Π²Π΅Π»ΠΈΡ‡ΠΈΠ½Π°ΠΌ x, v ΠΈ a . Π’ Ρ‚Π°Π±Π»ΠΈΡ†Π΅ 1 ΠΏΠΎΠΊΠ°Π·Π°Π½Ρ‹ Π²Ρ€Π°Ρ‰Π°Ρ‚Π΅Π»ΡŒΠ½Ρ‹Π΅ Π²Π΅Π»ΠΈΡ‡ΠΈΠ½Ρ‹, Π°Π½Π°Π»ΠΎΠ³ΠΈΡ‡Π½Ρ‹Π΅ ΠΏΠΎΡΡ‚ΡƒΠΏΠ°Ρ‚Π΅Π»ΡŒΠ½Ρ‹Π΅ Π²Π΅Π»ΠΈΡ‡ΠΈΠ½Ρ‹ ΠΈ ΠΎΡ‚Π½ΠΎΡˆΠ΅Π½ΠΈΡ ΠΌΠ΅ΠΆΠ΄Ρƒ Π½ΠΈΠΌΠΈ.

Π’Π°Π±Π»ΠΈΡ†Π° 1. Π’Ρ€Π°Ρ‰Π°Ρ‚Π΅Π»ΡŒΠ½Ρ‹Π΅ ΠΈ ΠΏΠΎΡΡ‚ΡƒΠΏΠ°Ρ‚Π΅Π»ΡŒΠ½Ρ‹Π΅ Π²Π΅Π»ΠΈΡ‡ΠΈΠ½Ρ‹
Ρ€ΠΎΡ‚Π°Ρ†ΠΈΠΎΠ½Π½Ρ‹ΠΉ Врансляционный ΠžΡ‚Π½ΠΎΡˆΠ΅Π½ΠΈΡ
ΞΈ x [латСкс] \ theta = \ frac {x} {r} \\ [/ latex]
Ο‰ Π² [латСкс] \ omega = \ frac {v} {r} \\ [/ латСкс]
Ξ± Π° [латСкс] \ alpha = \ frac {{a} _ {t}} {r} \\ [/ latex]

УстановлСниС соСдинСний: практичСский экспСримСнт

Π‘ΡΠ΄ΡŒΡ‚Π΅, поставив Π½ΠΎΠ³ΠΈ Π½Π° зСмлю, Π½Π° Π²Ρ€Π°Ρ‰Π°ΡŽΡ‰ΠΈΠΉΡΡ стул.ΠŸΠΎΠ΄Π½ΠΈΠΌΠΈΡ‚Π΅ ΠΎΠ΄Π½Ρƒ Π½ΠΎΠ³Ρƒ Ρ‚Π°ΠΊ, Ρ‡Ρ‚ΠΎΠ±Ρ‹ ΠΎΠ½Π° Π±Ρ‹Π»Π° Ρ€Π°Π·ΠΎΠ³Π½ΡƒΡ‚Π° (выпрямлСна). Π˜ΡΠΏΠΎΠ»ΡŒΠ·ΡƒΡ Π΄Ρ€ΡƒΠ³ΡƒΡŽ Π½ΠΎΠ³Ρƒ, Π½Π°Ρ‡Π½ΠΈΡ‚Π΅ Π²Ρ€Π°Ρ‰Π°Ρ‚ΡŒΡΡ, ΠΎΡ‚Ρ‚Π°Π»ΠΊΠΈΠ²Π°ΡΡΡŒ ΠΎΡ‚ Π·Π΅ΠΌΠ»ΠΈ. ΠŸΡ€Π΅ΠΊΡ€Π°Ρ‚ΠΈΡ‚Π΅ Ρ‚ΠΎΠ»ΠΊΠ°Ρ‚ΡŒ зСмлю Π½ΠΎΠ³ΠΎΠΉ, Π½ΠΎ ΠΏΠΎΠ·Π²ΠΎΠ»ΡŒΡ‚Π΅ стулу Π²Ρ€Π°Ρ‰Π°Ρ‚ΡŒΡΡ. ΠžΡ‚ исходной Ρ‚ΠΎΡ‡ΠΊΠΈ, с ΠΊΠΎΡ‚ΠΎΡ€ΠΎΠΉ Π²Ρ‹ Π½Π°Ρ‡Π°Π»ΠΈ, нарисуйтС ΡƒΠ³ΠΎΠ», ΡƒΠ³Π»ΠΎΠ²ΡƒΡŽ ΡΠΊΠΎΡ€ΠΎΡΡ‚ΡŒ ΠΈ ΡƒΠ³Π»ΠΎΠ²ΠΎΠ΅ ускорСниС вашСй Π½ΠΎΠ³ΠΈ ΠΊΠ°ΠΊ Ρ„ΡƒΠ½ΠΊΡ†ΠΈΡŽ Π²Ρ€Π΅ΠΌΠ΅Π½ΠΈ Π² Π²ΠΈΠ΄Π΅ Ρ‚Ρ€Π΅Ρ… ΠΎΡ‚Π΄Π΅Π»ΡŒΠ½Ρ‹Ρ… Π³Ρ€Π°Ρ„ΠΈΠΊΠΎΠ². ΠžΡ†Π΅Π½ΠΈΡ‚Π΅ Π²Π΅Π»ΠΈΡ‡ΠΈΠ½Ρƒ этих Π²Π΅Π»ΠΈΡ‡ΠΈΠ½.

ΠŸΡ€ΠΎΠ²Π΅Ρ€ΡŒΡ‚Π΅ своС ΠΏΠΎΠ½ΠΈΠΌΠ°Π½ΠΈΠ΅

Π£Π³Π»ΠΎΠ²ΠΎΠ΅ ускорСниС – это Π²Π΅ΠΊΡ‚ΠΎΡ€, ΠΈΠΌΠ΅ΡŽΡ‰ΠΈΠΉ ΠΊΠ°ΠΊ Π²Π΅Π»ΠΈΡ‡ΠΈΠ½Ρƒ, Ρ‚Π°ΠΊ ΠΈ Π½Π°ΠΏΡ€Π°Π²Π»Π΅Π½ΠΈΠ΅.Как ΠΎΠ±ΠΎΠ·Π½Π°Ρ‡ΠΈΡ‚ΡŒ Π΅Π³ΠΎ Π²Π΅Π»ΠΈΡ‡ΠΈΠ½Ρƒ ΠΈ Π½Π°ΠΏΡ€Π°Π²Π»Π΅Π½ΠΈΠ΅? ΠŸΡ€ΠΎΠΈΠ»Π»ΡŽΡΡ‚Ρ€ΠΈΡ€ΡƒΠΉΡ‚Π΅ Π½Π° ΠΏΡ€ΠΈΠΌΠ΅Ρ€Π΅.

РСшСниС

Π’Π΅Π»ΠΈΡ‡ΠΈΠ½Π° ΡƒΠ³Π»ΠΎΠ²ΠΎΠ³ΠΎ ускорСния составляСт Ξ± , Π° Π΅Π΅ Π½Π°ΠΈΠ±ΠΎΠ»Π΅Π΅ распространСнныС Π΅Π΄ΠΈΠ½ΠΈΡ†Ρ‹ – Ρ€Π°Π΄ / с 2 . НаправлСниС ΡƒΠ³Π»ΠΎΠ²ΠΎΠ³ΠΎ ускорСния вдоль фиксированной оси обозначаСтся Π·Π½Π°ΠΊΠΎΠΌ + ΠΈΠ»ΠΈ a -, Ρ‚Π°ΠΊ ΠΆΠ΅ ΠΊΠ°ΠΊ Π½Π°ΠΏΡ€Π°Π²Π»Π΅Π½ΠΈΠ΅ Π»ΠΈΠ½Π΅ΠΉΠ½ΠΎΠ³ΠΎ ускорСния Π² ΠΎΠ΄Π½ΠΎΠΌ ΠΈΠ·ΠΌΠ΅Ρ€Π΅Π½ΠΈΠΈ обозначаСтся Π·Π½Π°ΠΊΠΎΠΌ + ΠΈΠ»ΠΈ a -. НапримСр, ΠΏΡ€Π΅Π΄ΡΡ‚Π°Π²ΡŒΡ‚Π΅, Ρ‡Ρ‚ΠΎ гимнастка Π΄Π΅Π»Π°Π΅Ρ‚ ΡΠ°Π»ΡŒΡ‚ΠΎ Π²ΠΏΠ΅Ρ€Π΅Π΄. Π•Π΅ ΡƒΠ³Π»ΠΎΠ²ΠΎΠΉ ΠΌΠΎΠΌΠ΅Π½Ρ‚ Π±Ρ‹Π» Π±Ρ‹ ΠΏΠ°Ρ€Π°Π»Π»Π΅Π»Π΅Π½ ΠΊΠΎΠ²Ρ€Ρƒ слСва ΠΎΡ‚ Π½Π΅Π΅.Π’Π΅Π»ΠΈΡ‡ΠΈΠ½Π° Π΅Π΅ ΡƒΠ³Π»ΠΎΠ²ΠΎΠ³ΠΎ ускорСния Π±ΡƒΠ΄Π΅Ρ‚ ΠΏΡ€ΠΎΠΏΠΎΡ€Ρ†ΠΈΠΎΠ½Π°Π»ΡŒΠ½Π° Π΅Π΅ ΡƒΠ³Π»ΠΎΠ²ΠΎΠΉ скорости (скорости вращСния) ΠΈ Π΅Π΅ ΠΌΠΎΠΌΠ΅Π½Ρ‚Ρƒ ΠΈΠ½Π΅Ρ€Ρ†ΠΈΠΈ ΠΎΡ‚Π½ΠΎΡΠΈΡ‚Π΅Π»ΡŒΠ½ΠΎ оси вращСния.

ИсслСдования PhET: Ladybug Revolution

ΠŸΡ€ΠΈΡΠΎΠ΅Π΄ΠΈΠ½ΡΠΉΡ‚Π΅ΡΡŒ ΠΊ боТьСй ΠΊΠΎΡ€ΠΎΠ²ΠΊΠ΅ ΠΈ исслСдуйтС Π²Ρ€Π°Ρ‰Π°Ρ‚Π΅Π»ΡŒΠ½ΠΎΠ΅ Π΄Π²ΠΈΠΆΠ΅Π½ΠΈΠ΅. Π’Ρ€Π°Ρ‰Π°ΠΉΡ‚Π΅ ΠΊΠ°Ρ€ΡƒΡΠ΅Π»ΡŒ, Ρ‡Ρ‚ΠΎΠ±Ρ‹ ΠΈΠ·ΠΌΠ΅Π½ΠΈΡ‚ΡŒ Π΅Π΅ ΡƒΠ³ΠΎΠ», ΠΈΠ»ΠΈ Π²Ρ‹Π±Π΅Ρ€ΠΈΡ‚Π΅ ΠΏΠΎΡΡ‚ΠΎΡΠ½Π½ΡƒΡŽ ΡƒΠ³Π»ΠΎΠ²ΡƒΡŽ ΡΠΊΠΎΡ€ΠΎΡΡ‚ΡŒ ΠΈΠ»ΠΈ ΡƒΠ³Π»ΠΎΠ²ΠΎΠ΅ ускорСниС. Π˜Π·ΡƒΡ‡ΠΈΡ‚Π΅, ΠΊΠ°ΠΊ ΠΊΡ€ΡƒΠ³ΠΎΠ²ΠΎΠ΅ Π΄Π²ΠΈΠΆΠ΅Π½ΠΈΠ΅ связано с ΠΊΠΎΠΎΡ€Π΄ΠΈΠ½Π°Ρ‚Π°ΠΌΠΈ x, y, ΡΠΊΠΎΡ€ΠΎΡΡ‚ΡŒΡŽ ΠΈ ускорСниСм ΠΆΡƒΠΊΠ°, ΠΈΡΠΏΠΎΠ»ΡŒΠ·ΡƒΡ Π²Π΅ΠΊΡ‚ΠΎΡ€Ρ‹ ΠΈΠ»ΠΈ Π³Ρ€Π°Ρ„ΠΈΠΊΠΈ.

Π©Π΅Π»ΠΊΠ½ΠΈΡ‚Π΅, Ρ‡Ρ‚ΠΎΠ±Ρ‹ Π·Π°Π³Ρ€ΡƒΠ·ΠΈΡ‚ΡŒ ΡΠΈΠΌΡƒΠ»ΡΡ†ΠΈΡŽ. Π—Π°ΠΏΡƒΡΠΊΠ°Ρ‚ΡŒ Π½Π° Java.

Π‘Π²ΠΎΠ΄ΠΊΠ° Ρ€Π°Π·Π΄Π΅Π»Π°

  • Π Π°Π²Π½ΠΎΠΌΠ΅Ρ€Π½ΠΎΠ΅ ΠΊΡ€ΡƒΠ³ΠΎΠ²ΠΎΠ΅ Π΄Π²ΠΈΠΆΠ΅Π½ΠΈΠ΅ – это Π΄Π²ΠΈΠΆΠ΅Π½ΠΈΠ΅ с постоянной ΡƒΠ³Π»ΠΎΠ²ΠΎΠΉ ΡΠΊΠΎΡ€ΠΎΡΡ‚ΡŒΡŽ [latex] \ omega = \ frac {\ Delta \ theta} {\ Delta t} \\ [/ latex].
  • ΠŸΡ€ΠΈ Π½Π΅Ρ€Π°Π²Π½ΠΎΠΌΠ΅Ρ€Π½ΠΎΠΌ ΠΊΡ€ΡƒΠ³ΠΎΠ²ΠΎΠΌ Π΄Π²ΠΈΠΆΠ΅Π½ΠΈΠΈ ΡΠΊΠΎΡ€ΠΎΡΡ‚ΡŒ измСняСтся со Π²Ρ€Π΅ΠΌΠ΅Π½Π΅ΠΌ, Π° ΡΠΊΠΎΡ€ΠΎΡΡ‚ΡŒ измСнСния ΡƒΠ³Π»ΠΎΠ²ΠΎΠΉ скорости (Ρ‚.Π΅. ΡƒΠ³Π»ΠΎΠ²ΠΎΠ΅ ускорСниС) Ρ€Π°Π²Π½Π° [latex] \ alpha = \ frac {\ Delta \ omega} {\ Delta t} \\ [/ латСкс].
  • Π›ΠΈΠ½Π΅ΠΉΠ½ΠΎΠ΅ ΠΈΠ»ΠΈ Ρ‚Π°Π½Π³Π΅Π½Ρ†ΠΈΠ°Π»ΡŒΠ½ΠΎΠ΅ ускорСниС относится ΠΊ измСнСниям Π²Π΅Π»ΠΈΡ‡ΠΈΠ½Ρ‹ скорости, Π½ΠΎ Π½Π΅ Π΅Π΅ направлСния, Π·Π°Π΄Π°Π½Π½ΠΎΠΌΡƒ ΠΊΠ°ΠΊ [latex] {a} _ {\ text {t}} = \ frac {\ Delta v} {\ Delta t} \\ [ /латСкс].
  • Для ΠΊΡ€ΡƒΠ³ΠΎΠ²ΠΎΠ³ΠΎ двиТСния ΠΎΠ±Ρ€Π°Ρ‚ΠΈΡ‚Π΅ Π²Π½ΠΈΠΌΠ°Π½ΠΈΠ΅, Ρ‡Ρ‚ΠΎ [latex] v = \ mathrm {r \ omega} [/ latex], Ρ‚Π°ΠΊ Ρ‡Ρ‚ΠΎ

    [латСкс] {a} _ {\ mathrm {\ text {t}}} = \ frac {\ Delta \ left (\ mathrm {r \ omega} \ right)} {\ Delta t} \\ [/ latex] .

  • Радиус r постоянСн для ΠΊΡ€ΡƒΠ³ΠΎΠ²ΠΎΠ³ΠΎ двиТСния, поэтому [latex] \ mathrm {\ Delta} \ left (\ mathrm {r \ omega} \ right) = r \ Delta \ omega \\ [/ latex]. Π’Π°ΠΊΠΈΠΌ ΠΎΠ±Ρ€Π°Π·ΠΎΠΌ,

    [латСкс] {a} _ {\ text {t}} = r \ frac {\ Delta \ omega} {\ Delta t} \\ [/ latex].

  • По ΠΎΠΏΡ€Π΅Π΄Π΅Π»Π΅Π½ΠΈΡŽ [латСкс] \ Delta \ omega / \ Delta t = \ alpha \\ [/ latex].Π’Π°ΠΊΠΈΠΌ ΠΎΠ±Ρ€Π°Π·ΠΎΠΌ,

    [латСкс] {a} _ {\ text {t}} = \ mathrm {r \ alpha} \\ [/ latex]

    ΠΈΠ»ΠΈ

    [латСкс] \ alpha = \ frac {{a} _ {\ text {t}}} {r} \\ [/ latex].

ΠšΠΎΠ½Ρ†Π΅ΠΏΡ‚ΡƒΠ°Π»ΡŒΠ½Ρ‹Π΅ вопросы

1. ΠœΠ΅ΠΆΠ΄Ρƒ Π²Ρ€Π°Ρ‰Π°Ρ‚Π΅Π»ΡŒΠ½Ρ‹ΠΌΠΈ ΠΈ ΠΏΠΎΡΡ‚ΡƒΠΏΠ°Ρ‚Π΅Π»ΡŒΠ½Ρ‹ΠΌΠΈ физичСскими Π²Π΅Π»ΠΈΡ‡ΠΈΠ½Π°ΠΌΠΈ ΡΡƒΡ‰Π΅ΡΡ‚Π²ΡƒΡŽΡ‚ Π°Π½Π°Π»ΠΎΠ³ΠΈΠΈ. ΠžΠΏΡ€Π΅Π΄Π΅Π»ΠΈΡ‚Π΅ Π²Ρ€Π°Ρ‰Π°Ρ‚Π΅Π»ΡŒΠ½Ρ‹ΠΉ Ρ‡Π»Π΅Π½, Π°Π½Π°Π»ΠΎΠ³ΠΈΡ‡Π½Ρ‹ΠΉ ΠΊΠ°ΠΆΠ΄ΠΎΠΌΡƒ ΠΈΠ· ΡΠ»Π΅Π΄ΡƒΡŽΡ‰ΠΈΡ…: ускорСниС, сила, масса, Ρ€Π°Π±ΠΎΡ‚Π°, ΠΏΠΎΡΡ‚ΡƒΠΏΠ°Ρ‚Π΅Π»ΡŒΠ½Π°Ρ кинСтичСская энСргия, Π»ΠΈΠ½Π΅ΠΉΠ½Ρ‹ΠΉ ΠΈΠΌΠΏΡƒΠ»ΡŒΡ, ΠΈΠΌΠΏΡƒΠ»ΡŒΡ.

2. ΠžΠ±ΡŠΡΡΠ½ΠΈΡ‚Π΅, ΠΏΠΎΡ‡Π΅ΠΌΡƒ Ρ†Π΅Π½Ρ‚Ρ€ΠΎΡΡ‚Ρ€Π΅ΠΌΠΈΡ‚Π΅Π»ΡŒΠ½ΠΎΠ΅ ускорСниС мСняСт Π½Π°ΠΏΡ€Π°Π²Π»Π΅Π½ΠΈΠ΅ скорости ΠΏΡ€ΠΈ ΠΊΡ€ΡƒΠ³ΠΎΠ²ΠΎΠΌ Π΄Π²ΠΈΠΆΠ΅Π½ΠΈΠΈ, Π½ΠΎ Π½Π΅ Π΅Π³ΠΎ Π²Π΅Π»ΠΈΡ‡ΠΈΠ½Ρƒ.

3. ΠŸΡ€ΠΈ ΠΊΡ€ΡƒΠ³ΠΎΠ²ΠΎΠΌ Π΄Π²ΠΈΠΆΠ΅Π½ΠΈΠΈ Ρ‚Π°Π½Π³Π΅Π½Ρ†ΠΈΠ°Π»ΡŒΠ½ΠΎΠ΅ ускорСниС ΠΌΠΎΠΆΠ΅Ρ‚ ΠΈΠ·ΠΌΠ΅Π½ΡΡ‚ΡŒ Π²Π΅Π»ΠΈΡ‡ΠΈΠ½Ρƒ скорости, Π½ΠΎ Π½Π΅ Π΅Π΅ Π½Π°ΠΏΡ€Π°Π²Π»Π΅Π½ΠΈΠ΅. ΠŸΠΎΡΡΠ½ΠΈΡ‚Π΅ свой ΠΎΡ‚Π²Π΅Ρ‚.

4. ΠŸΡ€Π΅Π΄ΠΏΠΎΠ»ΠΎΠΆΠΈΠΌ, Ρ‡Ρ‚ΠΎ блюдо стоит Π½Π° ΠΊΡ€Π°ΡŽ Π²Ρ€Π°Ρ‰Π°ΡŽΡ‰Π΅ΠΉΡΡ пластины ΠΌΠΈΠΊΡ€ΠΎΠ²ΠΎΠ»Π½ΠΎΠ²ΠΎΠΉ ΠΏΠ΅Ρ‡ΠΈ. Π˜ΡΠΏΡ‹Ρ‚Ρ‹Π²Π°Π΅Ρ‚ Π»ΠΈ ΠΎΠ½Π° Π½Π΅Π½ΡƒΠ»Π΅Π²ΠΎΠ΅ Ρ‚Π°Π½Π³Π΅Π½Ρ†ΠΈΠ°Π»ΡŒΠ½ΠΎΠ΅ ускорСниС, Ρ†Π΅Π½Ρ‚Ρ€ΠΎΡΡ‚Ρ€Π΅ΠΌΠΈΡ‚Π΅Π»ΡŒΠ½ΠΎΠ΅ ускорСниС ΠΈΠ»ΠΈ ΠΈ Ρ‚ΠΎ, ΠΈ Π΄Ρ€ΡƒΠ³ΠΎΠ΅, ΠΊΠΎΠ³Π΄Π°: Π°) пластина Π½Π°Ρ‡ΠΈΠ½Π°Π΅Ρ‚ Π²Ρ€Π°Ρ‰Π°Ρ‚ΡŒΡΡ? (Π±) ΠŸΠ»Π°ΡΡ‚ΠΈΠ½Π° вращаСтся с постоянной ΡƒΠ³Π»ΠΎΠ²ΠΎΠΉ ΡΠΊΠΎΡ€ΠΎΡΡ‚ΡŒΡŽ? (c) ΠŸΠ»Π°ΡΡ‚ΠΈΠ½Π° замСдляСтся Π΄ΠΎ остановки?

Π—Π°Π΄Π°Ρ‡ΠΈ ΠΈ упраТнСния

1.На ΠΏΠΈΠΊΠ΅ Ρ‚ΠΎΡ€Π½Π°Π΄ΠΎ ΠΈΠΌΠ΅Π΅Ρ‚ Π΄ΠΈΠ°ΠΌΠ΅Ρ‚Ρ€ 60,0 ΠΌ ΠΈ ΡΠΊΠΎΡ€ΠΎΡΡ‚ΡŒ Π²Π΅Ρ‚Ρ€Π° 500 ΠΊΠΌ / Ρ‡. Какова Π΅Π³ΠΎ угловая ΡΠΊΠΎΡ€ΠΎΡΡ‚ΡŒ Π² ΠΎΠ±ΠΎΡ€ΠΎΡ‚Π°Ρ… Π² сСкунду?

2. Integrated Concepts Π£Π»ΡŒΡ‚Ρ€Π°Ρ†Π΅Π½Ρ‚Ρ€ΠΈΡ„ΡƒΠ³Π° ускоряСтся ΠΎΡ‚ состояния покоя Π΄ΠΎ 100 000 ΠΎΠ± / ΠΌΠΈΠ½ Π·Π° 2,00 ΠΌΠΈΠ½. Π°) Каково Π΅Π³ΠΎ ΡƒΠ³Π»ΠΎΠ²ΠΎΠ΅ ускорСниС Π² Ρ€Π°Π΄ / с 2 ? Π±) Каково Ρ‚Π°Π½Π³Π΅Π½Ρ†ΠΈΠ°Π»ΡŒΠ½ΠΎΠ΅ ускорСниС Ρ‚ΠΎΡ‡ΠΊΠΈ Π½Π° расстоянии 9,50 см ΠΎΡ‚ оси вращСния? (c) Каково Ρ€Π°Π΄ΠΈΠ°Π»ΡŒΠ½ΠΎΠ΅ ускорСниС Π² ΠΌ / с 2 ΠΈ ΠΊΡ€Π°Ρ‚Π½ΠΎΠ΅ g этой Ρ‚ΠΎΡ‡ΠΊΠΈ ΠΏΡ€ΠΈ ΠΏΠΎΠ»Π½Ρ‹Ρ… ΠΎΠ±ΠΎΡ€ΠΎΡ‚Π°Ρ… Π² ΠΌΠΈΠ½ΡƒΡ‚Ρƒ?

3. Integrated Concepts Π£ вас Π΅ΡΡ‚ΡŒ Ρ‚ΠΎΡ‡ΠΈΠ»ΡŒΠ½Ρ‹ΠΉ камСнь (диск) вСсом 90,0 ΠΊΠ³, радиусом 0,340 ΠΌ ΠΈ Π²Ρ€Π°Ρ‰Π°ΡŽΡ‰ΠΈΠΌΡΡ со ΡΠΊΠΎΡ€ΠΎΡΡ‚ΡŒΡŽ 90,0 ΠΎΠ± / ΠΌΠΈΠ½, ΠΈ Π²Ρ‹ ΠΏΡ€ΠΈΠΆΠΈΠΌΠ°Π΅Ρ‚Π΅ ΠΊ Π½Π΅ΠΌΡƒ ΡΡ‚Π°Π»ΡŒΠ½ΠΎΠΉ Ρ‚ΠΎΠΏΠΎΡ€ с Ρ€Π°Π΄ΠΈΠ°Π»ΡŒΠ½ΠΎΠΉ силой 20,0 Н. (a) ΠŸΡ€Π΅Π΄ΠΏΠΎΠ»Π°Π³Π°Ρ, Ρ‡Ρ‚ΠΎ кинСтичСский коэффициСнт трСния ΠΌΠ΅ΠΆΠ΄Ρƒ ΡΡ‚Π°Π»ΡŒΡŽ ΠΈ ΠΊΠ°ΠΌΠ½Π΅ΠΌ Ρ€Π°Π²Π΅Π½ 0,20, рассчитайтС ΡƒΠ³Π»ΠΎΠ²ΠΎΠ΅ ускорСниС Ρ‚ΠΎΡ‡ΠΈΠ»ΡŒΠ½ΠΎΠ³ΠΎ камня. Π±) Бколько ΠΎΠ±ΠΎΡ€ΠΎΡ‚ΠΎΠ² сдСлаСт камСнь, ΠΏΡ€Π΅ΠΆΠ΄Π΅ Ρ‡Π΅ΠΌ остановится?

4. НСобоснованныС Ρ€Π΅Π·ΡƒΠ»ΡŒΡ‚Π°Ρ‚Ρ‹ Π’Π°ΠΌ говорят, Ρ‡Ρ‚ΠΎ баскСтболист Π²Ρ€Π°Ρ‰Π°Π΅Ρ‚ мяч с ΡƒΠ³Π»ΠΎΠ²Ρ‹ΠΌ ускорСниСм 100 Ρ€Π°Π΄ / с 2 .(Π°) Какова конСчная угловая ΡΠΊΠΎΡ€ΠΎΡΡ‚ΡŒ мяча, Ссли мяч стартуСт ΠΈΠ· состояния покоя ΠΈ ускорСниС длится 2,00 с? Π±) Π§Ρ‚ΠΎ Π½Π΅Ρ€Π°Π·ΡƒΠΌΠ½ΠΎΠ³ΠΎ Π² Ρ€Π΅Π·ΡƒΠ»ΡŒΡ‚Π°Ρ‚Π΅? (c) КакиС посылки нСобоснованны ΠΈΠ»ΠΈ Π½Π΅ΠΏΠΎΡΠ»Π΅Π΄ΠΎΠ²Π°Ρ‚Π΅Π»ΡŒΠ½Ρ‹?

Глоссарий

ΡƒΠ³Π»ΠΎΠ²ΠΎΠ΅ ускорСниС:
Π‘ΠΊΠΎΡ€ΠΎΡΡ‚ΡŒ измСнСния ΡƒΠ³Π»ΠΎΠ²ΠΎΠΉ скорости Π²ΠΎ Π²Ρ€Π΅ΠΌΠ΅Π½ΠΈ
ΠΈΠ·ΠΌΠ΅Π½Π΅Π½ΠΈΠ΅ ΡƒΠ³Π»ΠΎΠ²ΠΎΠΉ скорости:
Ρ€Π°Π·Π½ΠΈΡ†Π° ΠΌΠ΅ΠΆΠ΄Ρƒ ΠΊΠΎΠ½Π΅Ρ‡Π½Ρ‹ΠΌ ΠΈ Π½Π°Ρ‡Π°Π»ΡŒΠ½Ρ‹ΠΌ значСниями ΡƒΠ³Π»ΠΎΠ²ΠΎΠΉ скорости
Ρ‚Π°Π½Π³Π΅Π½Ρ†ΠΈΠ°Π»ΡŒΠ½ΠΎΠ΅ ускорСниС
ускорСниС Π² Π½Π°ΠΏΡ€Π°Π²Π»Π΅Π½ΠΈΠΈ, ΠΊΠ°ΡΠ°Ρ‚Π΅Π»ΡŒΠ½ΠΎΠΌ ΠΊ окруТности Π² ΠΈΠ½Ρ‚Π΅Ρ€Π΅ΡΡƒΡŽΡ‰Π΅ΠΉ Ρ‚ΠΎΡ‡ΠΊΠ΅ ΠΏΡ€ΠΈ ΠΊΡ€ΡƒΠ³ΠΎΠ²ΠΎΠΌ Π΄Π²ΠΈΠΆΠ΅Π½ΠΈΠΈ

Π˜Π·Π±Ρ€Π°Π½Π½Ρ‹Π΅ Ρ€Π΅ΡˆΠ΅Π½ΠΈΡ ΠΏΡ€ΠΎΠ±Π»Π΅ΠΌ ΠΈ упраТнСния

1. Ο‰ = 0,737 ΠΎΠ± / с

3. (Π°) βˆ’0.26 Ρ€Π°Π΄ / с 2 (Π±) 27 ΠΎΠ±

4.4 Π Π°Π²Π½ΠΎΠΌΠ΅Ρ€Π½ΠΎΠ΅ ΠΊΡ€ΡƒΠ³ΠΎΠ²ΠΎΠ΅ Π΄Π²ΠΈΠΆΠ΅Π½ΠΈΠ΅ – УнивСрситСтская Ρ„ΠΈΠ·ΠΈΠΊΠ°, Ρ‚ΠΎΠΌ 1

Π¦Π΅Π»ΠΈ обучСния

К ΠΊΠΎΠ½Ρ†Ρƒ этого Ρ€Π°Π·Π΄Π΅Π»Π° Π²Ρ‹ смоТСтС:

  • НайдитС Ρ†Π΅Π½Ρ‚Ρ€ΠΎΡΡ‚Ρ€Π΅ΠΌΠΈΡ‚Π΅Π»ΡŒΠ½ΠΎΠ΅ ускорСниС ΠΎΠ±ΡŠΠ΅ΠΊΡ‚Π°, двиТущСгося ΠΏΠΎ ΠΊΡ€ΡƒΠ³ΠΎΠ²ΠΎΠΉ Ρ‚Ρ€Π°Π΅ΠΊΡ‚ΠΎΡ€ΠΈΠΈ.
  • Π˜ΡΠΏΠΎΠ»ΡŒΠ·ΡƒΠΉΡ‚Π΅ уравнСния ΠΊΡ€ΡƒΠ³ΠΎΠ²ΠΎΠ³ΠΎ двиТСния, Ρ‡Ρ‚ΠΎΠ±Ρ‹ Π½Π°ΠΉΡ‚ΠΈ ΠΏΠΎΠ»ΠΎΠΆΠ΅Π½ΠΈΠ΅, ΡΠΊΠΎΡ€ΠΎΡΡ‚ΡŒ ΠΈ ускорСниС частицы, ΡΠΎΠ²Π΅Ρ€ΡˆΠ°ΡŽΡ‰Π΅ΠΉ ΠΊΡ€ΡƒΠ³ΠΎΠ²ΠΎΠ΅ Π΄Π²ΠΈΠΆΠ΅Π½ΠΈΠ΅.
  • ΠžΠ±ΡŠΡΡΠ½ΠΈΡ‚Π΅ Ρ€Π°Π·Π½ΠΈΡ†Ρƒ ΠΌΠ΅ΠΆΠ΄Ρƒ Ρ†Π΅Π½Ρ‚Ρ€ΠΎΡΡ‚Ρ€Π΅ΠΌΠΈΡ‚Π΅Π»ΡŒΠ½Ρ‹ΠΌ ускорСниСм ΠΈ Ρ‚Π°Π½Π³Π΅Π½Ρ†ΠΈΠ°Π»ΡŒΠ½Ρ‹ΠΌ ускорСниСм, Π²ΠΎΠ·Π½ΠΈΠΊΠ°ΡŽΡ‰ΠΈΠΌ Π² Ρ€Π΅Π·ΡƒΠ»ΡŒΡ‚Π°Ρ‚Π΅ Π½Π΅Ρ€Π°Π²Π½ΠΎΠΌΠ΅Ρ€Π½ΠΎΠ³ΠΎ ΠΊΡ€ΡƒΠ³ΠΎΠ²ΠΎΠ³ΠΎ двиТСния.
  • ΠžΡ†Π΅Π½ΠΈΡ‚Π΅ Ρ†Π΅Π½Ρ‚Ρ€ΠΎΡΡ‚Ρ€Π΅ΠΌΠΈΡ‚Π΅Π»ΡŒΠ½ΠΎΠ΅ ΠΈ Ρ‚Π°Π½Π³Π΅Π½Ρ†ΠΈΠ°Π»ΡŒΠ½ΠΎΠ΅ ускорСниС ΠΏΡ€ΠΈ Π½Π΅Ρ€Π°Π²Π½ΠΎΠΌΠ΅Ρ€Π½ΠΎΠΌ ΠΊΡ€ΡƒΠ³ΠΎΠ²ΠΎΠΌ Π΄Π²ΠΈΠΆΠ΅Π½ΠΈΠΈ ΠΈ Π½Π°ΠΉΠ΄ΠΈΡ‚Π΅ Π²Π΅ΠΊΡ‚ΠΎΡ€ ΠΏΠΎΠ»Π½ΠΎΠ³ΠΎ ускорСния.

Π Π°Π²Π½ΠΎΠΌΠ΅Ρ€Π½ΠΎΠ΅ ΠΊΡ€ΡƒΠ³ΠΎΠ²ΠΎΠ΅ Π΄Π²ΠΈΠΆΠ΅Π½ΠΈΠ΅ – это особый Ρ‚ΠΈΠΏ двиТСния, ΠΏΡ€ΠΈ ΠΊΠΎΡ‚ΠΎΡ€ΠΎΠΌ ΠΎΠ±ΡŠΠ΅ΠΊΡ‚ двиТСтся ΠΏΠΎ ΠΊΡ€ΡƒΠ³Ρƒ с постоянной ΡΠΊΠΎΡ€ΠΎΡΡ‚ΡŒΡŽ. НапримСр, любая Ρ‚ΠΎΡ‡ΠΊΠ° ΠΏΡ€ΠΎΠΏΠ΅Π»Π»Π΅Ρ€Π°, Π²Ρ€Π°Ρ‰Π°ΡŽΡ‰Π΅Π³ΠΎΡΡ с постоянной ΡΠΊΠΎΡ€ΠΎΡΡ‚ΡŒΡŽ, ΡΠΎΠ²Π΅Ρ€ΡˆΠ°Π΅Ρ‚ Ρ€Π°Π²Π½ΠΎΠΌΠ΅Ρ€Π½ΠΎΠ΅ ΠΊΡ€ΡƒΠ³ΠΎΠ²ΠΎΠ΅ Π΄Π²ΠΈΠΆΠ΅Π½ΠΈΠ΅.Π”Ρ€ΡƒΠ³ΠΈΠΌΠΈ ΠΏΡ€ΠΈΠΌΠ΅Ρ€Π°ΠΌΠΈ ΡΠ²Π»ΡΡŽΡ‚ΡΡ сСкундная, минутная ΠΈ часовая стрСлки часов. ΠŸΡ€ΠΈΠΌΠ΅Ρ‡Π°Ρ‚Π΅Π»ΡŒΠ½ΠΎ, Ρ‡Ρ‚ΠΎ Ρ‚ΠΎΡ‡ΠΊΠΈ Π½Π° этих Π²Ρ€Π°Ρ‰Π°ΡŽΡ‰ΠΈΡ…ΡΡ ΠΎΠ±ΡŠΠ΅ΠΊΡ‚Π°Ρ… Π΄Π΅ΠΉΡΡ‚Π²ΠΈΡ‚Π΅Π»ΡŒΠ½ΠΎ ΡƒΡΠΊΠΎΡ€ΡΡŽΡ‚ΡΡ, хотя ΡΠΊΠΎΡ€ΠΎΡΡ‚ΡŒ вращСния постоянна. Π§Ρ‚ΠΎΠ±Ρ‹ ΡƒΠ²ΠΈΠ΄Π΅Ρ‚ΡŒ это, ΠΌΡ‹ Π΄ΠΎΠ»ΠΆΠ½Ρ‹ ΠΏΡ€ΠΎΠ°Π½Π°Π»ΠΈΠ·ΠΈΡ€ΠΎΠ²Π°Ρ‚ΡŒ Π΄Π²ΠΈΠΆΠ΅Π½ΠΈΠ΅ Π² Ρ‚Π΅Ρ€ΠΌΠΈΠ½Π°Ρ… Π²Π΅ΠΊΡ‚ΠΎΡ€ΠΎΠ².

Π¦Π΅Π½Ρ‚Ρ€ΠΎΡΡ‚Ρ€Π΅ΠΌΠΈΡ‚Π΅Π»ΡŒΠ½ΠΎΠ΅ ускорСниС

Π’ ΠΎΠ΄Π½ΠΎΠΌΠ΅Ρ€Π½ΠΎΠΉ ΠΊΠΈΠ½Π΅ΠΌΠ°Ρ‚ΠΈΠΊΠ΅ ΠΎΠ±ΡŠΠ΅ΠΊΡ‚Ρ‹ с постоянной ΡΠΊΠΎΡ€ΠΎΡΡ‚ΡŒΡŽ ΠΈΠΌΠ΅ΡŽΡ‚ Π½ΡƒΠ»Π΅Π²ΠΎΠ΅ ускорСниС. Однако Π² Π΄Π²ΡƒΡ…- ΠΈ Ρ‚Ρ€Π΅Ρ…ΠΌΠ΅Ρ€Π½ΠΎΠΉ ΠΊΠΈΠ½Π΅ΠΌΠ°Ρ‚ΠΈΠΊΠ΅, Π΄Π°ΠΆΠ΅ Ссли ΡΠΊΠΎΡ€ΠΎΡΡ‚ΡŒ постоянна, частица ΠΌΠΎΠΆΠ΅Ρ‚ ΠΈΠΌΠ΅Ρ‚ΡŒ ускорСниС, Ссли ΠΎΠ½Π° двиТСтся ΠΏΠΎ ΠΊΡ€ΠΈΠ²ΠΎΠ»ΠΈΠ½Π΅ΠΉΠ½ΠΎΠΉ Ρ‚Ρ€Π°Π΅ΠΊΡ‚ΠΎΡ€ΠΈΠΈ, Ρ‚Π°ΠΊΠΎΠΉ ΠΊΠ°ΠΊ ΠΎΠΊΡ€ΡƒΠΆΠ½ΠΎΡΡ‚ΡŒ.Π’ этом случаС Π²Π΅ΠΊΡ‚ΠΎΡ€ скорости мСняСтся, ΠΈΠ»ΠΈ

Π­Ρ‚ΠΎ ΠΏΠΎΠΊΠ°Π·Π°Π½ΠΎ Π½Π° (Рисунок). ΠŸΠΎΡΠΊΠΎΠ»ΡŒΠΊΡƒ частица двиТСтся ΠΏΡ€ΠΎΡ‚ΠΈΠ² часовой стрСлки Π²ΠΎ Π²Ρ€Π΅ΠΌΠ΅Π½ΠΈ

ΠΏΠΎ ΠΊΡ€ΡƒΠ³ΠΎΠ²ΠΎΠΉ Ρ‚Ρ€Π°Π΅ΠΊΡ‚ΠΎΡ€ΠΈΠΈ, Π΅Π³ΠΎ Π²Π΅ΠΊΡ‚ΠΎΡ€ полоТСния пСрСмСщаСтся ΠΈΠ·

Π‘

ΠΏΠΎ

Π’Π΅ΠΊΡ‚ΠΎΡ€ скорости ΠΈΠΌΠ΅Π΅Ρ‚ ΠΏΠΎΡΡ‚ΠΎΡΠ½Π½ΡƒΡŽ Π²Π΅Π»ΠΈΡ‡ΠΈΠ½Ρƒ ΠΈ касаСтся ΠΏΡƒΡ‚ΠΈ, ΠΏΠΎΡΠΊΠΎΠ»ΡŒΠΊΡƒ ΠΎΠ½ измСняСтся ΠΎΡ‚

.

Π‘

ΠΏΠΎ

Ρ‚ΠΎΠ»ΡŒΠΊΠΎ мСняСт Π½Π°ΠΏΡ€Π°Π²Π»Π΅Π½ΠΈΠ΅.ΠŸΠΎΡΠΊΠΎΠ»ΡŒΠΊΡƒ Π²Π΅ΠΊΡ‚ΠΎΡ€ скорости

пСрпСндикулярно Π²Π΅ΠΊΡ‚ΠΎΡ€Ρƒ полоТСния

Ρ‚Ρ€Π΅ΡƒΠ³ΠΎΠ»ΡŒΠ½ΠΈΠΊΠΎΠ², ΠΎΠ±Ρ€Π°Π·ΠΎΠ²Π°Π½Π½Ρ‹Ρ… Π²Π΅ΠΊΡ‚ΠΎΡ€Π°ΠΌΠΈ полоТСния ΠΈ

ΠΈ Π²Π΅ΠΊΡ‚ΠΎΡ€Ρ‹ скорости ΠΈ

ΠΏΠΎΡ…ΠΎΠΆΠΈ. ΠšΡ€ΠΎΠΌΠ΅ Ρ‚ΠΎΠ³ΠΎ, с

ΠΈ

Π΄Π²Π° Ρ€Π°Π²Π½ΠΎΠ±Π΅Π΄Ρ€Π΅Π½Π½Ρ‹Ρ… Ρ‚Ρ€Π΅ΡƒΠ³ΠΎΠ»ΡŒΠ½ΠΈΠΊΠ°. Из этих Ρ„Π°ΠΊΡ‚ΠΎΠ² ΠΌΡ‹ ΠΌΠΎΠΆΠ΅ΠΌ ΡΠ΄Π΅Π»Π°Ρ‚ΡŒ ΡƒΡ‚Π²Π΅Ρ€ΠΆΠ΄Π΅Π½ΠΈΠ΅

ΠΈΠ»ΠΈ

Рисунок 4.18 (a) Частица двиТСтся ΠΏΠΎ ΠΊΡ€ΡƒΠ³Ρƒ с постоянной ΡΠΊΠΎΡ€ΠΎΡΡ‚ΡŒΡŽ, Π²Ρ€Π΅ΠΌΠ΅Π½Π°ΠΌΠΈ имСя Π²Π΅ΠΊΡ‚ΠΎΡ€Ρ‹ полоТСния ΠΈ скорости.

ΠΈ

(b) Π’Π΅ΠΊΡ‚ΠΎΡ€Ρ‹ скорости, ΠΎΠ±Ρ€Π°Π·ΡƒΡŽΡ‰ΠΈΠ΅ Ρ‚Ρ€Π΅ΡƒΠ³ΠΎΠ»ΡŒΠ½ΠΈΠΊ. Π”Π²Π° Ρ‚Ρ€Π΅ΡƒΠ³ΠΎΠ»ΡŒΠ½ΠΈΠΊΠ° Π½Π° рисункС ΠΏΠΎΡ…ΠΎΠΆΠΈ. Π’Π΅ΠΊΡ‚ΠΎΡ€

ΡƒΠΊΠ°Π·Ρ‹Π²Π°Π΅Ρ‚ Π½Π° Ρ†Π΅Π½Ρ‚Ρ€ ΠΊΡ€ΡƒΠ³Π° Π² ΠΏΡ€Π΅Π΄Π΅Π»Π°Ρ…

ΠœΡ‹ ΠΌΠΎΠΆΠ΅ΠΌ Π½Π°ΠΉΡ‚ΠΈ Π²Π΅Π»ΠΈΡ‡ΠΈΠ½Ρƒ ускорСния ΠΎΡ‚

НаправлСниС ускорСния Ρ‚Π°ΠΊΠΆΠ΅ ΠΌΠΎΠΆΠ½ΠΎ Π½Π°ΠΉΡ‚ΠΈ, ΠΎΡ‚ΠΌΠ΅Ρ‚ΠΈΠ², Ρ‡Ρ‚ΠΎ ΠΊΠ°ΠΊ

ΠΈ, ΡΠ»Π΅Π΄ΠΎΠ²Π°Ρ‚Π΅Π»ΡŒΠ½ΠΎ,

ΠΏΡ€ΠΈΠ±Π»ΠΈΠΆΠ°ΡŽΡ‚ΡΡ ΠΊ Π½ΡƒΠ»ΡŽ, Π²Π΅ΠΊΡ‚ΠΎΡ€

приблиТаСтся ΠΊ Π½Π°ΠΏΡ€Π°Π²Π»Π΅Π½ΠΈΡŽ, пСрпСндикулярному

Π’ ΠΏΡ€Π΅Π΄Π΅Π»Π΅

пСрпСндикулярно

с

касаСтся окруТности, ускорСниС

ΡƒΠΊΠ°Π·Ρ‹Π²Π°Π΅Ρ‚ Π½Π° Ρ†Π΅Π½Ρ‚Ρ€ ΠΊΡ€ΡƒΠ³Π°.Π’Π°ΠΊΠΈΠΌ ΠΎΠ±Ρ€Π°Π·ΠΎΠΌ, частица, двиТущаяся ΠΏΠΎ ΠΊΡ€ΡƒΠ³Ρƒ с постоянной ΡΠΊΠΎΡ€ΠΎΡΡ‚ΡŒΡŽ, ΠΈΠΌΠ΅Π΅Ρ‚ ускорСниС с Π²Π΅Π»ΠΈΡ‡ΠΈΠ½ΠΎΠΉ

.

НаправлСниС Π²Π΅ΠΊΡ‚ΠΎΡ€Π° ускорСния – ΠΊ Ρ†Π΅Π½Ρ‚Ρ€Ρƒ ΠΊΡ€ΡƒΠ³Π° ((Рисунок)). Π­Ρ‚ΠΎ Ρ€Π°Π΄ΠΈΠ°Π»ΡŒΠ½ΠΎΠ΅ ускорСниС ΠΈ называСтся Ρ†Π΅Π½Ρ‚Ρ€ΠΎΡΡ‚Ρ€Π΅ΠΌΠΈΡ‚Π΅Π»ΡŒΠ½Ρ‹ΠΌ ускорСниСм , поэтому ΠΌΡ‹ Π΄Π°Π΅ΠΌ Π΅ΠΌΡƒ индСкс c. Π‘Π»ΠΎΠ²ΠΎ Ρ†Π΅Π½Ρ‚Ρ€ΠΎΡΡ‚Ρ€Π΅ΠΌΠΈΡ‚Π΅Π»ΡŒΠ½Ρ‹ΠΉ происходит ΠΎΡ‚ латинских слов centrum (Ρ‡Ρ‚ΠΎ ΠΎΠ·Π½Π°Ρ‡Π°Π΅Ρ‚ Β«Ρ†Π΅Π½Ρ‚Ρ€Β») ΠΈ petere (ΠΎΠ·Π½Π°Ρ‡Π°Π΅Ρ‚ ΠΈΡΠΊΠ°Ρ‚ΡŒ Β») ΠΈ, Ρ‚Π°ΠΊΠΈΠΌ ΠΎΠ±Ρ€Π°Π·ΠΎΠΌ, ΠΏΡ€ΠΈΠ½ΠΈΠΌΠ°Π΅Ρ‚ Π·Π½Π°Ρ‡Π΅Π½ΠΈΠ΅Β« поиск Ρ†Π΅Π½Ρ‚Ρ€Π° Β».”

Рис. 4.19. Π’Π΅ΠΊΡ‚ΠΎΡ€ Ρ†Π΅Π½Ρ‚Ρ€ΠΎΡΡ‚Ρ€Π΅ΠΌΠΈΡ‚Π΅Π»ΡŒΠ½ΠΎΠ³ΠΎ ускорСния ΡƒΠΊΠ°Π·Ρ‹Π²Π°Π΅Ρ‚ Π½Π° Ρ†Π΅Π½Ρ‚Ρ€ ΠΊΡ€ΡƒΠ³ΠΎΠ²ΠΎΠΉ Ρ‚Ρ€Π°Π΅ΠΊΡ‚ΠΎΡ€ΠΈΠΈ двиТСния ΠΈ прСдставляСт собой ускорСниС Π² Ρ€Π°Π΄ΠΈΠ°Π»ΡŒΠ½ΠΎΠΌ Π½Π°ΠΏΡ€Π°Π²Π»Π΅Π½ΠΈΠΈ. Π’Π°ΠΊΠΆΠ΅ ΠΏΠΎΠΊΠ°Π·Π°Π½ Π²Π΅ΠΊΡ‚ΠΎΡ€ скорости, ΠΊΠ°ΡΠ°ΡŽΡ‰ΠΈΠΉΡΡ окруТности.

Π”Π°Π²Π°ΠΉΡ‚Π΅ рассмотрим нСсколько ΠΏΡ€ΠΈΠΌΠ΅Ρ€ΠΎΠ², ΠΊΠΎΡ‚ΠΎΡ€Ρ‹Π΅ ΠΈΠ»Π»ΡŽΡΡ‚Ρ€ΠΈΡ€ΡƒΡŽΡ‚ ΠΎΡ‚Π½ΠΎΡΠΈΡ‚Π΅Π»ΡŒΠ½Ρ‹Π΅ Π²Π΅Π»ΠΈΡ‡ΠΈΠ½Ρ‹ скорости, радиуса ΠΈ Ρ†Π΅Π½Ρ‚Ρ€ΠΎΡΡ‚Ρ€Π΅ΠΌΠΈΡ‚Π΅Π»ΡŒΠ½ΠΎΠ³ΠΎ ускорСния.

ΠŸΡ€ΠΈΠΌΠ΅Ρ€

Π‘ΠΎΠ·Π΄Π°Π½ΠΈΠ΅ ускорСния 1
g

Π‘Π°ΠΌΠΎΠ»Π΅Ρ‚ Π»Π΅Ρ‚ΠΈΡ‚ со ΡΠΊΠΎΡ€ΠΎΡΡ‚ΡŒΡŽ 134,1 ΠΌ / с ΠΏΠΎ прямой ΠΈ Π΄Π΅Π»Π°Π΅Ρ‚ Ρ€Π°Π·Π²ΠΎΡ€ΠΎΡ‚ ΠΏΠΎ ΠΊΡ€ΡƒΠ³ΠΎΠ²ΠΎΠΉ Ρ‚Ρ€Π°Π΅ΠΊΡ‚ΠΎΡ€ΠΈΠΈ Π½Π° ΡƒΡ€ΠΎΠ²Π½Π΅ Π·Π΅ΠΌΠ»ΠΈ.Каким Π΄ΠΎΠ»ΠΆΠ΅Π½ Π±Ρ‹Ρ‚ΡŒ радиус окруТности, Ρ‡Ρ‚ΠΎΠ±Ρ‹ Π²Ρ‹Π·Π²Π°Ρ‚ΡŒ Ρ†Π΅Π½Ρ‚Ρ€ΠΎΡΡ‚Ρ€Π΅ΠΌΠΈΡ‚Π΅Π»ΡŒΠ½ΠΎΠ΅ ускорСниС 1 g для ΠΏΠΈΠ»ΠΎΡ‚Π° ΠΈ самолСта ΠΏΠΎ Π½Π°ΠΏΡ€Π°Π²Π»Π΅Π½ΠΈΡŽ ΠΊ Ρ†Π΅Π½Ρ‚Ρ€Ρƒ ΠΊΡ€ΡƒΠ³ΠΎΠ²ΠΎΠΉ Ρ‚Ρ€Π°Π΅ΠΊΡ‚ΠΎΡ€ΠΈΠΈ?

БтратСгия

Учитывая ΡΠΊΠΎΡ€ΠΎΡΡ‚ΡŒ струи, ΠΌΡ‹ ΠΌΠΎΠΆΠ΅ΠΌ Π½Π°ΠΉΡ‚ΠΈ радиус окруТности Π² Π²Ρ‹Ρ€Π°ΠΆΠ΅Π½ΠΈΠΈ для Ρ†Π΅Π½Ρ‚Ρ€ΠΎΡΡ‚Ρ€Π΅ΠΌΠΈΡ‚Π΅Π»ΡŒΠ½ΠΎΠ³ΠΎ ускорСния.

РСшСниС

УстановитС Ρ†Π΅Π½Ρ‚Ρ€ΠΎΡΡ‚Ρ€Π΅ΠΌΠΈΡ‚Π΅Π»ΡŒΠ½ΠΎΠ΅ ускорСниС Ρ€Π°Π²Π½Ρ‹ΠΌ ΡƒΡΠΊΠΎΡ€Π΅Π½ΠΈΡŽ свободного падСния:

РСшая для радиуса, Π½Π°Ρ…ΠΎΠ΄ΠΈΠΌ

Π—Π½Π°Ρ‡Π΅Π½ΠΈΠ΅

Π§Ρ‚ΠΎΠ±Ρ‹ ΡΠΎΠ·Π΄Π°Ρ‚ΡŒ Ρƒ ΠΏΠΈΠ»ΠΎΡ‚Π° большСС ускорСниС, Ρ‡Π΅ΠΌ g , струС придСтся Π»ΠΈΠ±ΠΎ ΡƒΠΌΠ΅Π½ΡŒΡˆΠΈΡ‚ΡŒ радиус своСй ΠΊΡ€ΡƒΠ³ΠΎΠ²ΠΎΠΉ Ρ‚Ρ€Π°Π΅ΠΊΡ‚ΠΎΡ€ΠΈΠΈ, Π»ΠΈΠ±ΠΎ ΡƒΠ²Π΅Π»ΠΈΡ‡ΠΈΡ‚ΡŒ ΡΠΊΠΎΡ€ΠΎΡΡ‚ΡŒ Π½Π° ΡΡƒΡ‰Π΅ΡΡ‚Π²ΡƒΡŽΡ‰Π΅ΠΉ Ρ‚Ρ€Π°Π΅ΠΊΡ‚ΠΎΡ€ΠΈΠΈ, Π»ΠΈΠ±ΠΎ ΠΈ Ρ‚ΠΎ, ΠΈ Π΄Ρ€ΡƒΠ³ΠΎΠ΅.

ΠŸΡ€ΠΎΠ²Π΅Ρ€ΡŒΡ‚Π΅ своС ΠΏΠΎΠ½ΠΈΠΌΠ°Π½ΠΈΠ΅

Радиус ΠΌΠ°Ρ…ΠΎΠ²ΠΈΠΊΠ° 20,0 см. Какова ΡΠΊΠΎΡ€ΠΎΡΡ‚ΡŒ Ρ‚ΠΎΡ‡ΠΊΠΈ Π½Π° ΠΊΡ€Π°ΡŽ ΠΌΠ°Ρ…ΠΎΠ²ΠΈΠΊΠ°, Ссли ΠΎΠ½Π° испытываСт Ρ†Π΅Π½Ρ‚Ρ€ΠΎΡΡ‚Ρ€Π΅ΠΌΠΈΡ‚Π΅Π»ΡŒΠ½ΠΎΠ΅ ускорСниС

?

[ΠΏΠΎΠΊΠ°Π·Ρ‹Π²Π°Ρ‚ΡŒ-ΠΎΡ‚Π²Π΅Ρ‚ q = ”fs-id116516

09 β€³] ΠŸΠΎΠΊΠ°Π·Π°Ρ‚ΡŒ Ρ€Π΅ΡˆΠ΅Π½ΠΈΠ΅ [/ ΠΏΠΎΠΊΠ°Π·Ρ‹Π²Π°Ρ‚ΡŒ-ΠΎΡ‚Π²Π΅Ρ‚]

[скрытый-ΠΎΡ‚Π²Π΅Ρ‚ a = ”fs-id116516

09 β€³]

134,0 см / с

[/ hidden-answer]

Π¦Π΅Π½Ρ‚Ρ€ΠΎΡΡ‚Ρ€Π΅ΠΌΠΈΡ‚Π΅Π»ΡŒΠ½ΠΎΠ΅ ускорСниС ΠΌΠΎΠΆΠ΅Ρ‚ ΠΈΠΌΠ΅Ρ‚ΡŒ ΡˆΠΈΡ€ΠΎΠΊΠΈΠΉ Π΄ΠΈΠ°ΠΏΠ°Π·ΠΎΠ½ Π·Π½Π°Ρ‡Π΅Π½ΠΈΠΉ Π² зависимости ΠΎΡ‚ скорости ΠΈ радиуса ΠΊΡ€ΠΈΠ²ΠΈΠ·Π½Ρ‹ ΠΊΡ€ΡƒΠ³ΠΎΠ²ΠΎΠΉ Ρ‚Ρ€Π°Π΅ΠΊΡ‚ΠΎΡ€ΠΈΠΈ.Π’ΠΈΠΏΠΈΡ‡Π½Ρ‹Π΅ Ρ†Π΅Π½Ρ‚Ρ€ΠΎΡΡ‚Ρ€Π΅ΠΌΠΈΡ‚Π΅Π»ΡŒΠ½Ρ‹Π΅ ускорСния ΠΏΡ€ΠΈΠ²Π΅Π΄Π΅Π½Ρ‹ Π² ΡΠ»Π΅Π΄ΡƒΡŽΡ‰Π΅ΠΉ Ρ‚Π°Π±Π»ΠΈΡ†Π΅.

Π’ΠΈΠΏΠΈΡ‡Π½ΠΎΠ΅ Ρ†Π΅Π½Ρ‚Ρ€ΠΎΡΡ‚Ρ€Π΅ΠΌΠΈΡ‚Π΅Π»ΡŒΠ½ΠΎΠ΅ ускорСниС
ΠžΠ±ΡŠΠ΅ΠΊΡ‚ Π¦Π΅Π½Ρ‚Ρ€ΠΎΡΡ‚Ρ€Π΅ΠΌΠΈΡ‚Π΅Π»ΡŒΠ½ΠΎΠ΅ ускорСниС (ΠΌ / с 2 ΠΈΠ»ΠΈ коэффициСнт g )
ЗСмля Π²ΠΎΠΊΡ€ΡƒΠ³ Π‘ΠΎΠ»Π½Ρ†Π°

Π›ΡƒΠ½Π° Π²ΠΎΠΊΡ€ΡƒΠ³ Π—Π΅ΠΌΠ»ΠΈ

Π‘ΠΏΡƒΡ‚Π½ΠΈΠΊ Π½Π° гСостационарной ΠΎΡ€Π±ΠΈΡ‚Π΅ 0.233
Π’Π½Π΅ΡˆΠ½ΠΈΠΉ ΠΊΡ€Π°ΠΉ ΠΊΠΎΠΌΠΏΠ°ΠΊΡ‚-диска ΠΏΡ€ΠΈ воспроизвСдСнии

Бтруя Π² Π±ΠΎΡ‡ΠΊΠ΅ Ρ€ΠΎΠ»ΠΈΠΊΠ° (2–3 Π³ )
АмСриканскиС Π³ΠΎΡ€ΠΊΠΈ (5 Π³ )
Π­Π»Π΅ΠΊΡ‚Ρ€ΠΎΠ½, Π²Ρ€Π°Ρ‰Π°ΡŽΡ‰ΠΈΠΉΡΡ Π²ΠΎΠΊΡ€ΡƒΠ³ ΠΏΡ€ΠΎΡ‚ΠΎΠ½Π° Π² простой ΠΌΠΎΠ΄Π΅Π»ΠΈ Π°Ρ‚ΠΎΠΌΠ° Π‘ΠΎΡ€Π°

УравнСния двиТСния для Ρ€Π°Π²Π½ΠΎΠΌΠ΅Ρ€Π½ΠΎΠ³ΠΎ ΠΊΡ€ΡƒΠ³ΠΎΠ²ΠΎΠ³ΠΎ двиТСния

Частица, ΡΠΎΠ²Π΅Ρ€ΡˆΠ°ΡŽΡ‰Π°Ρ ΠΊΡ€ΡƒΠ³ΠΎΠ²ΠΎΠ΅ Π΄Π²ΠΈΠΆΠ΅Π½ΠΈΠ΅, ΠΌΠΎΠΆΠ΅Ρ‚ Π±Ρ‹Ρ‚ΡŒ описана Π΅Π΅ Π²Π΅ΠΊΡ‚ΠΎΡ€ΠΎΠΌ полоТСния

(рисунок) ΠΏΠΎΠΊΠ°Π·Ρ‹Π²Π°Π΅Ρ‚ частицу, ΡΠΎΠ²Π΅Ρ€ΡˆΠ°ΡŽΡ‰ΡƒΡŽ ΠΊΡ€ΡƒΠ³ΠΎΠ²ΠΎΠ΅ Π΄Π²ΠΈΠΆΠ΅Π½ΠΈΠ΅ ΠΏΡ€ΠΎΡ‚ΠΈΠ² часовой стрСлки.Когда частица двиТСтся ΠΏΠΎ окруТности, Π΅Π΅ Π²Π΅ΠΊΡ‚ΠΎΡ€ полоТСния Π²Ρ‹ΠΌΠ΅Ρ‚Π°Π΅Ρ‚ ΡƒΠ³ΠΎΠ»

с осью x- . Π’Π΅ΠΊΡ‚ΠΎΡ€

ΠΎΠ±Ρ€Π°Π·ΡƒΡŽΡ‰ΠΈΠΉ ΡƒΠ³ΠΎΠ»

с осью x- ΠΏΠΎΠΊΠ°Π·Π°Π½ с Π΅Π³ΠΎ ΠΊΠΎΠΌΠΏΠΎΠ½Π΅Π½Ρ‚Π°ΠΌΠΈ вдоль осСй x ΠΈ y . Π’Π΅Π»ΠΈΡ‡ΠΈΠ½Π° Π²Π΅ΠΊΡ‚ΠΎΡ€Π° полоТСния

.

ΠΈ Ρ‚Π°ΠΊΠΆΠ΅ являСтся радиусом ΠΊΡ€ΡƒΠ³Π°, Ρ‚Π°ΠΊ Ρ‡Ρ‚ΠΎ с Ρ‚ΠΎΡ‡ΠΊΠΈ зрСния Π΅Π³ΠΎ ΡΠΎΡΡ‚Π°Π²Π»ΡΡŽΡ‰ΠΈΡ…

Π—Π΄Π΅ΡΡŒ,

– это константа, называСмая ΡƒΠ³Π»ΠΎΠ²ΠΎΠΉ частотой частицы.Угловая частота измСряСтся Π² Ρ€Π°Π΄ΠΈΠ°Π½Π°Ρ… (Ρ€Π°Π΄) Π² сСкунду ΠΈ прСдставляСт собой просто количСство Ρ€Π°Π΄ΠΈΠ°Π½ΠΎΠ² ΡƒΠ³Π»ΠΎΠ²ΠΎΠΉ ΠΌΠ΅Ρ€Ρ‹, Ρ‡Π΅Ρ€Π΅Π· ΠΊΠΎΡ‚ΠΎΡ€ΡƒΡŽ ΠΏΡ€ΠΎΡ…ΠΎΠ΄ΠΈΡ‚ частица Π·Π° сСкунду. Π£Π³ΠΎΠ»

, ΠΊΠΎΡ‚ΠΎΡ€ΠΎΠ΅ Π²Π΅ΠΊΡ‚ΠΎΡ€ полоТСния ΠΈΠΌΠ΅Π΅Ρ‚ Π² любой ΠΊΠΎΠ½ΠΊΡ€Π΅Ρ‚Π½Ρ‹ΠΉ ΠΌΠΎΠΌΠ΅Π½Ρ‚ Π²Ρ€Π΅ΠΌΠ΅Π½ΠΈ, Ρ€Π°Π²Π΅Π½

.

.

Если T – это ΠΏΠ΅Ρ€ΠΈΠΎΠ΄ двиТСния ΠΈΠ»ΠΈ врСмя для Π·Π°Π²Π΅Ρ€ΡˆΠ΅Π½ΠΈΡ ΠΎΠ΄Π½ΠΎΠ³ΠΎ ΠΎΠ±ΠΎΡ€ΠΎΡ‚Π° (

Ρ€Π°Π΄), Π·Π°Ρ‚Π΅ΠΌ

Рис. 4.20. Π’Π΅ΠΊΡ‚ΠΎΡ€ полоТСния частицы, двиТущСйся ΠΏΠΎ ΠΊΡ€ΡƒΠ³Ρƒ, с Π΅Π΅ ΠΊΠΎΠΌΠΏΠΎΠ½Π΅Π½Ρ‚Π°ΠΌΠΈ ΠΏΠΎ осям x ΠΈ y.Частица двиТСтся ΠΏΡ€ΠΎΡ‚ΠΈΠ² часовой стрСлки. Π£Π³ΠΎΠ»

– угловая частота

Π² Ρ€Π°Π΄ΠΈΠ°Π½Π°Ρ… Π² сСкунду, ΡƒΠΌΠ½ΠΎΠΆΠ΅Π½Π½ΠΎΠ΅ Π½Π° t.

Π‘ΠΊΠΎΡ€ΠΎΡΡ‚ΡŒ ΠΈ ускорСниС ΠΌΠΎΠΆΠ½ΠΎ ΠΏΠΎΠ»ΡƒΡ‡ΠΈΡ‚ΡŒ ΠΈΠ· Ρ„ΡƒΠ½ΠΊΡ†ΠΈΠΈ полоТСния ΠΏΡƒΡ‚Π΅ΠΌ диффСрСнцирования:

Из (Рисунок) ΠΌΠΎΠΆΠ½ΠΎ ΠΏΠΎΠΊΠ°Π·Π°Ρ‚ΡŒ, Ρ‡Ρ‚ΠΎ Π²Π΅ΠΊΡ‚ΠΎΡ€ скорости Ρ‚Π°Π½Π³Π΅Π½Ρ†ΠΈΠ°Π»ΡŒΠ½Ρ‹ΠΉ ΠΊ окруТности Π² мСстС нахоТдСния частицы с Π²Π΅Π»ΠΈΡ‡ΠΈΠ½ΠΎΠΉ

.

Аналогично, Π²Π΅ΠΊΡ‚ΠΎΡ€ ускорСния находится ΠΏΡƒΡ‚Π΅ΠΌ диффСрСнцирования скорости:

Из этого уравнСния ΠΌΡ‹ Π²ΠΈΠ΄ΠΈΠΌ, Ρ‡Ρ‚ΠΎ Π²Π΅ΠΊΡ‚ΠΎΡ€ ускорСния ΠΈΠΌΠ΅Π΅Ρ‚ Π²Π΅Π»ΠΈΡ‡ΠΈΠ½Ρƒ

.

ΠΈ Π½Π°ΠΏΡ€Π°Π²Π»Π΅Π½ ΠΏΡ€ΠΎΡ‚ΠΈΠ² Π²Π΅ΠΊΡ‚ΠΎΡ€Π° полоТСния, ΠΊ Π½Π°Ρ‡Π°Π»Ρƒ ΠΊΠΎΠΎΡ€Π΄ΠΈΠ½Π°Ρ‚, ΠΏΠΎΡ‚ΠΎΠΌΡƒ Ρ‡Ρ‚ΠΎ

ΠŸΡ€ΠΈΠΌΠ΅Ρ€

ΠšΡ€ΡƒΠ³ΠΎΠ²ΠΎΠ΅ Π΄Π²ΠΈΠΆΠ΅Π½ΠΈΠ΅ ΠΏΡ€ΠΎΡ‚ΠΎΠ½Π°

ΠŸΡ€ΠΎΡ‚ΠΎΠ½ ΠΈΠΌΠ΅Π΅Ρ‚ ΡΠΊΠΎΡ€ΠΎΡΡ‚ΡŒ

ΠΈ двиТСтся ΠΏΠΎ окруТности Π² плоскости xy радиуса r = 0.175 ΠΌ. Каково Π΅Π³ΠΎ ΠΏΠΎΠ»ΠΎΠΆΠ΅Π½ΠΈΠ΅ Π² плоскости xy Π² ΠΌΠΎΠΌΠ΅Π½Ρ‚ Π²Ρ€Π΅ΠΌΠ΅Π½ΠΈ

?

ΠŸΡ€ΠΈ t = 0 ΠΏΠΎΠ»ΠΎΠΆΠ΅Π½ΠΈΠ΅ ΠΏΡ€ΠΎΡ‚ΠΎΠ½Π°

ΠΈ вращаСтся ΠΏΡ€ΠΎΡ‚ΠΈΠ² часовой стрСлки. НабросайтС Ρ‚Ρ€Π°Π΅ΠΊΡ‚ΠΎΡ€ΠΈΡŽ.

РСшСниС

По ΠΏΡ€ΠΈΠ²Π΅Π΄Π΅Π½Π½Ρ‹ΠΌ Π΄Π°Π½Π½Ρ‹ΠΌ ΠΏΡ€ΠΎΡ‚ΠΎΠ½ ΠΈΠΌΠ΅Π΅Ρ‚ ΠΏΠ΅Ρ€ΠΈΠΎΠ΄ ΠΈ ΡƒΠ³Π»ΠΎΠ²ΡƒΡŽ частоту:

ПолоТСниС частицы Π²

с A = 0.175 ΠΌ –

Из этого Ρ€Π΅Π·ΡƒΠ»ΡŒΡ‚Π°Ρ‚Π° ΠΌΡ‹ Π²ΠΈΠ΄ΠΈΠΌ, Ρ‡Ρ‚ΠΎ ΠΏΡ€ΠΎΡ‚ΠΎΠ½ располоТСн Π½Π΅ΠΌΠ½ΠΎΠ³ΠΎ Π½ΠΈΠΆΠ΅ оси x . Π­Ρ‚ΠΎ ΠΏΠΎΠΊΠ°Π·Π°Π½ΠΎ Π½Π° (Рисунок).

Рис. 4.21 Π’Π΅ΠΊΡ‚ΠΎΡ€ полоТСния ΠΏΡ€ΠΎΡ‚ΠΎΠ½Π° Π½Π°

Показана траСктория ΠΏΡ€ΠΎΡ‚ΠΎΠ½Π°. Π£Π³ΠΎΠ», ΠΏΠΎΠ΄ ΠΊΠΎΡ‚ΠΎΡ€Ρ‹ΠΌ ΠΏΡ€ΠΎΡ‚ΠΎΠ½ двиТСтся ΠΏΠΎ окруТности, составляСт 5,712 Ρ€Π°Π΄, Ρ‡Ρ‚ΠΎ Π½Π΅ΠΌΠ½ΠΎΠ³ΠΎ мСньшС ΠΎΠ΄Π½ΠΎΠ³ΠΎ ΠΏΠΎΠ»Π½ΠΎΠ³ΠΎ ΠΎΠ±ΠΎΡ€ΠΎΡ‚Π°.

Π—Π½Π°Ρ‡Π΅Π½ΠΈΠ΅

ΠœΡ‹ Π²Ρ‹Π±Ρ€Π°Π»ΠΈ Π½Π°Ρ‡Π°Π»ΡŒΠ½ΠΎΠ΅ ΠΏΠΎΠ»ΠΎΠΆΠ΅Π½ΠΈΠ΅ частицы Π½Π° оси x –.Π­Ρ‚ΠΎ Π±Ρ‹Π»ΠΎ ΡΠΎΠ²Π΅Ρ€ΡˆΠ΅Π½Π½ΠΎ ΠΏΡ€ΠΎΠΈΠ·Π²ΠΎΠ»ΡŒΠ½ΠΎ. Если Π±Ρ‹ Π±Ρ‹Π»Π° Π·Π°Π΄Π°Π½Π° другая Π½Π°Ρ‡Π°Π»ΡŒΠ½Π°Ρ позиция, Ρƒ нас Π±Ρ‹Π»ΠΎ Π±Ρ‹ Π΄Ρ€ΡƒΠ³ΠΎΠ΅ ΠΊΠΎΠ½Π΅Ρ‡Π½ΠΎΠ΅ ΠΏΠΎΠ»ΠΎΠΆΠ΅Π½ΠΈΠ΅ ΠΏΡ€ΠΈ t = 200 нс.

НСравномСрноС ΠΊΡ€ΡƒΠ³ΠΎΠ²ΠΎΠ΅ Π΄Π²ΠΈΠΆΠ΅Π½ΠΈΠ΅

ΠšΡ€ΡƒΠ³ΠΎΠ²ΠΎΠ΅ Π΄Π²ΠΈΠΆΠ΅Π½ΠΈΠ΅ Π½Π΅ ΠΎΠ±ΡΠ·Π°Ρ‚Π΅Π»ΡŒΠ½ΠΎ Π΄ΠΎΠ»ΠΆΠ½ΠΎ ΠΈΠΌΠ΅Ρ‚ΡŒ ΠΏΠΎΡΡ‚ΠΎΡΠ½Π½ΡƒΡŽ ΡΠΊΠΎΡ€ΠΎΡΡ‚ΡŒ. Частица ΠΌΠΎΠΆΠ΅Ρ‚ Π΄Π²ΠΈΠ³Π°Ρ‚ΡŒΡΡ ΠΏΠΎ ΠΊΡ€ΡƒΠ³Ρƒ ΠΈ ΡƒΡΠΊΠΎΡ€ΡΡ‚ΡŒΡΡ ΠΈΠ»ΠΈ Π·Π°ΠΌΠ΅Π΄Π»ΡΡ‚ΡŒΡΡ, показывая ускорСниС Π² Π½Π°ΠΏΡ€Π°Π²Π»Π΅Π½ΠΈΠΈ двиТСния.

ΠŸΡ€ΠΈ Ρ€Π°Π²Π½ΠΎΠΌΠ΅Ρ€Π½ΠΎΠΌ ΠΊΡ€ΡƒΠ³ΠΎΠ²ΠΎΠΌ Π΄Π²ΠΈΠΆΠ΅Π½ΠΈΠΈ частица, ΡΠΎΠ²Π΅Ρ€ΡˆΠ°ΡŽΡ‰Π°Ρ ΠΊΡ€ΡƒΠ³ΠΎΠ²ΠΎΠ΅ Π΄Π²ΠΈΠΆΠ΅Π½ΠΈΠ΅, ΠΈΠΌΠ΅Π΅Ρ‚ ΠΏΠΎΡΡ‚ΠΎΡΠ½Π½ΡƒΡŽ ΡΠΊΠΎΡ€ΠΎΡΡ‚ΡŒ, Π° ΠΊΡ€ΡƒΠ³ ΠΈΠΌΠ΅Π΅Ρ‚ фиксированный радиус.Если ΡΠΊΠΎΡ€ΠΎΡΡ‚ΡŒ частицы Ρ‚ΠΎΠΆΠ΅ мСняСтся, Ρ‚ΠΎ ΠΌΡ‹ Π²Π²ΠΎΠ΄ΠΈΠΌ Π΄ΠΎΠΏΠΎΠ»Π½ΠΈΡ‚Π΅Π»ΡŒΠ½ΠΎΠ΅ ускорСниС Π² Π½Π°ΠΏΡ€Π°Π²Π»Π΅Π½ΠΈΠΈ, ΠΊΠ°ΡΠ°Ρ‚Π΅Π»ΡŒΠ½ΠΎΠΌ ΠΊ окруТности. Π’Π°ΠΊΠΎΠ΅ ускорСниС происходит Π² Ρ‚ΠΎΡ‡ΠΊΠ΅ Π½Π° Π²Π΅Ρ€ΡˆΠΈΠ½Π΅, которая измСняСт ΡΠΊΠΎΡ€ΠΎΡΡ‚ΡŒ вращСния, ΠΈΠ»ΠΈ Π² любом ΡƒΡΠΊΠΎΡ€ΡΡŽΡ‰Π΅ΠΌ Ρ€ΠΎΡ‚ΠΎΡ€Π΅. Π’ Ρ€Π°Π±ΠΎΡ‚Π΅ Β«Π’Π΅ΠΊΡ‚ΠΎΡ€Ρ‹ смСщСния ΠΈ скорости» ΠΌΡ‹ ΠΏΠΎΠΊΠ°Π·Π°Π»ΠΈ, Ρ‡Ρ‚ΠΎ Ρ†Π΅Π½Ρ‚Ρ€ΠΎΡΡ‚Ρ€Π΅ΠΌΠΈΡ‚Π΅Π»ΡŒΠ½ΠΎΠ΅ ускорСниС – это ΡΠΊΠΎΡ€ΠΎΡΡ‚ΡŒ измСнСния направлСния Π²Π΅ΠΊΡ‚ΠΎΡ€Π° скорости Π²ΠΎ Π²Ρ€Π΅ΠΌΠ΅Π½ΠΈ. Если ΡΠΊΠΎΡ€ΠΎΡΡ‚ΡŒ частицы измСняСтся, Ρ‚ΠΎ ΠΎΠ½Π° ΠΈΠΌΠ΅Π΅Ρ‚ Ρ‚Π°Π½Π³Π΅Π½Ρ†ΠΈΠ°Π»ΡŒΠ½ΠΎΠ΅ ускорСниС , Ρ‚ΠΎ Π΅ΡΡ‚ΡŒ ΡΠΊΠΎΡ€ΠΎΡΡ‚ΡŒ измСнСния Π²Π΅Π»ΠΈΡ‡ΠΈΠ½Ρ‹ скорости Π²ΠΎ Π²Ρ€Π΅ΠΌΠ΅Π½ΠΈ:

НаправлСниС Ρ‚Π°Π½Π³Π΅Π½Ρ†ΠΈΠ°Π»ΡŒΠ½ΠΎΠ³ΠΎ ускорСния касаСтся окруТности, Ρ‚ΠΎΠ³Π΄Π° ΠΊΠ°ΠΊ Π½Π°ΠΏΡ€Π°Π²Π»Π΅Π½ΠΈΠ΅ Ρ†Π΅Π½Ρ‚Ρ€ΠΎΡΡ‚Ρ€Π΅ΠΌΠΈΡ‚Π΅Π»ΡŒΠ½ΠΎΠ³ΠΎ ускорСния Π½Π°ΠΏΡ€Π°Π²Π»Π΅Π½ΠΎ Ρ€Π°Π΄ΠΈΠ°Π»ΡŒΠ½ΠΎ Π²Π½ΡƒΡ‚Ρ€ΡŒ ΠΊ Ρ†Π΅Π½Ρ‚Ρ€Ρƒ окруТности.Π’Π°ΠΊΠΈΠΌ ΠΎΠ±Ρ€Π°Π·ΠΎΠΌ, частица, двиТущаяся ΠΏΠΎ ΠΊΡ€ΡƒΠ³Ρƒ с Ρ‚Π°Π½Π³Π΅Π½Ρ†ΠΈΠ°Π»ΡŒΠ½Ρ‹ΠΌ ускорСниСм, ΠΈΠΌΠ΅Π΅Ρ‚ ΠΎΠ±Ρ‰Π΅Π΅ ускорСниС , ΠΊΠΎΡ‚ΠΎΡ€ΠΎΠ΅ являСтся Π²Π΅ΠΊΡ‚ΠΎΡ€Π½ΠΎΠΉ суммой Ρ†Π΅Π½Ρ‚Ρ€ΠΎΡΡ‚Ρ€Π΅ΠΌΠΈΡ‚Π΅Π»ΡŒΠ½ΠΎΠ³ΠΎ ΠΈ Ρ‚Π°Π½Π³Π΅Π½Ρ†ΠΈΠ°Π»ΡŒΠ½ΠΎΠ³ΠΎ ускорСний:

Π’Π΅ΠΊΡ‚ΠΎΡ€Ρ‹ ускорСния ΠΏΠΎΠΊΠ°Π·Π°Π½Ρ‹ Π½Π° (Рисунок). ΠžΠ±Ρ€Π°Ρ‚ΠΈΡ‚Π΅ Π²Π½ΠΈΠΌΠ°Π½ΠΈΠ΅, Ρ‡Ρ‚ΠΎ Π΄Π²Π° Π²Π΅ΠΊΡ‚ΠΎΡ€Π° ускорСния

ΠΈ

пСрпСндикулярны Π΄Ρ€ΡƒΠ³ Π΄Ρ€ΡƒΠ³Ρƒ, ΠΏΡ€ΠΈ этом

Π² Ρ€Π°Π΄ΠΈΠ°Π»ΡŒΠ½ΠΎΠΌ Π½Π°ΠΏΡ€Π°Π²Π»Π΅Π½ΠΈΠΈ ΠΈ

Π² Ρ‚Π°Π½Π³Π΅Π½Ρ†ΠΈΠ°Π»ΡŒΠ½ΠΎΠΌ Π½Π°ΠΏΡ€Π°Π²Π»Π΅Π½ΠΈΠΈ.ΠžΠ±Ρ‰Π΅Π΅ ускорСниС

Ρ‚ΠΎΡ‡ΠΊΠΈ ΠΏΠΎΠ΄ ΡƒΠ³Π»ΠΎΠΌ

ΠΈ

Рис. 4.22. Π¦Π΅Π½Ρ‚Ρ€ΠΎΡΡ‚Ρ€Π΅ΠΌΠΈΡ‚Π΅Π»ΡŒΠ½ΠΎΠ΅ ускорСниС ΡƒΠΊΠ°Π·Ρ‹Π²Π°Π΅Ρ‚ Π½Π° Ρ†Π΅Π½Ρ‚Ρ€ ΠΊΡ€ΡƒΠ³Π°. Π’Π°Π½Π³Π΅Π½Ρ†ΠΈΠ°Π»ΡŒΠ½ΠΎΠ΅ ускорСниС являСтся ΠΊΠ°ΡΠ°Ρ‚Π΅Π»ΡŒΠ½Ρ‹ΠΌ ΠΊ окруТности Π² мСстС располоТСния частицы. ΠžΠ±Ρ‰Π΅Π΅ ускорСниС – это вСкторная сумма Ρ‚Π°Π½Π³Π΅Π½Ρ†ΠΈΠ°Π»ΡŒΠ½ΠΎΠ³ΠΎ ΠΈ Ρ†Π΅Π½Ρ‚Ρ€ΠΎΡΡ‚Ρ€Π΅ΠΌΠΈΡ‚Π΅Π»ΡŒΠ½ΠΎΠ³ΠΎ ускорСний, ΠΊΠΎΡ‚ΠΎΡ€Ρ‹Π΅ пСрпСндикулярны.

ΠŸΡ€ΠΈΠΌΠ΅Ρ€

ПолноС ускорСниС ΠΏΡ€ΠΈ ΠΊΡ€ΡƒΠ³ΠΎΠ²ΠΎΠΌ Π΄Π²ΠΈΠΆΠ΅Π½ΠΈΠΈ

Частица двиТСтся ΠΏΠΎ окруТности радиуса r = 2.0 мСс. Π’ ΠΈΠ½Ρ‚Π΅Ρ€Π²Π°Π»Π΅ Π²Ρ€Π΅ΠΌΠ΅Π½ΠΈ ΠΎΡ‚ t = 1,5 с Π΄ΠΎ t = 4,0 с Π΅Π³ΠΎ ΡΠΊΠΎΡ€ΠΎΡΡ‚ΡŒ измСняСтся со Π²Ρ€Π΅ΠΌΠ΅Π½Π΅ΠΌ согласно

.

Каково ΠΏΠΎΠ»Π½ΠΎΠ΅ ускорСниС частицы ΠΏΡ€ΠΈ t = 2,0 с?

БтратСгия

Нам Π΄Π°Π½Ρ‹ ΡΠΊΠΎΡ€ΠΎΡΡ‚ΡŒ частицы ΠΈ радиус ΠΊΡ€ΡƒΠ³Π°, поэтому ΠΌΡ‹ ΠΌΠΎΠΆΠ΅ΠΌ Π»Π΅Π³ΠΊΠΎ Π²Ρ‹Ρ‡ΠΈΡΠ»ΠΈΡ‚ΡŒ Ρ†Π΅Π½Ρ‚Ρ€ΠΎΡΡ‚Ρ€Π΅ΠΌΠΈΡ‚Π΅Π»ΡŒΠ½ΠΎΠ΅ ускорСниС. НаправлСниС Ρ†Π΅Π½Ρ‚Ρ€ΠΎΡΡ‚Ρ€Π΅ΠΌΠΈΡ‚Π΅Π»ΡŒΠ½ΠΎΠ³ΠΎ ускорСния – ΠΊ Ρ†Π΅Π½Ρ‚Ρ€Ρƒ ΠΊΡ€ΡƒΠ³Π°. ΠœΡ‹ Π½Π°Ρ…ΠΎΠ΄ΠΈΠΌ Π²Π΅Π»ΠΈΡ‡ΠΈΠ½Ρƒ Ρ‚Π°Π½Π³Π΅Π½Ρ†ΠΈΠ°Π»ΡŒΠ½ΠΎΠ³ΠΎ ускорСния, взяв ΠΏΡ€ΠΎΠΈΠ·Π²ΠΎΠ΄Π½ΡƒΡŽ ΠΏΠΎ Π²Ρ€Π΅ΠΌΠ΅Π½ΠΈ

.

, ΠΈΡΠΏΠΎΠ»ΡŒΠ·ΡƒΡ (рисунок) ΠΈ ΠΎΡ†Π΅Π½ΠΈΠ² Π΅Π³ΠΎ ΠΊΠ°ΠΊ t = 2.0 с. ΠœΡ‹ ΠΈΡΠΏΠΎΠ»ΡŒΠ·ΡƒΠ΅ΠΌ это ΠΈ Π²Π΅Π»ΠΈΡ‡ΠΈΠ½Ρƒ Ρ†Π΅Π½Ρ‚Ρ€ΠΎΡΡ‚Ρ€Π΅ΠΌΠΈΡ‚Π΅Π»ΡŒΠ½ΠΎΠ³ΠΎ ускорСния, Ρ‡Ρ‚ΠΎΠ±Ρ‹ Π½Π°ΠΉΡ‚ΠΈ ΠΏΠΎΠ»Π½ΠΎΠ΅ ускорСниС.

РСшСниС

Π¦Π΅Π½Ρ‚Ρ€ΠΎΡΡ‚Ρ€Π΅ΠΌΠΈΡ‚Π΅Π»ΡŒΠ½ΠΎΠ΅ ускорСниС

Π½Π°ΠΏΡ€Π°Π²Π»Π΅Π½ ΠΊ Ρ†Π΅Π½Ρ‚Ρ€Ρƒ ΠΊΡ€ΡƒΠ³Π°. ΠšΠ°ΡΠ°Ρ‚Π΅Π»ΡŒΠ½ΠΎΠ΅ ускорСниС

Π‘ΡƒΠΌΠΌΠ°Ρ€Π½ΠΎΠ΅ ускорСниС

ΠΈ

ΠΎΡ‚ ΠΊΠ°ΡΠ°Ρ‚Π΅Π»ΡŒΠ½ΠΎΠΉ ΠΊ окруТности. Π‘ΠΌ. (Рисунок).

Рис. 4.23 Π’Π΅ΠΊΡ‚ΠΎΡ€Ρ‹ Ρ‚Π°Π½Π³Π΅Π½Ρ†ΠΈΠ°Π»ΡŒΠ½ΠΎΠ³ΠΎ ΠΈ Ρ†Π΅Π½Ρ‚Ρ€ΠΎΡΡ‚Ρ€Π΅ΠΌΠΈΡ‚Π΅Π»ΡŒΠ½ΠΎΠ³ΠΎ ускорСния.ЧистоС ускорСниС

– это вСкторная сумма Π΄Π²ΡƒΡ… ускорСний.

Π—Π½Π°Ρ‡Π΅Π½ΠΈΠ΅

НаправлСния Ρ†Π΅Π½Ρ‚Ρ€ΠΎΡΡ‚Ρ€Π΅ΠΌΠΈΡ‚Π΅Π»ΡŒΠ½ΠΎΠ³ΠΎ ΠΈ Ρ‚Π°Π½Π³Π΅Π½Ρ†ΠΈΠ°Π»ΡŒΠ½ΠΎΠ³ΠΎ ускорСний ΠΌΠΎΠΆΠ½ΠΎ ΠΎΠΏΠΈΡΠ°Ρ‚ΡŒ Π±ΠΎΠ»Π΅Π΅ ΡƒΠ΄ΠΎΠ±Π½ΠΎ Π² Ρ‚Π΅Ρ€ΠΌΠΈΠ½Π°Ρ… полярной систСмы ΠΊΠΎΠΎΡ€Π΄ΠΈΠ½Π°Ρ‚ с Π΅Π΄ΠΈΠ½ΠΈΡ‡Π½Ρ‹ΠΌΠΈ Π²Π΅ΠΊΡ‚ΠΎΡ€Π°ΠΌΠΈ Π² Ρ€Π°Π΄ΠΈΠ°Π»ΡŒΠ½ΠΎΠΌ ΠΈ Ρ‚Π°Π½Π³Π΅Π½Ρ†ΠΈΠ°Π»ΡŒΠ½ΠΎΠΌ направлСниях. Π­Ρ‚Π° систСма ΠΊΠΎΠΎΡ€Π΄ΠΈΠ½Π°Ρ‚, которая ΠΈΡΠΏΠΎΠ»ΡŒΠ·ΡƒΠ΅Ρ‚ΡΡ для двиТСния ΠΏΠΎ ΠΊΡ€ΠΈΠ²ΠΎΠ»ΠΈΠ½Π΅ΠΉΠ½Ρ‹ΠΌ траСкториям, ΠΏΠΎΠ΄Ρ€ΠΎΠ±Π½ΠΎ обсуТдаСтся Π΄Π°Π»Π΅Π΅ Π² ΠΊΠ½ΠΈΠ³Π΅.

Π‘Π²ΠΎΠ΄ΠΊΠ°

  • Π Π°Π²Π½ΠΎΠΌΠ΅Ρ€Π½ΠΎΠ΅ ΠΊΡ€ΡƒΠ³ΠΎΠ²ΠΎΠ΅ Π΄Π²ΠΈΠΆΠ΅Π½ΠΈΠ΅ – это Π΄Π²ΠΈΠΆΠ΅Π½ΠΈΠ΅ ΠΏΠΎ окруТности с постоянной ΡΠΊΠΎΡ€ΠΎΡΡ‚ΡŒΡŽ.
  • Π¦Π΅Π½Ρ‚Ρ€ΠΎΡΡ‚Ρ€Π΅ΠΌΠΈΡ‚Π΅Π»ΡŒΠ½ΠΎΠ΅ ускорСниС

    – это ускорСниС, ΠΊΠΎΡ‚ΠΎΡ€ΠΎΠ΅ Π΄ΠΎΠ»ΠΆΠ½ΠΎ ΠΈΠΌΠ΅Ρ‚ΡŒ частица, Ρ‡Ρ‚ΠΎΠ±Ρ‹ Π΄Π²ΠΈΠ³Π°Ρ‚ΡŒΡΡ ΠΏΠΎ ΠΊΡ€ΡƒΠ³ΠΎΠ²ΠΎΠΉ Ρ‚Ρ€Π°Π΅ΠΊΡ‚ΠΎΡ€ΠΈΠΈ. Π¦Π΅Π½Ρ‚Ρ€ΠΎΡΡ‚Ρ€Π΅ΠΌΠΈΡ‚Π΅Π»ΡŒΠ½ΠΎΠ΅ ускорСниС всСгда Π½Π°ΠΏΡ€Π°Π²Π»Π΅Π½ΠΎ ΠΊ Ρ†Π΅Π½Ρ‚Ρ€Ρƒ вращСния ΠΈ ΠΈΠΌΠ΅Π΅Ρ‚ Π²Π΅Π»ΠΈΡ‡ΠΈΠ½Ρƒ

    .

  • НСравномСрноС ΠΊΡ€ΡƒΠ³ΠΎΠ²ΠΎΠ΅ Π΄Π²ΠΈΠΆΠ΅Π½ΠΈΠ΅ Π²ΠΎΠ·Π½ΠΈΠΊΠ°Π΅Ρ‚, ΠΊΠΎΠ³Π΄Π° сущСствуСт Ρ‚Π°Π½Π³Π΅Π½Ρ†ΠΈΠ°Π»ΡŒΠ½ΠΎΠ΅ ускорСниС ΠΎΠ±ΡŠΠ΅ΠΊΡ‚Π°, Π²Ρ‹ΠΏΠΎΠ»Π½ΡΡŽΡ‰Π΅Π³ΠΎ ΠΊΡ€ΡƒΠ³ΠΎΠ²ΠΎΠ΅ Π΄Π²ΠΈΠΆΠ΅Π½ΠΈΠ΅, Ρ‚Π°ΠΊ Ρ‡Ρ‚ΠΎ ΡΠΊΠΎΡ€ΠΎΡΡ‚ΡŒ ΠΎΠ±ΡŠΠ΅ΠΊΡ‚Π° измСняСтся. Π­Ρ‚ΠΎ ускорСниС называСтся Ρ‚Π°Π½Π³Π΅Π½Ρ†ΠΈΠ°Π»ΡŒΠ½Ρ‹ΠΌ ускорСниСм.

    Π’Π΅Π»ΠΈΡ‡ΠΈΠ½Π° Ρ‚Π°Π½Π³Π΅Π½Ρ†ΠΈΠ°Π»ΡŒΠ½ΠΎΠ³ΠΎ ускорСния – это ΡΠΊΠΎΡ€ΠΎΡΡ‚ΡŒ измСнСния Π²Π΅Π»ΠΈΡ‡ΠΈΠ½Ρ‹ скорости Π²ΠΎ Π²Ρ€Π΅ΠΌΠ΅Π½ΠΈ.Π’Π΅ΠΊΡ‚ΠΎΡ€ Ρ‚Π°Π½Π³Π΅Π½Ρ†ΠΈΠ°Π»ΡŒΠ½ΠΎΠ³ΠΎ ускорСния касаСтся окруТности, Ρ‚ΠΎΠ³Π΄Π° ΠΊΠ°ΠΊ Π²Π΅ΠΊΡ‚ΠΎΡ€ Ρ†Π΅Π½Ρ‚Ρ€ΠΎΡΡ‚Ρ€Π΅ΠΌΠΈΡ‚Π΅Π»ΡŒΠ½ΠΎΠ³ΠΎ ускорСния Π½Π°ΠΏΡ€Π°Π²Π»Π΅Π½ Ρ€Π°Π΄ΠΈΠ°Π»ΡŒΠ½ΠΎ Π²Π½ΡƒΡ‚Ρ€ΡŒ ΠΊ Ρ†Π΅Π½Ρ‚Ρ€Ρƒ окруТности. ΠžΠ±Ρ‰Π΅Π΅ ускорСниС – это вСкторная сумма Ρ‚Π°Π½Π³Π΅Π½Ρ†ΠΈΠ°Π»ΡŒΠ½ΠΎΠ³ΠΎ ΠΈ Ρ†Π΅Π½Ρ‚Ρ€ΠΎΡΡ‚Ρ€Π΅ΠΌΠΈΡ‚Π΅Π»ΡŒΠ½ΠΎΠ³ΠΎ ускорСний.

  • ΠžΠ±ΡŠΠ΅ΠΊΡ‚, Π²Ρ‹ΠΏΠΎΠ»Π½ΡΡŽΡ‰ΠΈΠΉ Ρ€Π°Π²Π½ΠΎΠΌΠ΅Ρ€Π½ΠΎΠ΅ ΠΊΡ€ΡƒΠ³ΠΎΠ²ΠΎΠ΅ Π΄Π²ΠΈΠΆΠ΅Π½ΠΈΠ΅, ΠΌΠΎΠΆΠ½ΠΎ ΠΎΠΏΠΈΡΠ°Ρ‚ΡŒ уравнСниями двиТСния. Π’Π΅ΠΊΡ‚ΠΎΡ€ полоТСния ΠΎΠ±ΡŠΠ΅ΠΊΡ‚Π°

    , Π³Π΄Π΅ A – Π²Π΅Π»ΠΈΡ‡ΠΈΠ½Π°

    .

    , ΠΊΠΎΡ‚ΠΎΡ€Ρ‹ΠΉ Ρ‚Π°ΠΊΠΆΠ΅ являСтся радиусом ΠΊΡ€ΡƒΠ³Π°, ΠΈ

    – угловая частота.

ΠšΠΎΠ½Ρ†Π΅ΠΏΡ‚ΡƒΠ°Π»ΡŒΠ½Ρ‹Π΅ вопросы

ΠœΠΎΠΆΠ΅Ρ‚ Π»ΠΈ Ρ†Π΅Π½Ρ‚Ρ€ΠΎΡΡ‚Ρ€Π΅ΠΌΠΈΡ‚Π΅Π»ΡŒΠ½ΠΎΠ΅ ускорСниС ΠΈΠ·ΠΌΠ΅Π½ΠΈΡ‚ΡŒ ΡΠΊΠΎΡ€ΠΎΡΡ‚ΡŒ частицы, ΡΠΎΠ²Π΅Ρ€ΡˆΠ°ΡŽΡ‰Π΅ΠΉ ΠΊΡ€ΡƒΠ³ΠΎΠ²ΠΎΠ΅ Π΄Π²ΠΈΠΆΠ΅Π½ΠΈΠ΅?

ΠœΠΎΠΆΠ΅Ρ‚ Π»ΠΈ Ρ‚Π°Π½Π³Π΅Π½Ρ†ΠΈΠ°Π»ΡŒΠ½ΠΎΠ΅ ускорСниС ΠΈΠ·ΠΌΠ΅Π½ΠΈΡ‚ΡŒ ΡΠΊΠΎΡ€ΠΎΡΡ‚ΡŒ частицы, ΡΠΎΠ²Π΅Ρ€ΡˆΠ°ΡŽΡ‰Π΅ΠΉ ΠΊΡ€ΡƒΠ³ΠΎΠ²ΠΎΠ΅ Π΄Π²ΠΈΠΆΠ΅Π½ΠΈΠ΅?

[show-answer q = ”fs-id11651623 β€³] ΠŸΠΎΠΊΠ°Π·Π°Ρ‚ΡŒ Ρ€Π΅ΡˆΠ΅Π½ΠΈΠ΅ [/ show-answer]

[скрытый-ΠΎΡ‚Π²Π΅Ρ‚ a = ”fs-id11651623 β€³]

Π΄Π°

[/ hidden-answer]

ΠŸΡ€ΠΎΠ±Π»Π΅ΠΌΡ‹

ΠœΠ°Ρ…ΠΎΠ²ΠΈΠΊ вращаСтся со ΡΠΊΠΎΡ€ΠΎΡΡ‚ΡŒΡŽ 30 ΠΎΠ± / с.Каков ΠΏΠΎΠ»Π½Ρ‹ΠΉ ΡƒΠ³ΠΎΠ» Π² Ρ€Π°Π΄ΠΈΠ°Π½Π°Ρ…, Π½Π° ΠΊΠΎΡ‚ΠΎΡ€Ρ‹ΠΉ Ρ‚ΠΎΡ‡ΠΊΠ° Π½Π° ΠΌΠ°Ρ…ΠΎΠ²ΠΈΠΊΠ΅ поворачиваСтся Π·Π° 40 с?

Частица двиТСтся ΠΏΠΎ ΠΊΡ€ΡƒΠ³Ρƒ радиусом 10 ΠΌ с постоянной ΡΠΊΠΎΡ€ΠΎΡΡ‚ΡŒΡŽ 20 ΠΌ / с. Какая Π²Π΅Π»ΠΈΡ‡ΠΈΠ½Π° ускорСния?

[ΠΏΠΎΠΊΠ°Π·Ρ‹Π²Π°Ρ‚ΡŒ-ΠΎΡ‚Π²Π΅Ρ‚ q = ”fs-id11651685 β€³] ΠŸΠΎΠΊΠ°Π·Π°Ρ‚ΡŒ Ρ€Π΅ΡˆΠ΅Π½ΠΈΠ΅ [/ ΠΏΠΎΠΊΠ°Π·Ρ‹Π²Π°Ρ‚ΡŒ-ΠΎΡ‚Π²Π΅Ρ‚]

[скрытый-ΠΎΡ‚Π²Π΅Ρ‚ a = ”fs-id11651685 β€³]

[/ hidden-answer]

Кэм ΠΡŒΡŽΡ‚ΠΎΠ½ ΠΈΠ· Carolina Panthers бросаСт ΠΈΠ΄Π΅Π°Π»ΡŒΠ½ΡƒΡŽ Ρ„ΡƒΡ‚Π±ΠΎΠ»ΡŒΠ½ΡƒΡŽ ΡΠΏΠΈΡ€Π°Π»ΡŒ Π½Π° 8.0 ΠΎΠ± / с. Радиус ΠΏΡ€ΠΎΡ„Π΅ΡΡΠΈΠΎΠ½Π°Π»ΡŒΠ½ΠΎΠ³ΠΎ Ρ„ΡƒΡ‚Π±ΠΎΠ»Π° составляСт 8,5 см ΠΏΠΎ сСрСдинС ΠΊΠΎΡ€ΠΎΡ‚ΠΊΠΎΠΉ стороны. Π§Ρ‚ΠΎ Ρ‚Π°ΠΊΠΎΠ΅ Ρ†Π΅Π½Ρ‚Ρ€ΠΎΡΡ‚Ρ€Π΅ΠΌΠΈΡ‚Π΅Π»ΡŒΠ½ΠΎΠ΅ ускорСниС ΡˆΠ½ΡƒΡ€ΠΊΠΎΠ² Π½Π° Ρ„ΡƒΡ‚Π±ΠΎΠ»ΡŒΠ½ΠΎΠΌ мячС?

Аттракцион ярмарки Π²Ρ€Π°Ρ‰Π°Π΅Ρ‚ своих пассаТиров Π²Π½ΡƒΡ‚Ρ€ΠΈ ΠΊΠΎΠ½Ρ‚Π΅ΠΉΠ½Π΅Ρ€Π° Π² Ρ„ΠΎΡ€ΠΌΠ΅ Π»Π΅Ρ‚Π°ΡŽΡ‰Π΅ΠΉ Ρ‚Π°Ρ€Π΅Π»ΠΊΠΈ. Если Π³ΠΎΡ€ΠΈΠ·ΠΎΠ½Ρ‚Π°Π»ΡŒΠ½Ρ‹ΠΉ ΠΊΡ€ΡƒΠ³ΠΎΠ²ΠΎΠΉ ΠΏΡƒΡ‚ΡŒ, ΠΏΠΎ ΠΊΠΎΡ‚ΠΎΡ€ΠΎΠΌΡƒ ΡΠ»Π΅Π΄ΡƒΡŽΡ‚ Π³ΠΎΠ½Ρ‰ΠΈΠΊΠΈ, ΠΈΠΌΠ΅Π΅Ρ‚ радиус 8,00 ΠΌ, ΠΏΡ€ΠΈ ΡΠΊΠΎΠ»ΡŒΠΊΠΈΡ… ΠΎΠ±ΠΎΡ€ΠΎΡ‚Π°Ρ… Π² ΠΌΠΈΠ½ΡƒΡ‚Ρƒ Π³ΠΎΠ½Ρ‰ΠΈΠΊΠΈ ΠΏΠΎΠ΄Π²Π΅Ρ€Π³Π°ΡŽΡ‚ΡΡ Ρ†Π΅Π½Ρ‚Ρ€ΠΎΡΡ‚Ρ€Π΅ΠΌΠΈΡ‚Π΅Π»ΡŒΠ½ΠΎΠΌΡƒ ΡƒΡΠΊΠΎΡ€Π΅Π½ΠΈΡŽ, Ρ€Π°Π²Π½ΠΎΠΌΡƒ ΡƒΡΠΊΠΎΡ€Π΅Π½ΠΈΡŽ силы тяТСсти?

[ΠΏΠΎΠΊΠ°Π·Ρ‹Π²Π°Ρ‚ΡŒ-ΠΎΡ‚Π²Π΅Ρ‚ q = ”fs-id11651661 β€³] ΠŸΠΎΠΊΠ°Π·Π°Ρ‚ΡŒ Ρ€Π΅ΡˆΠ΅Π½ΠΈΠ΅ [/ ΠΏΠΎΠΊΠ°Π·Ρ‹Π²Π°Ρ‚ΡŒ-ΠΎΡ‚Π²Π΅Ρ‚]

[скрытый-ΠΎΡ‚Π²Π΅Ρ‚ a = ”fs-id11651661 β€³]

, Ρ‡Ρ‚ΠΎ Ρ€Π°Π²Π½ΠΎ

[/ hidden-answer]

Π‘Π΅Π³ΡƒΠ½, ΡƒΡ‡Π°ΡΡ‚Π²ΡƒΡŽΡ‰ΠΈΠΉ Π² Π·Π°Π±Π΅Π³Π΅ Π½Π° 200 ΠΌ, Π΄ΠΎΠ»ΠΆΠ΅Π½ ΠΎΠ±ΠΎΠΉΡ‚ΠΈ ΠΊΠΎΠ½Π΅Ρ† трассы, ΠΈΠΌΠ΅ΡŽΡ‰Π΅ΠΉ Π΄ΡƒΠ³Ρƒ окруТности с радиусом ΠΊΡ€ΠΈΠ²ΠΈΠ·Π½Ρ‹ 30.0 мСс. Π‘Π΅Π³ΡƒΠ½ Π½Π°Ρ‡ΠΈΠ½Π°Π΅Ρ‚ Π·Π°Π±Π΅Π³ с постоянной ΡΠΊΠΎΡ€ΠΎΡΡ‚ΡŒΡŽ. Если ΠΎΠ½Π° ΠΏΡ€Π΅ΠΎΠ΄ΠΎΠ»Π΅Π²Π°Π΅Ρ‚ 200-ΠΌΠ΅Ρ‚Ρ€ΠΎΠ²Ρ‹ΠΉ Ρ€Ρ‹Π²ΠΎΠΊ Π·Π° 23,2 с ΠΈ Π±Π΅ΠΆΠΈΡ‚ с постоянной ΡΠΊΠΎΡ€ΠΎΡΡ‚ΡŒΡŽ Π½Π° протяТСнии всСй Π³ΠΎΠ½ΠΊΠΈ, ΠΊΠ°ΠΊΠΎΠ²ΠΎ Π΅Π΅ Ρ†Π΅Π½Ρ‚Ρ€ΠΎΡΡ‚Ρ€Π΅ΠΌΠΈΡ‚Π΅Π»ΡŒΠ½ΠΎΠ΅ ускорСниС ΠΏΡ€ΠΈ ΠΏΡ€ΠΎΡ…ΠΎΠΆΠ΄Π΅Π½ΠΈΠΈ ΠΊΡ€ΠΈΠ²ΠΎΠ»ΠΈΠ½Π΅ΠΉΠ½ΠΎΠΉ части трассы?

Каково ускорСниС Π’Π΅Π½Π΅Ρ€Ρ‹ ΠΏΠΎ Π½Π°ΠΏΡ€Π°Π²Π»Π΅Π½ΠΈΡŽ ΠΊ Π‘ΠΎΠ»Π½Ρ†Ρƒ, Ссли ΠΏΡ€ΠΈΠ½ΡΡ‚ΡŒ ΠΊΡ€ΡƒΠ³ΠΎΠ²ΡƒΡŽ ΠΎΡ€Π±ΠΈΡ‚Ρƒ?

[ΠΏΠΎΠΊΠ°Π·Ρ‹Π²Π°Ρ‚ΡŒ-ΠΎΡ‚Π²Π΅Ρ‚ q = ”fs-id116516

11 β€³] ΠŸΠΎΠΊΠ°Π·Π°Ρ‚ΡŒ Ρ€Π΅ΡˆΠ΅Π½ΠΈΠ΅ [/ ΠΏΠΎΠΊΠ°Π·Ρ‹Π²Π°Ρ‚ΡŒ-ΠΎΡ‚Π²Π΅Ρ‚]

[скрытый-ΠΎΡ‚Π²Π΅Ρ‚ a = ”fs-id116516

11 β€³]

Π’Π΅Π½Π΅Ρ€Π° находится Π½Π° расстоянии 108,2 ΠΌΠΈΠ»Π»ΠΈΠΎΠ½Π° ΠΊΠΌ ΠΎΡ‚ Π‘ΠΎΠ»Π½Ρ†Π° ΠΈ ΠΈΠΌΠ΅Π΅Ρ‚ ΠΏΠ΅Ρ€ΠΈΠΎΠ΄ обращСния 0.6152 Π³.

[/ hidden-answer]

Π­ΠΊΡΠΏΠ΅Ρ€ΠΈΠΌΠ΅Π½Ρ‚Π°Π»ΡŒΠ½Π°Ρ рСактивная Ρ€Π°ΠΊΠ΅Ρ‚Π° двиТСтся Π²ΠΎΠΊΡ€ΡƒΠ³ Π—Π΅ΠΌΠ»ΠΈ вдоль экватора прямо Π½Π°Π΄ Π΅Π΅ ΠΏΠΎΠ²Π΅Ρ€Ρ…Π½ΠΎΡΡ‚ΡŒΡŽ. Π‘ ΠΊΠ°ΠΊΠΎΠΉ ΡΠΊΠΎΡ€ΠΎΡΡ‚ΡŒΡŽ Π΄ΠΎΠ»ΠΆΠ½Π° Π΄Π²ΠΈΠ³Π°Ρ‚ΡŒΡΡ струя, Ссли Π²Π΅Π»ΠΈΡ‡ΠΈΠ½Π° Π΅Π΅ ускорСния составляСт g ?

ВСнтилятор вращаСтся с постоянной ΡΠΊΠΎΡ€ΠΎΡΡ‚ΡŒΡŽ 360,0 ΠΎΠ± / ΠΌΠΈΠ½. Какова Π²Π΅Π»ΠΈΡ‡ΠΈΠ½Π° ускорСния Ρ‚ΠΎΡ‡ΠΊΠΈ Π½Π° ΠΎΠ΄Π½ΠΎΠΉ ΠΈΠ· лопастСй Π½Π° расстоянии 10,0 см ΠΎΡ‚ оси вращСния?

[ΠΏΠΎΠΊΠ°Π·Ρ‹Π²Π°Ρ‚ΡŒ-ΠΎΡ‚Π²Π΅Ρ‚ q = ”fs-id1165168934870 β€³] ΠŸΠΎΠΊΠ°Π·Π°Ρ‚ΡŒ Ρ€Π΅ΡˆΠ΅Π½ΠΈΠ΅ [/ ΠΏΠΎΠΊΠ°Π·Ρ‹Π²Π°Ρ‚ΡŒ-ΠΎΡ‚Π²Π΅Ρ‚]

[скрытый-ΠΎΡ‚Π²Π΅Ρ‚ a = ”fs-id1165168934870 β€³]

[/ hidden-answer]

Π’ΠΎΡ‡ΠΊΠ°, располоТСнная Π½Π° сСкундной стрСлкС Π±ΠΎΠ»ΡŒΡˆΠΈΡ… часов, ΠΈΠΌΠ΅Π΅Ρ‚ Ρ€Π°Π΄ΠΈΠ°Π»ΡŒΠ½ΠΎΠ΅ ускорСниС

.

Как Π΄Π°Π»Π΅ΠΊΠΎ находится Ρ‚ΠΎΡ‡ΠΊΠ° ΠΎΡ‚ оси вращСния сСкундной стрСлки?

Глоссарий

угловая частота

ΡΠΊΠΎΡ€ΠΎΡΡ‚ΡŒ измСнСния ΡƒΠ³Π»Π°, с ΠΊΠΎΡ‚ΠΎΡ€Ρ‹ΠΌ ΠΎΠ±ΡŠΠ΅ΠΊΡ‚ двиТСтся ΠΏΠΎ ΠΊΡ€ΡƒΠ³ΠΎΠ²ΠΎΠΉ Ρ‚Ρ€Π°Π΅ΠΊΡ‚ΠΎΡ€ΠΈΠΈ

Ρ†Π΅Π½Ρ‚Ρ€ΠΎΡΡ‚Ρ€Π΅ΠΌΠΈΡ‚Π΅Π»ΡŒΠ½ΠΎΠ΅ ускорСниС
ΠšΠΎΠΌΠΏΠΎΠ½Π΅Π½Ρ‚ ускорСния ΠΎΠ±ΡŠΠ΅ΠΊΡ‚Π°, двиТущСгося ΠΏΠΎ окруТности, Ρ€Π°Π΄ΠΈΠ°Π»ΡŒΠ½ΠΎ Π½Π°ΠΏΡ€Π°Π²Π»Π΅Π½Π½ΠΎΠΉ Π²Π½ΡƒΡ‚Ρ€ΡŒ ΠΊ Ρ†Π΅Π½Ρ‚Ρ€Ρƒ окруТности
Ρ‚Π°Π½Π³Π΅Π½Ρ†ΠΈΠ°Π»ΡŒΠ½ΠΎΠ΅ ускорСниС
Π’Π΅Π»ΠΈΡ‡ΠΈΠ½Π°
– это врСмСнная ΡΠΊΠΎΡ€ΠΎΡΡ‚ΡŒ измСнСния скорости.Π•Π³ΠΎ Π½Π°ΠΏΡ€Π°Π²Π»Π΅Π½ΠΈΠ΅ касаСтся окруТности.
ΠΎΠ±Ρ‰Π΅Π΅ ускорСниС
вСкторная сумма Ρ†Π΅Π½Ρ‚Ρ€ΠΎΡΡ‚Ρ€Π΅ΠΌΠΈΡ‚Π΅Π»ΡŒΠ½ΠΎΠ³ΠΎ ΠΈ Ρ‚Π°Π½Π³Π΅Π½Ρ†ΠΈΠ°Π»ΡŒΠ½ΠΎΠ³ΠΎ ускорСний

Π¦Π΅Π½Ρ‚Ρ€ΠΎΡΡ‚Ρ€Π΅ΠΌΠΈΡ‚Π΅Π»ΡŒΠ½Π°Ρ сила ΠΈ ускорСниС – AP Physics 1

Если Π²Ρ‹ считаСтС, Ρ‡Ρ‚ΠΎ ΠΊΠΎΠ½Ρ‚Π΅Π½Ρ‚, доступный Ρ‡Π΅Ρ€Π΅Π· Π’Π΅Π±-сайт (ΠΊΠ°ΠΊ ΠΎΠΏΡ€Π΅Π΄Π΅Π»Π΅Π½ΠΎ Π² Π½Π°ΡˆΠΈΡ… Условиях обслуТивания), Π½Π°Ρ€ΡƒΡˆΠ°Π΅Ρ‚ ΠΈΠ»ΠΈ Π΄Ρ€ΡƒΠ³ΠΈΠ΅ ваши авторскиС ΠΏΡ€Π°Π²Π°, сообщитС Π½Π°ΠΌ, ΠΎΡ‚ΠΏΡ€Π°Π²ΠΈΠ² письмСнноС ΡƒΠ²Π΅Π΄ΠΎΠΌΠ»Π΅Π½ΠΈΠ΅ (Β«Π£Π²Π΅Π΄ΠΎΠΌΠ»Π΅Π½ΠΈΠ΅ ΠΎ Π½Π°Ρ€ΡƒΡˆΠ΅Π½ΠΈΠΈΒ»), содСрТащСС Π² ΠΈΠ½Ρ„ΠΎΡ€ΠΌΠ°Ρ†ΠΈΡŽ, ΠΎΠΏΠΈΡΠ°Π½Π½ΡƒΡŽ Π½ΠΈΠΆΠ΅, Π½Π°Π·Π½Π°Ρ‡Π΅Π½Π½ΠΎΠΌΡƒ Π½ΠΈΠΆΠ΅ Π°Π³Π΅Π½Ρ‚Ρƒ.Если Ρ€Π΅ΠΏΠ΅Ρ‚ΠΈΡ‚ΠΎΡ€Ρ‹ унивСрситСта ΠΏΡ€Π΅Π΄ΠΏΡ€ΠΈΠΌΡƒΡ‚ дСйствия Π² ΠΎΡ‚Π²Π΅Ρ‚ Π½Π° Π°Π½ Π£Π²Π΅Π΄ΠΎΠΌΠ»Π΅Π½ΠΈΠ΅ ΠΎ Π½Π°Ρ€ΡƒΡˆΠ΅Π½ΠΈΠΈ, ΠΎΠ½ΠΎ ΠΏΡ€Π΅Π΄ΠΏΡ€ΠΈΠΌΠ΅Ρ‚ Π΄ΠΎΠ±Ρ€ΠΎΡΠΎΠ²Π΅ΡΡ‚Π½ΡƒΡŽ ΠΏΠΎΠΏΡ‹Ρ‚ΠΊΡƒ ΡΠ²ΡΠ·Π°Ρ‚ΡŒΡΡ со стороной, которая прСдоставила Ρ‚Π°ΠΊΠΎΠΉ ΠΊΠΎΠ½Ρ‚Π΅Π½Ρ‚ срСдствами самого послСднСго адрСса элСктронной ΠΏΠΎΡ‡Ρ‚Ρ‹, Ссли Ρ‚Π°ΠΊΠΎΠ²ΠΎΠΉ имССтся, прСдоставлСнного Ρ‚Π°ΠΊΠΎΠΉ стороной Varsity Tutors.

Π’Π°ΡˆΠ΅ Π£Π²Π΅Π΄ΠΎΠΌΠ»Π΅Π½ΠΈΠ΅ ΠΎ Π½Π°Ρ€ΡƒΡˆΠ΅Π½ΠΈΠΈ ΠΏΡ€Π°Π² ΠΌΠΎΠΆΠ΅Ρ‚ Π±Ρ‹Ρ‚ΡŒ ΠΎΡ‚ΠΏΡ€Π°Π²Π»Π΅Π½ΠΎ сторонС, ΠΏΡ€Π΅Π΄ΠΎΡΡ‚Π°Π²ΠΈΠ²ΡˆΠ΅ΠΉ доступ ΠΊ ΠΊΠΎΠ½Ρ‚Π΅Π½Ρ‚Ρƒ, ΠΈΠ»ΠΈ Ρ‚Ρ€Π΅Ρ‚ΡŒΠΈΠΌ Π»ΠΈΡ†Π°ΠΌ, Ρ‚Π°ΠΊΠΈΠΌ ΠΊΠ°ΠΊ Π² качСствС ChillingEffects.org.

ΠžΠ±Ρ€Π°Ρ‚ΠΈΡ‚Π΅ Π²Π½ΠΈΠΌΠ°Π½ΠΈΠ΅, Ρ‡Ρ‚ΠΎ Π²Ρ‹ Π±ΡƒΠ΄Π΅Ρ‚Π΅ нСсти ΠΎΡ‚Π²Π΅Ρ‚ΡΡ‚Π²Π΅Π½Π½ΠΎΡΡ‚ΡŒ Π·Π° ΡƒΡ‰Π΅Ρ€Π± (Π²ΠΊΠ»ΡŽΡ‡Π°Ρ расходы ΠΈ Π³ΠΎΠ½ΠΎΡ€Π°Ρ€Ρ‹ Π°Π΄Π²ΠΎΠΊΠ°Ρ‚Π°ΠΌ), Ссли Π²Ρ‹ сущСствСнно ΠΈΡΠΊΠ°ΠΆΠ°Ρ‚ΡŒ ΠΈΠ½Ρ„ΠΎΡ€ΠΌΠ°Ρ†ΠΈΡŽ ΠΎ Ρ‚ΠΎΠΌ, Ρ‡Ρ‚ΠΎ ΠΏΡ€ΠΎΠ΄ΡƒΠΊΡ‚ ΠΈΠ»ΠΈ дСйствиС Π½Π°Ρ€ΡƒΡˆΠ°Π΅Ρ‚ ваши авторскиС ΠΏΡ€Π°Π²Π°.Π’Π°ΠΊΠΈΠΌ ΠΎΠ±Ρ€Π°Π·ΠΎΠΌ, Ссли Π²Ρ‹ Π½Π΅ ΡƒΠ²Π΅Ρ€Π΅Π½Ρ‹, Ρ‡Ρ‚ΠΎ ΠΊΠΎΠ½Ρ‚Π΅Π½Ρ‚ находится Π½Π° Π’Π΅Π±-сайтС ΠΈΠ»ΠΈ ΠΏΠΎ ссылкС с Π½Π΅Π³ΠΎ Π½Π°Ρ€ΡƒΡˆΠ°Π΅Ρ‚ ваши авторскиС ΠΏΡ€Π°Π²Π°, Π²Π°ΠΌ слСдуСт сначала ΠΎΠ±Ρ€Π°Ρ‚ΠΈΡ‚ΡŒΡΡ ΠΊ ΡŽΡ€ΠΈΡΡ‚Ρƒ.

Π§Ρ‚ΠΎΠ±Ρ‹ ΠΎΡ‚ΠΏΡ€Π°Π²ΠΈΡ‚ΡŒ ΡƒΠ²Π΅Π΄ΠΎΠΌΠ»Π΅Π½ΠΈΠ΅, Π²Ρ‹ΠΏΠΎΠ»Π½ΠΈΡ‚Π΅ ΡΠ»Π΅Π΄ΡƒΡŽΡ‰ΠΈΠ΅ дСйствия:

Π’Ρ‹ Π΄ΠΎΠ»ΠΆΠ½Ρ‹ Π²ΠΊΠ»ΡŽΡ‡ΠΈΡ‚ΡŒ ΡΠ»Π΅Π΄ΡƒΡŽΡ‰Π΅Π΅:

ЀизичСская ΠΈΠ»ΠΈ элСктронная подпись правообладатСля ΠΈΠ»ΠΈ Π»ΠΈΡ†Π°, ΡƒΠΏΠΎΠ»Π½ΠΎΠΌΠΎΡ‡Π΅Π½Π½ΠΎΠ³ΠΎ Π΄Π΅ΠΉΡΡ‚Π²ΠΎΠ²Π°Ρ‚ΡŒ ΠΎΡ‚ ΠΈΡ… ΠΈΠΌΠ΅Π½ΠΈ; Π˜Π΄Π΅Π½Ρ‚ΠΈΡ„ΠΈΠΊΠ°Ρ†ΠΈΡ авторских ΠΏΡ€Π°Π², ΠΊΠΎΡ‚ΠΎΡ€Ρ‹Π΅, ΠΊΠ°ΠΊ утвСрТдаСтся, Π±Ρ‹Π»ΠΈ Π½Π°Ρ€ΡƒΡˆΠ΅Π½Ρ‹; ОписаниС Ρ…Π°Ρ€Π°ΠΊΡ‚Π΅Ρ€Π° ΠΈ Ρ‚ΠΎΡ‡Π½ΠΎΠ³ΠΎ мСстонахоТдСния ΠΊΠΎΠ½Ρ‚Π΅Π½Ρ‚Π°, ΠΊΠΎΡ‚ΠΎΡ€Ρ‹ΠΉ, ΠΏΠΎ Π²Π°ΡˆΠ΅ΠΌΡƒ мнСнию, Π½Π°Ρ€ΡƒΡˆΠ°Π΅Ρ‚ ваши авторскиС ΠΏΡ€Π°Π²Π°, Π² \ достаточно подробностСй, Ρ‡Ρ‚ΠΎΠ±Ρ‹ ΠΏΠΎΠ·Π²ΠΎΠ»ΠΈΡ‚ΡŒ Ρ€Π΅ΠΏΠ΅Ρ‚ΠΈΡ‚ΠΎΡ€Π°ΠΌ унивСрситСтских школ Π½Π°ΠΉΡ‚ΠΈ ΠΈ Ρ‚ΠΎΡ‡Π½ΠΎ ΠΈΠ΄Π΅Π½Ρ‚ΠΈΡ„ΠΈΡ†ΠΈΡ€ΠΎΠ²Π°Ρ‚ΡŒ этот ΠΊΠΎΠ½Ρ‚Π΅Π½Ρ‚; Π½Π°ΠΏΡ€ΠΈΠΌΠ΅Ρ€ Π½Π°ΠΌ трСбуСтся Π° ссылка Π½Π° ΠΊΠΎΠ½ΠΊΡ€Π΅Ρ‚Π½Ρ‹ΠΉ вопрос (Π° Π½Π΅ Ρ‚ΠΎΠ»ΡŒΠΊΠΎ Π½Π° Π½Π°Π·Π²Π°Π½ΠΈΠ΅ вопроса), ΠΊΠΎΡ‚ΠΎΡ€Ρ‹ΠΉ содСрТит содСрТаниС ΠΈ описаниС ΠΊ ΠΊΠ°ΠΊΠΎΠΉ ΠΊΠΎΠ½ΠΊΡ€Π΅Ρ‚Π½ΠΎΠΉ части вопроса – ΠΈΠ·ΠΎΠ±Ρ€Π°ΠΆΠ΅Π½ΠΈΡŽ, ссылкС, тСксту ΠΈ Ρ‚. Π΄. – относится ваша ΠΆΠ°Π»ΠΎΠ±Π°; Π’Π°ΡˆΠ΅ имя, адрСс, Π½ΠΎΠΌΠ΅Ρ€ Ρ‚Π΅Π»Π΅Ρ„ΠΎΠ½Π° ΠΈ адрСс элСктронной ΠΏΠΎΡ‡Ρ‚Ρ‹; Π° Ρ‚Π°ΠΊΠΆΠ΅ Π’Π°ΡˆΠ΅ заявлСниС: (Π°) Π²Ρ‹ добросовСстно ΠΏΠΎΠ»Π°Π³Π°Π΅Ρ‚Π΅, Ρ‡Ρ‚ΠΎ использованиС ΠΊΠΎΠ½Ρ‚Π΅Π½Ρ‚Π°, ΠΊΠΎΡ‚ΠΎΡ€Ρ‹ΠΉ, ΠΏΠΎ Π²Π°ΡˆΠ΅ΠΌΡƒ мнСнию, Π½Π°Ρ€ΡƒΡˆΠ°Π΅Ρ‚ ваши авторскиС ΠΏΡ€Π°Π²Π° Π½Π΅ Ρ€Π°Π·Ρ€Π΅ΡˆΠ΅Π½Ρ‹ Π·Π°ΠΊΠΎΠ½ΠΎΠΌ, Π²Π»Π°Π΄Π΅Π»ΡŒΡ†Π΅ΠΌ авторских ΠΏΡ€Π°Π² ΠΈΠ»ΠΈ Π΅Π³ΠΎ Π°Π³Π΅Π½Ρ‚ΠΎΠΌ; (Π±) Ρ‡Ρ‚ΠΎ всС информация, содСрТащаяся Π² вашСм Π£Π²Π΅Π΄ΠΎΠΌΠ»Π΅Π½ΠΈΠΈ ΠΎ Π½Π°Ρ€ΡƒΡˆΠ΅Π½ΠΈΠΈ, являСтся Ρ‚ΠΎΡ‡Π½ΠΎΠΉ, ΠΈ (c) ΠΏΠΎΠ΄ страхом наказания Π·Π° Π»ΠΆΠ΅ΡΠ²ΠΈΠ΄Π΅Ρ‚Π΅Π»ΡŒΡΡ‚Π²ΠΎ, Ρ‡Ρ‚ΠΎ Π²Ρ‹ Π»ΠΈΠ±ΠΎ Π²Π»Π°Π΄Π΅Π»Π΅Ρ† авторских ΠΏΡ€Π°Π², Π»ΠΈΠ±ΠΎ Π»ΠΈΡ†ΠΎ, ΡƒΠΏΠΎΠ»Π½ΠΎΠΌΠΎΡ‡Π΅Π½Π½ΠΎΠ΅ Π΄Π΅ΠΉΡΡ‚Π²ΠΎΠ²Π°Ρ‚ΡŒ ΠΎΡ‚ ΠΈΡ… ΠΈΠΌΠ΅Π½ΠΈ.

ΠžΡ‚ΠΏΡ€Π°Π²ΡŒΡ‚Π΅ ΠΆΠ°Π»ΠΎΠ±Ρƒ Π½Π°ΡˆΠ΅ΠΌΡƒ ΡƒΠΏΠΎΠ»Π½ΠΎΠΌΠΎΡ‡Π΅Π½Π½ΠΎΠΌΡƒ Π°Π³Π΅Π½Ρ‚Ρƒ ΠΏΠΎ адрСсу:

Π§Π°Ρ€Π»ΡŒΠ· Кон Varsity Tutors LLC
101 S. Hanley Rd, Suite 300
St. Louis, MO 63105

Или Π·Π°ΠΏΠΎΠ»Π½ΠΈΡ‚Π΅ Ρ„ΠΎΡ€ΠΌΡƒ Π½ΠΈΠΆΠ΅:

РСзюмС уравнСния для Π²Π΅Π»ΠΈΡ‡ΠΈΠ½Ρ‹ Ρ†Π΅Π½Ρ‚Ρ€ΠΎΡΡ‚Ρ€Π΅ΠΌΠΈΡ‚Π΅Π»ΡŒΠ½ΠΎΠΉ силы

ЀСлиция Π§Π΅Ρ€Ρ€ΠΈ
ΠœΠ΅Π½Π΅Π΄ΠΆΠ΅Ρ€ ΠΏΠΎ ΠΏΡ€ΠΎΠ΄ΡƒΠΊΡ‚Π°ΠΌ Π² области физичСских Π½Π°ΡƒΠΊ, Ρ„ΠΈΠ·ΠΈΠΊΠΈ ΠΈ Π½Π°ΡƒΠΊ ΠΎ Π—Π΅ΠΌΠ»Π΅

ΠŸΡ€Π°ΠΊΡ‚ΠΈΡ‡Π΅ΡΠΊΠΈ ΠΊΠ°ΠΆΠ΄Ρ‹ΠΉ ΡƒΡ€ΠΎΠΊ Ρ„ΠΈΠ·ΠΈΠΊΠΈ ΠΈΠ·ΡƒΡ‡Π°Π΅Ρ‚ ΠΊΡ€ΡƒΠ³ΠΎΠ²ΠΎΠ΅ Π΄Π²ΠΈΠΆΠ΅Π½ΠΈΠ΅.Π‘Ρ‚ΡƒΠ΄Π΅Π½Ρ‚Ρ‹ тратят ΠΌΠ½ΠΎΠ³ΠΎ Π²Ρ€Π΅ΠΌΠ΅Π½ΠΈ Π½Π° ΠΈΠ·ΡƒΡ‡Π΅Π½ΠΈΠ΅ основной Ρ„ΠΎΡ€ΠΌΡƒΠ»Ρ‹ ΠΈ Ρ€Π΅ΡˆΠ΅Π½ΠΈΠ΅ Π·Π°Π΄Π°Ρ‡. Π’ этой ΡΡ‚Π°Ρ‚ΡŒΠ΅ описываСтся Π²Ρ‹Π²ΠΎΠ΄ уравнСния для Ρ†Π΅Π½Ρ‚Ρ€ΠΎΡΡ‚Ρ€Π΅ΠΌΠΈΡ‚Π΅Π»ΡŒΠ½ΠΎΠΉ силы Π½Π° основС Π°Π»Π³Π΅Π±Ρ€Ρ‹.


Π’Ρ‹Π²ΠΎΠ΄ уравнСния

Π­Ρ‚ΠΎ ΠΏΠ΅Ρ€Π²ΠΎΠ΅ ΡƒΡ€Π°Π²Π½Π΅Π½ΠΈΠ΅ Π΄ΠΎΠ»ΠΆΠ½ΠΎ Π±Ρ‹Ρ‚ΡŒ Π²Π°ΠΌ Π·Π½Π°ΠΊΠΎΠΌΠΎ. Π­Ρ‚ΠΎ, вСроятно, Π½Π°ΠΈΠ±ΠΎΠ»Π΅Π΅ часто ΠΈΡΠΏΠΎΠ»ΡŒΠ·ΡƒΠ΅ΠΌΠΎΠ΅ ΡƒΡ€Π°Π²Π½Π΅Π½ΠΈΠ΅ Π² любом ΠΏΠ΅Ρ€Π²ΠΎΠΌ курсС Ρ„ΠΈΠ·ΠΈΠΊΠΈ ΠΈ способ, ΠΊΠΎΡ‚ΠΎΡ€Ρ‹ΠΌ Π±ΠΎΠ»ΡŒΡˆΠΈΠ½ΡΡ‚Π²ΠΎ студСнтов-Ρ„ΠΈΠ·ΠΈΠΊΠΎΠ² учатся Π²Ρ‹Ρ€Π°ΠΆΠ°Ρ‚ΡŒ Π’Ρ‚ΠΎΡ€ΠΎΠΉ Π·Π°ΠΊΠΎΠ½ ΠΡŒΡŽΡ‚ΠΎΠ½Π°:


Π£Ρ€Π°Π²Π½Π΅Π½ΠΈΠ΅ 1.

Чистая сила, Π΄Π΅ΠΉΡΡ‚Π²ΡƒΡŽΡ‰Π°Ρ Π½Π° ΠΎΠ±ΡŠΠ΅ΠΊΡ‚, Ρ€Π°Π²Π½Π° массС ΠΎΠ±ΡŠΠ΅ΠΊΡ‚Π°, ΡƒΠΌΠ½ΠΎΠΆΠ΅Π½Π½ΠΎΠΉ Π½Π° ускорСниС ΠΎΠ±ΡŠΠ΅ΠΊΡ‚Π°. Π£Ρ€Π°Π²Π½Π΅Π½ΠΈΠ΅ 2 примСняСт этот ΠΏΡ€ΠΈΠ½Ρ†ΠΈΠΏ ΠΊ ΠΎΠ±ΡŠΠ΅ΠΊΡ‚Π°ΠΌ, двиТущимся с Ρ€Π°Π²Π½ΠΎΠΌΠ΅Ρ€Π½Ρ‹ΠΌ ΠΊΡ€ΡƒΠ³ΠΎΠ²Ρ‹ΠΌ Π΄Π²ΠΈΠΆΠ΅Π½ΠΈΠ΅ΠΌ, ΠΊΠΎΠ³Π΄Π° ΠΎΠ±ΡŠΠ΅ΠΊΡ‚ двиТСтся с постоянной ΡΠΊΠΎΡ€ΠΎΡΡ‚ΡŒΡŽ, Π½ΠΎ ΠΏΡ€ΠΎΠ΄ΠΎΠ»ΠΆΠ°Π΅Ρ‚ ΠΌΠ΅Π½ΡΡ‚ΡŒ Π½Π°ΠΏΡ€Π°Π²Π»Π΅Π½ΠΈΠ΅, Ρ‚Π°ΠΊ Ρ‡Ρ‚ΠΎ ΠΎΠ½ двиТСтся ΠΏΠΎ ΠΊΡ€ΡƒΠ³ΠΎΠ²ΠΎΠΉ Ρ‚Ρ€Π°Π΅ΠΊΡ‚ΠΎΡ€ΠΈΠΈ. На ΠΎΠ±ΡŠΠ΅ΠΊΡ‚, двиТущийся Π² Ρ€Π°Π²Π½ΠΎΠΌΠ΅Ρ€Π½ΠΎΠΌ ΠΊΡ€ΡƒΠ³ΠΎΠ²ΠΎΠΌ Π΄Π²ΠΈΠΆΠ΅Π½ΠΈΠΈ, дСйствуСт сила, направлСнная Π² Ρ†Π΅Π½Ρ‚Ρ€ ΠΊΡ€ΡƒΠ³ΠΎΠ²ΠΎΠΉ Ρ‚Ρ€Π°Π΅ΠΊΡ‚ΠΎΡ€ΠΈΠΈ, называСмая Ρ†Π΅Π½Ρ‚Ρ€ΠΎΡΡ‚Ρ€Π΅ΠΌΠΈΡ‚Π΅Π»ΡŒΠ½ΠΎΠΉ силой . Π£Ρ€Π°Π²Π½Π΅Π½ΠΈΠ΅ 2 повторяСт Π£Ρ€Π°Π²Π½Π΅Π½ΠΈΠ΅ 1 для ΠΎΠ±ΡŠΠ΅ΠΊΡ‚ΠΎΠ², двиТущихся с Ρ€Π°Π²Π½ΠΎΠΌΠ΅Ρ€Π½Ρ‹ΠΌ ΠΊΡ€ΡƒΠ³ΠΎΠ²Ρ‹ΠΌ Π΄Π²ΠΈΠΆΠ΅Π½ΠΈΠ΅ΠΌ. F c ΠΎΠ±ΠΎΠ·Π½Π°Ρ‡Π°Π΅Ρ‚ Ρ†Π΅Π½Ρ‚Ρ€ΠΎΡΡ‚Ρ€Π΅ΠΌΠΈΡ‚Π΅Π»ΡŒΠ½ΡƒΡŽ силу, Π° c ΠΎΠ±ΠΎΠ·Π½Π°Ρ‡Π°Π΅Ρ‚ Ρ†Π΅Π½Ρ‚Ρ€ΠΎΡΡ‚Ρ€Π΅ΠΌΠΈΡ‚Π΅Π»ΡŒΠ½ΠΎΠ΅ ускорСниС.


Π£Ρ€Π°Π²Π½Π΅Π½ΠΈΠ΅ 2.

Π£Ρ€Π°Π²Π½Π΅Π½ΠΈΠ΅ для Π²Π΅Π»ΠΈΡ‡ΠΈΠ½Ρ‹ Ρ†Π΅Π½Ρ‚Ρ€ΠΎΡΡ‚Ρ€Π΅ΠΌΠΈΡ‚Π΅Π»ΡŒΠ½ΠΎΠΉ силы ΠΈΠΌΠ΅Π΅Ρ‚ ΡΠ»Π΅Π΄ΡƒΡŽΡ‰ΠΈΠΉ Π²ΠΈΠ΄:


Π£Ρ€Π°Π²Π½Π΅Π½ΠΈΠ΅ 3.

ΠŸΡ€ΠΈ описании Π»ΠΈΠ½Π΅ΠΉΠ½ΠΎΠ³ΠΎ двиТСния ускорСниС ΠΎΠ±Ρ‹Ρ‡Π½ΠΎ описываСтся ΠΊΠ°ΠΊ ΠΈΠ·ΠΌΠ΅Π½Π΅Π½ΠΈΠ΅ скорости, Π΄Π΅Π»Π΅Π½Π½ΠΎΠ΅ Π½Π° ΠΈΠ·ΠΌΠ΅Π½Π΅Π½ΠΈΠ΅ Π²ΠΎ Π²Ρ€Π΅ΠΌΠ΅Π½ΠΈ. ΠœΠ°Ρ‚Π΅ΠΌΠ°Ρ‚ΠΈΡ‡Π΅ΡΠΊΠΈ ускорСниС ΠΌΠΎΠΆΠ½ΠΎ Π²Ρ‹Ρ€Π°Π·ΠΈΡ‚ΡŒ Ρ‚Π°ΠΊ:


Π£Ρ€Π°Π²Π½Π΅Π½ΠΈΠ΅ 4.

Из Π£Ρ€Π°Π²Π½Π΅Π½ΠΈΠ΅ 2 ΠΈ Π£Ρ€Π°Π²Π½Π΅Π½ΠΈΠ΅ 3 , Ρ†Π΅Π½Ρ‚Ρ€ΠΎΡΡ‚Ρ€Π΅ΠΌΠΈΡ‚Π΅Π»ΡŒΠ½ΠΎΠ΅ ускорСниС ΠΈΠ»ΠΈ c , ΠΌΠΎΠΆΠ½ΠΎ Π²Ρ‹Ρ€Π°Π·ΠΈΡ‚ΡŒ ΠΊΠ°ΠΊ:


Π£Ρ€Π°Π²Π½Π΅Π½ΠΈΠ΅ 5.

Π’ ΡΠ»Π΅Π΄ΡƒΡŽΡ‰Π΅ΠΌ Ρ€Π°Π·Π΄Π΅Π»Π΅ Π±ΡƒΠ΄Π΅Ρ‚ объяснСно, ΠΏΠΎΡ‡Π΅ΠΌΡƒ Ρ†Π΅Π½Ρ‚Ρ€ΠΎΡΡ‚Ρ€Π΅ΠΌΠΈΡ‚Π΅Π»ΡŒΠ½ΠΎΠ΅ ускорСниС Ρ€Π°Π²Π½ΠΎ ΠΊΠ²Π°Π΄Ρ€Π°Ρ‚Ρƒ скорости, Π΄Π΅Π»Π΅Π½Π½ΠΎΠΌΡƒ Π½Π° радиус, ΠΊΠ°ΠΊ записано Π² ΡƒΡ€Π°Π²Π½Π΅Π½ΠΈΠΈ 5 .

Π’ ΡΠ»Π΅Π΄ΡƒΡŽΡ‰Π΅ΠΌ спискС ΠΎΠΏΡ€Π΅Π΄Π΅Π»Π΅Π½Ρ‹ символы, ΠΈΡΠΏΠΎΠ»ΡŒΠ·ΡƒΠ΅ΠΌΡ‹Π΅ Π² этом Ρ€Π°Π·Π΄Π΅Π»Π΅:
F = Π²Π΅Π»ΠΈΡ‡ΠΈΠ½Π° силы (Π² Π½ΡŒΡŽΡ‚ΠΎΠ½Π°Ρ…, Н)
F c = Ρ†Π΅Π½Ρ‚Ρ€ΠΎΡΡ‚Ρ€Π΅ΠΌΠΈΡ‚Π΅Π»ΡŒΠ½Π°Ρ сила, направлСнная Π² Ρ†Π΅Π½Ρ‚Ρ€ ΠΊΡ€ΡƒΠ³ΠΎΠ²ΠΎΠΉ Ρ‚Ρ€Π°Π΅ΠΊΡ‚ΠΎΡ€ΠΈΠΈ (Н )
ΠΌ = масса ΠΎΠ±ΡŠΠ΅ΠΊΡ‚Π°, двиТущСгося ΠΏΠΎ ΠΊΡ€ΡƒΠ³ΠΎΠ²ΠΎΠΉ Ρ‚Ρ€Π°Π΅ΠΊΡ‚ΠΎΡ€ΠΈΠΈ (Π² ΠΊΠΈΠ»ΠΎΠ³Ρ€Π°ΠΌΠΌΠ°Ρ…, ΠΊΠ³)
v = Ρ‚Π°Π½Π³Π΅Π½Ρ†ΠΈΠ°Π»ΡŒΠ½Π°Ρ ΡΠΊΠΎΡ€ΠΎΡΡ‚ΡŒ ΠΎΠ±ΡŠΠ΅ΠΊΡ‚Π° (ΠΌΠ΅Ρ‚Ρ€Ρ‹ Π² сСкунду, ΠΌ / с)
v 1 = Π½Π°Ρ‡Π°Π»ΡŒΠ½Π°Ρ ΡΠΊΠΎΡ€ΠΎΡΡ‚ΡŒ (ΠΌΠ΅Ρ‚Ρ€Ρ‹ Π² сСкунду, ΠΌ / с) (Π² Π½Π΅ΠΊΠΎΡ‚ΠΎΡ€Ρ‹Ρ… тСкстах это vi ΠΈΠ»ΠΈ v 0 )
v 2 = конСчная ΡΠΊΠΎΡ€ΠΎΡΡ‚ΡŒ (ΠΌΠ΅Ρ‚Ρ€Ρ‹ Π² сСкунду, ΠΌ / с) (Π² Π½Π΅ΠΊΠΎΡ‚ΠΎΡ€Ρ‹Ρ… тСкстах это v f )
r = радиус ΠΊΡ€ΡƒΠ³ΠΎΠ²ΠΎΠΉ Ρ‚Ρ€Π°Π΅ΠΊΡ‚ΠΎΡ€ΠΈΠΈ (Π² ΠΌΠ΅Ρ‚Ρ€Π°Ρ…, ΠΌ)
a = Π²Π΅Π»ΠΈΡ‡ΠΈΠ½Π° ускорСния (Π² ΠΌΠ΅Ρ‚Ρ€Π°Ρ… Π² сСкунду Π² ΠΊΠ²Π°Π΄Ρ€Π°Ρ‚Π΅, ΠΌ / с 2 )
a c = Ρ†Π΅Π½Ρ‚Ρ€ΠΎΡΡ‚Ρ€Π΅ΠΌΠΈΡ‚Π΅Π»ΡŒΠ½ΠΎΠ΅ ускорСниС, Π½Π°ΠΏΡ€Π°Π²Π»Π΅Π½Π½ΠΎΠ΅ Π² Ρ†Π΅Π½Ρ‚Ρ€ ΠΊΡ€ΡƒΠ³ΠΎΠ²ΠΎΠΉ Ρ‚Ρ€Π°Π΅ΠΊΡ‚ΠΎΡ€ΠΈΠΈ (ΠΌ / с 2 ) 9117 9 Ξ” ΠΎΠ±ΠΎΠ·Π½Π°Ρ‡Π°Π΅Ρ‚ ΠΈΠ·ΠΌΠ΅Π½Π΅Π½ΠΈΠ΅, Π½Π°ΠΏΡ€ΠΈΠΌΠ΅Ρ€ Ξ”v = ( v 2 v 1 )

ΠŸΡ€ΠΈΠΌΠ΅Ρ‡Π°Π½ΠΈΠ΅: Π­Ρ‚ΠΎΡ‚ Π²Ρ‹Π²ΠΎΠ΄ Π°Π½Π°Π»ΠΈΠ·ΠΈΡ€ΡƒΠ΅Ρ‚ Π²Π΅ΠΊΡ‚ΠΎΡ€Ρ‹, ΠΎΠΏΠΈΡΡ‹Π²Π°ΡŽΡ‰ΠΈΠ΅ Π΄Π²ΠΈΠΆΠ΅Π½ΠΈΠ΅ ΠΎΠ±ΡŠΠ΅ΠΊΡ‚Π°, двиТущСгося Π² Ρ€Π°Π²Π½ΠΎΠΌΠ΅Ρ€Π½ΠΎΠΌ ΠΊΡ€ΡƒΠ³ΠΎΠ²ΠΎΠΌ Π΄Π²ΠΈΠΆΠ΅Π½ΠΈΠΈ, ΠΈ ΠΈΡΠΏΠΎΠ»ΡŒΠ·ΡƒΠ΅Ρ‚ Π°Π½Π°Π»ΠΎΠ³ΠΈΡ‡Π½Ρ‹Π΅ Ρ‚Ρ€Π΅ΡƒΠ³ΠΎΠ»ΡŒΠ½ΠΈΠΊΠΈ, Ρ‡Ρ‚ΠΎΠ±Ρ‹ ΡΠ²ΡΠ·Π°Ρ‚ΡŒ ΡΠΊΠΎΡ€ΠΎΡΡ‚ΡŒ ΠΎΠ±ΡŠΠ΅ΠΊΡ‚Π° с радиусом ΠΊΡ€ΡƒΠ³ΠΎΠ²ΠΎΠΉ Ρ‚Ρ€Π°Π΅ΠΊΡ‚ΠΎΡ€ΠΈΠΈ.


ΠŸΠΎΡ‡Π΅ΠΌΡƒ Ρ†Π΅Π½Ρ‚Ρ€ΠΎΡΡ‚Ρ€Π΅ΠΌΠΈΡ‚Π΅Π»ΡŒΠ½ΠΎΠ΅ ускорСниС Ρ€Π°Π²Π½ΠΎ ΠΊΠ²Π°Π΄Ρ€Π°Ρ‚Ρƒ скорости, Ρ€Π°Π·Π΄Π΅Π»Π΅Π½Π½ΠΎΠΌΡƒ Π½Π° радиус

Рассмотрим ΠΎΠ±ΡŠΠ΅ΠΊΡ‚, двиТущийся с постоянной ΡΠΊΠΎΡ€ΠΎΡΡ‚ΡŒΡŽ ΠΏΠΎ ΠΊΡ€ΡƒΠ³ΠΎΠ²ΠΎΠΉ Ρ‚Ρ€Π°Π΅ΠΊΡ‚ΠΎΡ€ΠΈΠΈ. Π‘ΠΊΠΎΡ€ΠΎΡΡ‚ΡŒ ΠΎΠ±ΡŠΠ΅ΠΊΡ‚Π° постоянна, Π½ΠΎ ΡΠΊΠΎΡ€ΠΎΡΡ‚ΡŒ мСняСтся, ΠΏΠΎΡ‚ΠΎΠΌΡƒ Ρ‡Ρ‚ΠΎ Π½Π°ΠΏΡ€Π°Π²Π»Π΅Π½ΠΈΠ΅ ΠΎΠ±ΡŠΠ΅ΠΊΡ‚Π° постоянно мСняСтся. Рассмотрим ΡΠΊΠΎΡ€ΠΎΡΡ‚ΡŒ Π² Π΄Π²Π° ΠΌΠΎΠΌΠ΅Π½Ρ‚Π° Π½Π° ΠΊΡ€ΡƒΠ³ΠΎΠ²ΠΎΠΉ Ρ‚Ρ€Π°Π΅ΠΊΡ‚ΠΎΡ€ΠΈΠΈ. На рис. 1 v 1 прСдставляСт Π½Π°Ρ‡Π°Π»ΡŒΠ½ΡƒΡŽ ΡΠΊΠΎΡ€ΠΎΡΡ‚ΡŒ. На рис. 2 v 2 прСдставляСт ΡΠΊΠΎΡ€ΠΎΡΡ‚ΡŒ Ρ‡Π΅Ρ€Π΅Π· ΠΊΠΎΡ€ΠΎΡ‚ΠΊΠΈΠΉ ΠΏΡ€ΠΎΠΌΠ΅ΠΆΡƒΡ‚ΠΎΠΊ Π²Ρ€Π΅ΠΌΠ΅Π½ΠΈ.


Π£Ρ€Π°Π²Π½Π΅Π½ΠΈΠ΅ 1.

Рис. 1. Π”ΠΈΠ°Π³Ρ€Π°ΠΌΠΌΡ‹ 1 (слСва) ΠΈ 2 (справа) ΠΈΠ»Π»ΡŽΡΡ‚Ρ€ΠΈΡ€ΡƒΡŽΡ‚ ΠΈΠ·ΠΌΠ΅Π½Π΅Π½ΠΈΠ΅ направлСния Π²Π΅ΠΊΡ‚ΠΎΡ€Π° скорости ΠΎΡ‚ Π²Ρ€Π΅ΠΌΠ΅Π½ΠΈ 1 ΠΊΠΎ Π²Ρ€Π΅ΠΌΠ΅Π½ΠΈ 2. На Π΄ΠΈΠ°Π³Ρ€Π°ΠΌΠΌΠ΅ 2 ΠΏΠΎΠΊΠ°Π·Π°Π½ΠΎ, Ρ‡Ρ‚ΠΎ ΡƒΠ³ΠΎΠ» ΠΌΠ΅ΠΆΠ΄Ρƒ Π²Π΅ΠΊΡ‚ΠΎΡ€Π°ΠΌΠΈ скорости Ρ€Π°Π²Π΅Π½ ΡƒΠ³Π»Ρƒ ΠΌΠ΅ΠΆΠ΄Ρƒ радиусами Π² ΠΌΠΎΠΌΠ΅Π½Ρ‚ Π²Ρ€Π΅ΠΌΠ΅Π½ΠΈ 1 ΠΈ ΠΌΠΎΠΌΠ΅Π½Ρ‚ Π²Ρ€Π΅ΠΌΠ΅Π½ΠΈ 2.


Рис. 2. Π”ΠΈΠ°Π³Ρ€Π°ΠΌΠΌΡ‹ 3, 4 ΠΈ 5.

Пока Π΄Π»ΠΈΠ½Π° ΠΈ Π½Π°ΠΏΡ€Π°Π²Π»Π΅Π½ΠΈΠ΅ Π²Π΅ΠΊΡ‚ΠΎΡ€ΠΎΠ² ΠΎΡΡ‚Π°ΡŽΡ‚ΡΡ Π½Π΅ΠΈΠ·ΠΌΠ΅Π½Π½Ρ‹ΠΌΠΈ, ΠΈΡ… ΠΌΠΎΠΆΠ½ΠΎ Ρ€ΠΈΡΠΎΠ²Π°Ρ‚ΡŒ Π»ΡŽΠ±Ρ‹ΠΌ ΡƒΠ΄ΠΎΠ±Π½Ρ‹ΠΌ способом.На Π΄ΠΈΠ°Π³Ρ€Π°ΠΌΠΌΠ΅ 3 ΠΏΠΎΠΊΠ°Π·Π°Π½ΠΎ слоТСниС ΠΈ Π²Ρ‹Ρ‡ΠΈΡ‚Π°Π½ΠΈΠ΅ Π²Π΅ΠΊΡ‚ΠΎΡ€ΠΎΠ² v 1 ΠΈ v 2 .

Π’Π΅ΠΊΡ‚ΠΎΡ€Ρ‹ -v 1 , v 2 ΠΈ Ξ”v ΠΎΠ±Ρ€Π°Π·ΡƒΡŽΡ‚ Ρ‚Ρ€Π΅ΡƒΠ³ΠΎΠ»ΡŒΠ½ΠΈΠΊ, ΠΊΠΎΡ‚ΠΎΡ€Ρ‹ΠΉ ΠΏΠΎΡ…ΠΎΠΆ Π½Π° Ρ‚Ρ€Π΅ΡƒΠ³ΠΎΠ»ΡŒΠ½ΠΈΠΊ, ΠΎΠ±Ρ€Π°Π·ΠΎΠ²Π°Π½Π½Ρ‹ΠΉ радиусами, нарисованными Π² ΠΌΠΎΠΌΠ΅Π½Ρ‚ Π²Ρ€Π΅ΠΌΠ΅Π½ΠΈ 1 ΠΈ ΠΌΠΎΠΌΠ΅Π½Ρ‚ Π²Ρ€Π΅ΠΌΠ΅Π½ΠΈ 2, ΠΈ Ρ…ΠΎΡ€Π΄ΠΎΠΉ Ξ”r .

Π’Π΅Π»ΠΈΡ‡ΠΈΠ½Π° Π²Π΅ΠΊΡ‚ΠΎΡ€ΠΎΠ² v 1 ΠΈ v 2 Ρ€Π°Π²Π½Π° ΠΈ ΠΈΠΌΠ΅Π΅Ρ‚ Π²Π΅Π»ΠΈΡ‡ΠΈΠ½Ρƒ v.

β”‚ v 1 β”‚ = β”‚ v 2 β”‚ = v

ΠŸΠΎΡΠΊΠΎΠ»ΡŒΠΊΡƒ Ρ‚Ρ€Π΅ΡƒΠ³ΠΎΠ»ΡŒΠ½ΠΈΠΊΠΈ ΠΏΠΎΡ…ΠΎΠΆΠΈ, Π²Π΅Ρ€Π½ΠΎ ΡΠ»Π΅Π΄ΡƒΡŽΡ‰Π΅Π΅:


Π£Ρ€Π°Π²Π½Π΅Π½ΠΈΠ΅ 6.

Ξ”r = Π΄Π»ΠΈΠ½Π° Ρ…ΠΎΡ€Π΄Ρ‹ ΠΌΠ΅ΠΆΠ΄Ρƒ мСстополоТСниСм ΠΎΠ±ΡŠΠ΅ΠΊΡ‚Π° Π² ΠΌΠΎΠΌΠ΅Π½Ρ‚ Π²Ρ€Π΅ΠΌΠ΅Π½ΠΈ 1 ΠΈ ΠΌΠΎΠΌΠ΅Π½Ρ‚ Π²Ρ€Π΅ΠΌΠ΅Π½ΠΈ 2.
r = радиус окруТности.

ΠšΡ€ΡƒΠ³ΠΎΠ²ΠΎΠΉ сСгмСнт s прСдставляСт собой расстояниС, ΠΏΡ€ΠΎΠΉΠ΄Π΅Π½Π½ΠΎΠ΅ ΠΎΠ±ΡŠΠ΅ΠΊΡ‚ΠΎΠΌ с ΠΌΠΎΠΌΠ΅Π½Ρ‚Π° 1 Π΄ΠΎ ΠΌΠΎΠΌΠ΅Π½Ρ‚Π° 2.


Π£Ρ€Π°Π²Π½Π΅Π½ΠΈΠ΅ 7.

Если ΡƒΠ³ΠΎΠ» ΞΈ ΠΎΡ‡Π΅Π½ΡŒ ΠΌΠ°Π», ΠΌΠΎΠΆΠ½ΠΎ ΡΠ΄Π΅Π»Π°Ρ‚ΡŒ ΡΠ»Π΅Π΄ΡƒΡŽΡ‰Π΅Π΅ ΠΏΡ€ΠΈΠ±Π»ΠΈΠΆΠ΅Π½ΠΈΠ΅.


Π£Ρ€Π°Π²Π½Π΅Π½ΠΈΠ΅ 8.

Π­Ρ‚ΠΎ позволяСт ΡΠ΄Π΅Π»Π°Ρ‚ΡŒ ΡΠ»Π΅Π΄ΡƒΡŽΡ‰ΡƒΡŽ Π·Π°ΠΌΠ΅Π½Ρƒ:


Π£Ρ€Π°Π²Π½Π΅Π½ΠΈΠ΅ 9.

ΠŸΠΎΠ΄ΡΡ‚Π°Π²Π»ΡΡ v Ξ”t вмСсто Ξ”r Π² Π£Ρ€Π°Π²Π½Π΅Π½ΠΈΠ΅ 6 :


Π£Ρ€Π°Π²Π½Π΅Π½ΠΈΠ΅ 10.

ΠŸΠ΅Ρ€Π΅ΡΡ‚Π°Π½ΠΎΠ²ΠΊΠ° для Ξ” v / Ξ”t Π΄Π°Π΅Ρ‚ Π£Ρ€Π°Π²Π½Π΅Π½ΠΈΠ΅ 11.


Π£Ρ€Π°Π²Π½Π΅Π½ΠΈΠ΅ 11.

Ξ”v / Ξ”t – это ускорСниС, ΡƒΠΊΠ°Π·Π°Π½Π½ΠΎΠ΅ Π² ΡƒΡ€Π°Π²Π½Π΅Π½ΠΈΠΈ 4 :

Π£ΠΌΠ½ΠΎΠΆΠ΅Π½ΠΈΠ΅ ускорСния Π½Π° массу ΠΎΠ±ΡŠΠ΅ΠΊΡ‚Π° Π΄Π°Π΅Ρ‚ Ρ†Π΅Π½Ρ‚Ρ€ΠΎΡΡ‚Ρ€Π΅ΠΌΠΈΡ‚Π΅Π»ΡŒΠ½ΡƒΡŽ силу, ΠΊΠ°ΠΊ ΠΏΠΎΠΊΠ°Π·Π°Π½ΠΎ Π² ΡƒΡ€Π°Π²Π½Π΅Π½ΠΈΠΈ (Π£Ρ€Π°Π²Π½Π΅Π½ΠΈΠ΅ 3) .

Π£Ρ€Π°Π²Π½Π΅Π½ΠΈΠ΅ 3 ΠΈ Π£Ρ€Π°Π²Π½Π΅Π½ΠΈΠ΅ 4 ΠΌΠΎΠΆΠ½ΠΎ ΠΈΡΠΏΠΎΠ»ΡŒΠ·ΠΎΠ²Π°Ρ‚ΡŒ для Ρ€Π΅ΡˆΠ΅Π½ΠΈΡ ΠΌΠ½ΠΎΠ³ΠΈΡ… физичСских Π·Π°Π΄Π°Ρ‡.


РСсурсы

К этим издСлиям ΠΌΠΎΠ³ΡƒΡ‚ ΠΏΡ€ΠΈΠΌΠ΅Π½ΡΡ‚ΡŒΡΡ уравнСния Ρ†Π΅Π½Ρ‚Ρ€ΠΎΡΡ‚Ρ€Π΅ΠΌΠΈΡ‚Π΅Π»ΡŒΠ½ΠΎΠΉ силы ΠΈ Ρ†Π΅Π½Ρ‚Ρ€ΠΎΡΡ‚Ρ€Π΅ΠΌΠΈΡ‚Π΅Π»ΡŒΠ½ΠΎΠ³ΠΎ ускорСния.

Π¦Π΅Π½Ρ‚Ρ€ΠΎΡΡ‚Ρ€Π΅ΠΌΠΈΡ‚Π΅Π»ΡŒΠ½Π°Ρ ΠΈ цСнтробСТная сила – ускорСниС

Π¦Π΅Π½Ρ‚Ρ€ΠΎΡΡ‚Ρ€Π΅ΠΌΠΈΡ‚Π΅Π»ΡŒΠ½Π°Ρ ΠΈ цСнтробСТная сила – это ΠΏΠ°Ρ€Π° сил дСйствия-противодСйствия, связанная с ΠΊΡ€ΡƒΠ³ΠΎΠ²Ρ‹ΠΌ Π΄Π²ΠΈΠΆΠ΅Π½ΠΈΠ΅ΠΌ.

Π¦Π΅Π½Ρ‚Ρ€ΠΎΡΡ‚Ρ€Π΅ΠΌΠΈΡ‚Π΅Π»ΡŒΠ½ΠΎΠ΅ ускорСниС

Π‘ΠΊΠΎΡ€ΠΎΡΡ‚ΡŒ – это Π²Π΅ΠΊΡ‚ΠΎΡ€, ΠΎΠΏΡ€Π΅Π΄Π΅Π»ΡΡŽΡ‰ΠΈΠΉ, насколько быстро (ΠΈΠ»ΠΈ ΠΌΠ΅Π΄Π»Π΅Π½Π½ΠΎ) ΠΏΡ€ΠΎΠΉΠ΄Π΅Π½ΠΎ расстояниС ΠΈ Π½Π°ΠΏΡ€Π°Π²Π»Π΅Π½ΠΈΠ΅ двиТСния. ΠŸΠΎΡΠΊΠΎΠ»ΡŒΠΊΡƒ Π²Π΅ΠΊΡ‚ΠΎΡ€ скорости (Π½Π°ΠΏΡ€Π°Π²Π»Π΅Π½ΠΈΠ΅) Ρ‚Π΅Π»Π° измСняСтся ΠΏΡ€ΠΈ Π΄Π²ΠΈΠΆΠ΅Π½ΠΈΠΈ ΠΏΠΎ окруТности – Π²ΠΎΠ·Π½ΠΈΠΊΠ°Π΅Ρ‚ ускорСниС.

Π­Ρ‚ΠΎ ускорСниС называСтся Ρ†Π΅Π½Ρ‚Ρ€ΠΎΡΡ‚Ρ€Π΅ΠΌΠΈΡ‚Π΅Π»ΡŒΠ½Ρ‹ΠΌ ускорСниСм ΠΈ ΠΌΠΎΠΆΠ΅Ρ‚ Π±Ρ‹Ρ‚ΡŒ Π²Ρ‹Ρ€Π°ΠΆΠ΅Π½ΠΎ ΠΊΠ°ΠΊ

a c = v 2 / r

= Ο‰ 2 r

= (2 Ο€ n ΠΎΠ± / с ) 2 r

= (2 Ο€ n ΠΎΠ± / ΠΌΠΈΠ½ /60) 2 r

= (Ο€ n ΠΎΠ± / ΠΌΠΈΠ½ /30) r (1)

Π³Π΄Π΅

a c = Ρ†Π΅Π½Ρ‚Ρ€ΠΎΡΡ‚Ρ€Π΅ΠΌΠΈΡ‚Π΅Π»ΡŒΠ½ΠΎΠ΅ ускорСниС (ΠΌ / с 2 , Ρ„ΡƒΡ‚ / с 2 )

v = Ρ‚Π°Π½Π³Π΅Π½Ρ†ΠΈΠ°Π»ΡŒΠ½Π°Ρ ΡΠΊΠΎΡ€ΠΎΡΡ‚ΡŒ (ΠΌ / с, Ρ„ΡƒΡ‚ / с)

r = радиус окруТности (ΠΌ, Ρ„ΡƒΡ‚)

Ο‰ = угловая ΡΠΊΠΎΡ€ΠΎΡΡ‚ΡŒ (Ρ€Π°Π΄ / с)

n ΠΎΠ± / с = ΠΎΠ±ΠΎΡ€ΠΎΡ‚ΠΎΠ² Π² сСкунду (ΠΎΠ± / с, 1 / с)

n ΠΎΠ± / ΠΌΠΈΠ½ = ΠΎΠ±ΠΎΡ€ΠΎΡ‚ΠΎΠ² Π² ΠΌΠΈΠ½ΡƒΡ‚Ρƒ (ΠΎΠ± / ΠΌΠΈΠ½, 1 / ΠΌΠΈΠ½)

Π¦Π΅Π½Ρ‚Ρ€ΠΎΡΡ‚Ρ€Π΅ΠΌΠΈΡ‚Π΅Π»ΡŒΠ½Π°Ρ сила

По сСкундам ΠΡŒΡŽΡ‚ΠΎΠ½Π° По Π·Π°ΠΊΠΎΠ½Ρƒ Ρ†Π΅Π½Ρ‚Ρ€ΠΎΡΡ‚Ρ€Π΅ΠΌΠΈΡ‚Π΅Π»ΡŒΠ½Π°Ρ сила ΠΌΠΎΠΆΠ΅Ρ‚ Π±Ρ‹Ρ‚ΡŒ Π²Ρ‹Ρ€Π°ΠΆΠ΅Π½Π° ΠΊΠ°ΠΊ

F c = ma c

= mv 2 / r

= m Ο‰ 2 r 9144 = m (2 Ο€ n s ) 2 r

= m (2 Ο€ n ΠΎΠ± / ΠΌΠΈΠ½ /60) 2 r

= ΠΌ (Ο€ n ΠΎΠ± / ΠΌΠΈΠ½ /30) 2 ΠΎΠ± (2)

Π³Π΄Π΅

F c = Ρ†Π΅Π½Ρ‚Ρ€ΠΎΡΡ‚Ρ€Π΅ΠΌΠΈΡ‚Π΅Π»ΡŒΠ½Π°Ρ сила (Н, Ρ„ΡƒΠ½Ρ‚ f )

m = масса (ΠΊΠ³, ΠΏΡƒΠ»ΠΈ )

Π’Ρ€Π΅Ρ‚ΠΈΠΉ Π·Π°ΠΊΠΎΠ½ ΠΡŒΡŽΡ‚ΠΎΠ½Π° Π¦Π΅Π½Ρ‚Ρ€ΠΎΡΡ‚Ρ€Π΅ΠΌΠΈΡ‚Π΅Π»ΡŒΠ½Π°Ρ сила, Π΄Π΅ΠΉΡΡ‚Π²ΡƒΡŽΡ‰Π°Ρ Π½Π° ΠΎΠ±ΡŠΠ΅ΠΊΡ‚, ΠΈΠΌΠ΅Π΅Ρ‚ Ρ†Π΅Π½Ρ‚Ρ€ΠΎΠ±Π΅ΠΆΠ½ΡƒΡŽ силу Ρ‚Π°ΠΊΠΎΠΉ ΠΆΠ΅ Π²Π΅Π»ΠΈΡ‡ΠΈΠ½Ρ‹, Π΄Π΅ΠΉΡΡ‚Π²ΡƒΡŽΡ‰ΡƒΡŽ Π² ΠΏΡ€ΠΎΡ‚ΠΈΠ²ΠΎΠΏΠΎΠ»ΠΎΠΆΠ½ΠΎΠΌ Π½Π°ΠΏΡ€Π°Π²Π»Π΅Π½ΠΈΠΈ.

ΠŸΡ€ΠΈΠΌΠ΅Ρ€ – Ρ†Π΅Π½Ρ‚Ρ€ΠΎΡΡ‚Ρ€Π΅ΠΌΠΈΡ‚Π΅Π»ΡŒΠ½ΠΎΠ΅ ускорСниС ΠΈ сила, Π΄Π΅ΠΉΡΡ‚Π²ΡƒΡŽΡ‰Π°Ρ Π½Π° Π°Π²Ρ‚ΠΎΠΌΠΎΠ±ΠΈΠ»ΡŒ ΠΏΠΎ ΠΊΡ€ΠΈΠ²ΠΎΠΉ

ΠœΠ΅Ρ‚Ρ€ΠΈΡ‡Π΅ΡΠΊΠΈΠ΅ Π΅Π΄ΠΈΠ½ΠΈΡ†Ρ‹

ΠΠ²Ρ‚ΠΎΠΌΠΎΠ±ΠΈΠ»ΡŒ с массой 1000 ΠΊΠ³ ΠΏΡ€ΠΎΠ΅Π·ΠΆΠ°Π΅Ρ‚ ΠΏΠΎ ΠΊΡ€ΠΈΠ²ΠΎΠΉ радиусом 200 ΠΌ со ΡΠΊΠΎΡ€ΠΎΡΡ‚ΡŒΡŽ 50 ΠΊΠΌ / Ρ‡ . Π¦Π΅Π½Ρ‚Ρ€ΠΎΡΡ‚Ρ€Π΅ΠΌΠΈΡ‚Π΅Π»ΡŒΠ½ΠΎΠ΅ ускорСниС ΠΌΠΎΠΆΠ½ΠΎ Ρ€Π°ΡΡΡ‡ΠΈΡ‚Π°Ρ‚ΡŒ ΠΊΠ°ΠΊ

a c = ((50 ΠΊΠΌ / Ρ‡) (1000 ΠΌ / ΠΊΠΌ) (1/3600 Ρ‡ / с)) 2 / (200 ΠΌ)

= 0,965 м / с 2

= 0.1 g

, Π³Π΄Π΅

1 g = ускорСниС свободного падСния (9,81 ΠΌ / с 2 )

Π¦Π΅Π½Ρ‚Ρ€ΠΎΡΡ‚Ρ€Π΅ΠΌΠΈΡ‚Π΅Π»ΡŒΠ½Π°Ρ сила ΠΌΠΎΠΆΠ΅Ρ‚ Π±Ρ‹Ρ‚ΡŒ рассчитана ΠΊΠ°ΠΊ

F c = (1000 кг) ( 0,965 м / с 2 )

= 965 Н

= 0,97 кН

Π’ зависимости ΠΎΡ‚ силы тяТСсти – вСс:

F г = (1000 кг) ( 9,81 м / с 2 )

= 9810 N

= 9.8 кН

Π˜ΠΌΠΏΠ΅Ρ€ΡΠΊΠΈΠ΅ Π΅Π΄ΠΈΠ½ΠΈΡ†Ρ‹

ΠΠ²Ρ‚ΠΎΠΌΠΎΠ±ΠΈΠ»ΡŒ с массой (сила тяТСсти) 3000 Ρ„ΡƒΠ½Ρ‚ΠΎΠ² двиТСтся ΠΏΠΎ ΠΊΡ€ΠΈΠ²ΠΎΠΉ радиусом 100 Ρ„ΡƒΡ‚ΠΎΠ² со ΡΠΊΠΎΡ€ΠΎΡΡ‚ΡŒΡŽ 15 миль / Ρ‡ .

ΠœΠ°ΡΡΡƒ автомобиля ΠΌΠΎΠΆΠ½ΠΎ Ρ€Π°ΡΡΡ‡ΠΈΡ‚Π°Ρ‚ΡŒ ΠΊΠ°ΠΊ

ΠΌ = (3000 Ρ„ΡƒΠ½Ρ‚ΠΎΠ²) / (32 Ρ„ΡƒΡ‚ / с 2 )

= 94 ΠΏΡƒΠ»ΠΈ

Π¦Π΅Π½Ρ‚Ρ€ΠΎΡΡ‚Ρ€Π΅ΠΌΠΈΡ‚Π΅Π»ΡŒΠ½ΠΎΠ΅ ускорСниС ΠΌΠΎΠΆΠ½ΠΎ Ρ€Π°ΡΡΡ‡ΠΈΡ‚Π°Ρ‚ΡŒ ΠΊΠ°ΠΊ

a c = ((15 миль / Ρ‡) (5280 Ρ„ΡƒΡ‚ΠΎΠ² / миля) / (3600 с / Ρ‡)) 2 / (100 Ρ„ΡƒΡ‚ΠΎΠ²)

= 4.84 Ρ„ΡƒΡ‚ / с 2

Π¦Π΅Π½Ρ‚Ρ€ΠΎΡΡ‚Ρ€Π΅ΠΌΠΈΡ‚Π΅Π»ΡŒΠ½ΡƒΡŽ силу ΠΌΠΎΠΆΠ½ΠΎ Ρ€Π°ΡΡΡ‡ΠΈΡ‚Π°Ρ‚ΡŒ ΠΊΠ°ΠΊ

F c = (94 ΠΏΡƒΠ»ΠΈ) (4,84 Ρ„ΡƒΡ‚ / с 2 )

= 455 Ρ„ΡƒΠ½Ρ‚ΠΎΠ² f

Π¦Π΅Π½Ρ‚Ρ€ΠΎΡΡ‚Ρ€Π΅ΠΌΠΈΡ‚Π΅Π»ΡŒΠ½Ρ‹ΠΉ (Ρ†Π΅Π½Ρ‚Ρ€ΠΎΠ±Π΅ΠΆΠ½Ρ‹ΠΉ) ΠΊΠ°Π»ΡŒΠΊΡƒΠ»ΡΡ‚ΠΎΡ€ – ΡΠΊΠΎΡ€ΠΎΡΡ‚ΡŒ

Π­Ρ‚ΠΎΡ‚ ΠΊΠ°Π»ΡŒΠΊΡƒΠ»ΡΡ‚ΠΎΡ€ ΠΌΠΎΠΆΠ½ΠΎ ΠΈΡΠΏΠΎΠ»ΡŒΠ·ΠΎΠ²Π°Ρ‚ΡŒ, Ссли извСстна ΡΠΊΠΎΡ€ΠΎΡΡ‚ΡŒ ΠΎΠ±ΡŠΠ΅ΠΊΡ‚Π° – Π½Π°ΠΏΡ€ΠΈΠΌΠ΅Ρ€, автомобиля Π½Π° ΠΏΠΎΠ²ΠΎΡ€ΠΎΡ‚Π΅.

Π¦Π΅Π½Ρ‚Ρ€ΠΎΡΡ‚Ρ€Π΅ΠΌΠΈΡ‚Π΅Π»ΡŒΠ½Π°Ρ (цСнтробСТная) сила – ΠΎΠ± / ΠΌΠΈΠ½

Π£Ρ€Π°Π²Π½Π΅Π½ΠΈΠ΅ (2) ΠΌΠΎΠΆΠ½ΠΎ ΠΈΠ·ΠΌΠ΅Π½ΠΈΡ‚ΡŒ, Ρ‡Ρ‚ΠΎΠ±Ρ‹ Π²Ρ‹Ρ€Π°Π·ΠΈΡ‚ΡŒ Ρ†Π΅Π½Ρ‚Ρ€ΠΎΡΡ‚Ρ€Π΅ΠΌΠΈΡ‚Π΅Π»ΡŒΠ½ΡƒΡŽ ΠΈΠ»ΠΈ Ρ†Π΅Π½Ρ‚Ρ€ΠΎΠ±Π΅ΠΆΠ½ΡƒΡŽ силу ΠΊΠ°ΠΊ Ρ„ΡƒΠ½ΠΊΡ†ΠΈΡŽ числа ΠΎΠ±ΠΎΡ€ΠΎΡ‚ΠΎΠ² Π² ΠΌΠΈΠ½ΡƒΡ‚Ρƒ – ΠΎΠ± / ΠΌΠΈΠ½ – ΠΊΠ°ΠΊ

F c = 0.01097 mrn ΠΎΠ± / ΠΌΠΈΠ½ 2 (3)

Π³Π΄Π΅

n ΠΎΠ± / ΠΌΠΈΠ½ = число ΠΎΠ±ΠΎΡ€ΠΎΡ‚ΠΎΠ² Π² ΠΌΠΈΠ½ΡƒΡ‚Ρƒ (ΠΎΠ± / ΠΌΠΈΠ½)

Π¦Π΅Π½Ρ‚Ρ€ΠΎΡΡ‚Ρ€Π΅ΠΌΠΈΡ‚Π΅Π»ΡŒΠ½Ρ‹ΠΉ (Ρ†Π΅Π½Ρ‚Ρ€ΠΎΠ±Π΅ΠΆΠ½Ρ‹ΠΉ) ΠšΠ°Π»ΡŒΠΊΡƒΠ»ΡΡ‚ΠΎΡ€ – ΠΎΠ± / ΠΌΠΈΠ½

ΠΌΠΎΠΆΠ½ΠΎ ΠΈΡΠΏΠΎΠ»ΡŒΠ·ΠΎΠ²Π°Ρ‚ΡŒ ΡΠΊΠΎΡ€ΠΎΡΡ‚ΡŒ вращСния ΠΎΠ±ΡŠΠ΅ΠΊΡ‚Π° извСстна – ΠΊΠ°ΠΊ Ρ‚ΠΎΠΊΠ°Ρ€Π½Ρ‹ΠΉ стакан Π² Ρ‚ΠΎΠΊΠ°Ρ€Π½ΠΎΠΌ станкС.

ЦСнтробСТная сила

Π‘ΠΈΠ»Π° – это абстракция, ΠΏΡ€Π΅Π΄ΡΡ‚Π°Π²Π»ΡΡŽΡ‰Π°Ρ Ρ‚ΠΎΠ»ΠΊΠ°ΡŽΡ‰Π΅Π΅ ΠΈ тянущСС взаимодСйствиС ΠΌΠ΅ΠΆΠ΄Ρƒ ΠΎΠ±ΡŠΠ΅ΠΊΡ‚Π°ΠΌΠΈ.Π’Ρ€Π΅Ρ‚ΠΈΠΉ Π·Π°ΠΊΠΎΠ½ ΠΡŒΡŽΡ‚ΠΎΠ½Π° гласит, Ρ‡Ρ‚ΠΎ

  • для ΠΊΠ°ΠΆΠ΄ΠΎΠΉ Π΄Π΅ΠΉΡΡ‚Π²ΡƒΡŽΡ‰Π΅ΠΉ силы сущСствуСт равная ΠΈ противополоТная сила Ρ€Π΅Π°ΠΊΡ†ΠΈΠΈ.

Π‘Π»Π΅Π΄ΠΎΠ²Π°Ρ‚Π΅Π»ΡŒΠ½ΠΎ, Π΄ΠΎΠ»ΠΆΠ½Π° Π±Ρ‹Ρ‚ΡŒ равная ΠΈ противополоТная Ρ†Π΅Π½Ρ‚Ρ€ΠΎΡΡ‚Ρ€Π΅ΠΌΠΈΡ‚Π΅Π»ΡŒΠ½ΠΎΠΉ силС сила Ρ€Π΅Π°ΠΊΡ†ΠΈΠΈ – цСнтробСТная сила .

ΠšΡ€ΡƒΠ³ΠΎΠ²ΠΎΠ΅ Π΄Π²ΠΈΠΆΠ΅Π½ΠΈΠ΅

Π€Π˜Π—Π˜ΠšΠ Автомобили ΠΈ ΠΈΡ… ΡΠΏΠΎΡΠΎΠ±Π½ΠΎΡΡ‚ΡŒ ΠΏΡ€Π΅ΠΎΠ΄ΠΎΠ»Π΅Π²Π°Ρ‚ΡŒ ΠΏΠΎΠ²ΠΎΡ€ΠΎΡ‚Ρ‹


Π’Π΅Ρ€ΠΌΠΈΠ½ “ΠΏΡ€ΠΎΡ…ΠΎΠΆΠ΄Π΅Π½ΠΈΠ΅ ΠΏΠΎΠ²ΠΎΡ€ΠΎΡ‚ΠΎΠ²” описываСт Π±Π΅Π· Π½ΠΎΠΌΠ΅Ρ€ΠΎΠ² ΡΠΏΠΎΡΠΎΠ±Π½ΠΎΡΡ‚ΡŒ автомобиля Π΄Π²ΠΈΠ³Π°Ρ‚ΡŒΡΡ быстро ΠΏΠΎ ΠΊΡ€ΠΈΠ²ΠΎΠΉ.Π’ Ρ„ΠΈΠ·ΠΈΠΊΡƒ плоской ΠΊΡ€ΠΈΠ²ΠΎΠΉ встроСно ΠΎΠ³Ρ€Π°Π½ΠΈΡ‡Π΅Π½ΠΈΠ΅ прохоТдСния ΠΏΠΎΠ²ΠΎΡ€ΠΎΡ‚ΠΎΠ². Ρ„Π°ΠΊΡ‚ΠΎΡ€. Π­Ρ‚ΠΎΡ‚ Ρ„Π°ΠΊΡ‚ΠΎΡ€ – Ρ†Π΅Π½Ρ‚Ρ€ΠΎΡΡ‚Ρ€Π΅ΠΌΠΈΡ‚Π΅Π»ΡŒΠ½ΠΎΠ΅ ускорСниС. Π¦Π΅Π½Ρ‚Ρ€ΠΎΡΡ‚Ρ€Π΅ΠΌΠΈΡ‚Π΅Π»ΡŒΠ½ΠΎΠ΅ ускорСниС ΠΌΠΎΠΆΠ½ΠΎ Π½Π°ΠΉΡ‚ΠΈ ΠΏΠΎ Ρ‚Π΅Π»Π΅Ρ„ΠΎΠ½Ρƒ


Π“Π΄Π΅ с Ρ€Π°Π²Π½ΠΎ Ρ†Π΅Π½Ρ‚Ρ€ΠΎΡΡ‚Ρ€Π΅ΠΌΠΈΡ‚Π΅Π»ΡŒΠ½ΠΎΠΌΡƒ ΡƒΡΠΊΠΎΡ€Π΅Π½ΠΈΡŽ, v Ρ€Π°Π²Π½ΠΎ скорости автомобиля Π²ΠΎΠΊΡ€ΡƒΠ³ кривая, Π° R прСдставляСт радиус ΠΊΡ€ΠΈΠ²ΠΎΠΉ. ΠšΡ€ΠΈΠ²Π°Ρ, которая ΠΊΠ°ΠΊ Π Π°Π·ΠΌΠ΅Ρ‰Π΅Π½ΠΎ ΠΎΠ³Ρ€Π°Π½ΠΈΡ‡Π΅Π½ΠΈΠ΅ скорости 25 ΠΌ / с ΠΈ ΠΈΠΌΠ΅Π΅Ρ‚ радиус 50 ΠΌ, Ρ‚Ρ€Π΅Π±ΡƒΠ΅Ρ‚ Ρ†Π΅Π½Ρ‚Ρ€ΠΎΡΡ‚Ρ€Π΅ΠΌΠΈΡ‚Π΅Π»ΡŒΠ½ΠΎΠ³ΠΎ ускорСния 6.5 ΠΌ / с 2 . Π’ пСрСсчСтС Π½Π° g это 0,66 g ( 6,5 / 9,8 = 0,66).
Π¦Π΅Π½Ρ‚Ρ€ΠΎΡΡ‚Ρ€Π΅ΠΌΠΈΡ‚Π΅Π»ΡŒΠ½Π°Ρ сила, нСобходимая Π°Π²Ρ‚ΠΎΠΌΠΎΠ±ΠΈΠ»ΡŽ, Ρ‡Ρ‚ΠΎΠ±Ρ‹ ΠΏΡ€ΠΎΠ΅Ρ…Π°Ρ‚ΡŒ ΠΏΠΎ ΠΏΠΎΠ»ΠΎΠ³ΠΎΠΌΡƒ ΡƒΠ³Π»Ρƒ. обСспСчиваСтся Ρ‚Ρ€Π΅Π½ΠΈΠ΅ΠΌ ΠΌΠ΅ΠΆΠ΄Ρƒ шинами автомобиля ΠΈ Π΄ΠΎΡ€ΠΎΠ³ΠΎΠΉ. Π­Ρ‚ΠΎ Π·Π½Π°Ρ‡Π΅Π½ΠΈΠ΅ описываСтся Π² Ρ‚Π°ΠΊΠΈΡ… ΠΆΡƒΡ€Π½Π°Π»Π°Ρ…, ΠΊΠ°ΠΊ Car ΠΈ Driver , ΠΊΠ°ΠΊ Π°Π²Ρ‚ΠΎΠΌΠΎΠ±ΠΈΠ»ΡŒ Π±ΠΎΠΊΠΎΠ²ΠΎΠ΅ ускорСниС . Π§Ρ‚ΠΎΠ±Ρ‹ Π½Π°ΠΉΡ‚ΠΈ ΠΏΠΎΠΏΠ΅Ρ€Π΅Ρ‡Π½ΠΎΠ΅ ускорСниС автомобиля, Π°Π²Ρ‚ΠΎΠΌΠΎΠ±ΠΈΠ»ΡŒ ΠΏΠΎΠ²ΠΎΡ€Π°Ρ‡ΠΈΠ²Π°Π΅Ρ‚ ΠΊΡ€ΡƒΠ³ Π·Π°Π΄Π°Π½Π½ΠΎΠ³ΠΎ радиуса. Π—Π°Ρ‚Π΅ΠΌ ΠΎΠ½ ΠΌΠ΅Π΄Π»Π΅Π½Π½ΠΎ ускоряСтся, ΠΏΠΎΠΊΠ° Π½Π΅ Π½Π°Ρ‡ΠΈΠ½Π°Π΅Ρ‚ ΡΠΎΡΠΊΠ°Π»ΡŒΠ·Ρ‹Π²Π°Ρ‚ΡŒ Π·Π° ΠΏΡ€Π΅Π΄Π΅Π»Ρ‹ ΠΊΡ€ΡƒΠ³Π°.Π­Ρ‚ΠΎ Ρ‚ΠΎΡ‚ ΠΌΠΎΠΌΠ΅Π½Ρ‚, ΠΊΠΎΠ³Π΄Π° Ρ‚Ρ€Π΅Π½ΠΈΠ΅ ΠΌΠ΅ΠΆΠ΄Ρƒ шинами ΠΈ Π΄ΠΎΡ€ΠΎΠ³ΠΎΠΉ ΠΎΠΊΠ°Π·Π°Π»ΠΎ максимальноС воздСйствиС трСния. МаксимальноС ΠΏΠΎΠΏΠ΅Ρ€Π΅Ρ‡Π½ΠΎΠ΅ ускорСниС автомобиля ΠΌΠΎΠΆΠ½ΠΎ Ρ€Π°ΡΡΡ‡ΠΈΡ‚Π°Ρ‚ΡŒ ΠΏΠΎ Ρ„ΠΎΡ€ΠΌΡƒΠ»Π΅ Ρ„ΠΎΡ€ΠΌΡƒΠ»Π° для Ρ†Π΅Π½Ρ‚Ρ€ΠΎΡΡ‚Ρ€Π΅ΠΌΠΈΡ‚Π΅Π»ΡŒΠ½ΠΎΠ³ΠΎ ускорСния, показанная Π²Ρ‹ΡˆΠ΅.
ΠšΡ€ΠΈΠ²Π°Ρ опрСдСляСт, ΠΊΠ°ΠΊΠΎΠ΅ Ρ†Π΅Π½Ρ‚Ρ€ΠΎΡΡ‚Ρ€Π΅ΠΌΠΈΡ‚Π΅Π»ΡŒΠ½ΠΎΠ΅ ускорСниС Π½Π΅ΠΎΠ±Ρ…ΠΎΠ΄ΠΈΠΌΠΎ для бСзопасного ΠΏΠ΅Ρ€Π΅ΠΌΠ΅Ρ‰Π°Ρ‚ΡŒΡΡ ΠΏΠΎ ΠΊΡ€ΠΈΠ²ΠΎΠΉ. Он опрСдСляСт это ΠΏΠΎ скорости двиТСния ΠΊΡ€ΠΈΠ²ΠΎΠΉ. ΠΏΡ€ΠΎΠΉΠ΄Π΅Π½Π½Ρ‹ΠΉ ΠΏΡƒΡ‚ΡŒ ΠΈ радиус ΠΊΡ€ΠΈΠ²ΠΎΠΉ. Π˜ΡΠΏΠΎΠ»ΡŒΠ·ΡƒΠΉΡ‚Π΅ ΡƒΡ€Π°Π²Π½Π΅Π½ΠΈΠ΅: V 2 / R.

Π’ ΠΏΡ€ΠΈΠ²Π΅Π΄Π΅Π½Π½ΠΎΠΌ Π²Ρ‹ΡˆΠ΅ ΠΏΡ€ΠΈΠΌΠ΅Ρ€Π΅ для ΠΊΡ€ΠΈΠ²ΠΎΠΉ трСбуСтся 0,66 Π³ Π±Π΅Π· скольТСниС. Если ΠΏΠΎΠΏΠ΅Ρ€Π΅Ρ‡Π½ΠΎΠ΅ ускорСниС автомобиля большС ΠΈΠ»ΠΈ Ρ€Π°Π²Π½ΠΎ расчСтноС Ρ†Π΅Π½Ρ‚Ρ€ΠΎΡΡ‚Ρ€Π΅ΠΌΠΈΡ‚Π΅Π»ΡŒΠ½ΠΎΠ΅ ускорСниС ΠΊΡ€ΠΈΠ²ΠΎΠΉ, Ρ‚ΠΎΠ³Π΄Π° ΠΎΠ½Π° Π±ΡƒΠ΄Π΅Ρ‚ Π²Ρ€Π°Ρ‰Π°Ρ‚ΡŒΡΡ Π²ΠΎΠΊΡ€ΡƒΠ³ ΠΏΠΎ ΠΊΡ€ΠΈΠ²ΠΎΠΉ Π±Π΅Π· ΠΏΡ€ΠΎΡΠΊΠ°Π»ΡŒΠ·Ρ‹Π²Π°Π½ΠΈΡ. Если Π±ΠΎΠΊΠΎΠ²ΠΎΠ΅ ускорСниС автомобиля мСньшС Ρ†Π΅Π½Ρ‚Ρ€ΠΎΡΡ‚Ρ€Π΅ΠΌΠΈΡ‚Π΅Π»ΡŒΠ½ΠΎΠ³ΠΎ ускорСния, Ρ‚ΠΎΠ³Π΄Π° Π°Π²Ρ‚ΠΎΠΌΠΎΠ±ΠΈΠ»ΡŒ Π²Ρ‹ΡΠΊΠΎΠ»ΡŒΠ·Π½Π΅Ρ‚ Π½Π°Ρ€ΡƒΠΆΡƒ ΠΊΡ€ΠΈΠ²ΠΎΠΉ Π΄ΠΎ Ρ‚Π΅Ρ… ΠΏΠΎΡ€, ΠΏΠΎΠΊΠ° радиус Π½Π΅ станСт достаточно большим, Ρ‡Ρ‚ΠΎΠ±Ρ‹ ΠΏΠΎΠ·Π²ΠΎΠ»ΠΈΡ‚ΡŒ ΠΊΡ€ΠΈΠ²Ρ‹ΠΌ Ρ†Π΅Π½Ρ‚Ρ€ΠΎΡΡ‚Ρ€Π΅ΠΌΠΈΡ‚Π΅Π»ΡŒΠ½Ρ‹ΠΌ ускорСниС равняСтся Π±ΠΎΠΊΠΎΠ²ΠΎΠΌΡƒ ΡƒΡΠΊΠΎΡ€Π΅Π½ΠΈΡŽ автомобиля.

ΠŸΠ Π˜ΠœΠ•Π  β„– 1
1. На ΠΊΡ€ΠΈΠ²ΠΎΠΉ ΡƒΠΊΠ°Π·Π°Π½ΠΎ ΠΎΠ³Ρ€Π°Π½ΠΈΡ‡Π΅Π½ΠΈΠ΅ скорости 25 миль Π² час, 11,16 ΠΌ / с. Радиус ΠΊΡ€ΠΈΠ²Ρ‹Ρ… – 40 ΠΌΠ΅Ρ‚Ρ€ΠΎΠ².
β€’ Π‘ ΠΊΠ°ΠΊΠΈΠΌ Ρ†Π΅Π½Ρ‚Ρ€ΠΎΡΡ‚Ρ€Π΅ΠΌΠΈΡ‚Π΅Π»ΡŒΠ½Ρ‹ΠΌ ускорСниСм Π°Π²Ρ‚ΠΎΠΌΠΎΠ±ΠΈΠ»ΡŒ ΠΌΠΎΠΆΠ΅Ρ‚ Π΄Π²ΠΈΠ³Π°Ρ‚ΡŒΡΡ ΠΏΠΎ ΠΊΡ€ΠΈΠ²ΠΎΠΉ Π±Π΅Π· скольТСния?

Π­Ρ‚ΠΎ ΠΎΠ·Π½Π°Ρ‡Π°Π΅Ρ‚, Ρ‡Ρ‚ΠΎ Π°Π²Ρ‚ΠΎΠΌΠΎΠ±ΠΈΠ»ΡŒ ΠΌΠΎΠΆΠ΅Ρ‚ Π΅Ρ…Π°Ρ‚ΡŒ ΠΏΠΎ ΠΊΡ€ΠΈΠ²ΠΎΠΉ Π±Π΅Π· ΠΏΡ€ΠΎΡΠΊΠ°Π»ΡŒΠ·Ρ‹Π²Π°Π½ΠΈΡ, Ссли ΡˆΠΈΠ½Ρ‹ ΡΠΎΠ·Π΄Π°ΡŽΡ‚ Ρ„Ρ€ΠΈΠΊΡ†ΠΈΠΎΠ½Π½Ρ‹Π΅ усилиС 3,11 ΠΌ / с 2 , 0,318 Π³. Π’ машинС ΠΈ Π£ водитСля эта сила трСния описываСтся ΠΊΠ°ΠΊ Β«Π±ΠΎΠΊΠΎΠ²ΠΎΠ΅ ускорСниС».” ΠžΠ³Ρ€Π°Π½ΠΈΡ‡ΠΈΠ²Π°ΡŽΡ‰ΠΈΠΉ Ρ„Π°ΠΊΡ‚ΠΎΡ€ для этой ΠΊΡ€ΠΈΠ²ΠΎΠΉ составляСт 0,317 Π³.

ΠŸΠ Π˜ΠœΠ•Π  β„– 2
ΠŸΡ€ΠΈ Π΄Π²ΠΈΠΆΠ΅Π½ΠΈΠΈ ΠΏΠΎ Π΄ΠΎΡ€ΠΎΠ³Π΅ Π½Π° высотС 25 ΠΌ / с, Π°Π²Ρ‚ΠΎΠΌΠΎΠ±ΠΈΠ»ΡŽ Π½Π΅ΠΎΠ±Ρ…ΠΎΠ΄ΠΈΠΌΠΎ ΠΏΡ€Π΅ΠΎΠ΄ΠΎΠ»Π΅Ρ‚ΡŒ ΠΏΠΎΠ²ΠΎΡ€ΠΎΡ‚ радиусом 19 ΠΌ. Какая Ρ†Π΅Π½Ρ‚Ρ€ΠΎΡΡ‚Ρ€Π΅ΠΌΠΈΡ‚Π΅Π»ΡŒΠ½Π°Ρ сила Π½Π΅ΠΎΠ±Ρ…ΠΎΠ΄ΠΈΠΌΠΎ ΡΠ΄Π΅Π»Π°Ρ‚ΡŒ ΠΊΡ€ΠΈΠ²ΡƒΡŽ ΠΈ Ρ‡Ρ‚ΠΎ Π±ΡƒΠ΄Π΅Ρ‚, Ссли Ρ‚Ρ€Π΅Π½ΠΈΠ΅ ΠΌΠ΅ΠΆΠ΄Ρƒ ΡˆΠΈΠ½Ρ‹ автомобиля ΠΈ Π΄ΠΎΡ€ΠΎΠ³ΠΈ Π½Π΅ Ρ€Π°Π²Π½Ρ‹ ΠΈΠ»ΠΈ Π½Π΅ ΠΏΡ€Π΅Π²Ρ‹ΡˆΠ°ΡŽΡ‚ Ρ†Π΅Π½Ρ‚Ρ€ΠΎΡΡ‚Ρ€Π΅ΠΌΠΈΡ‚Π΅Π»ΡŒΠ½Π°Ρ сила, нСобходимая для ΠΊΡ€ΠΈΠ²ΠΎΠΉ.
Π Π•Π¨Π•ΠΠ˜Π•



3.36 Π³! НСт Π»Π΅Π³ΠΊΠΎΠ²Ρ‹Π΅ Π°Π²Ρ‚ΠΎΠΌΠΎΠ±ΠΈΠ»ΠΈ, ΠΊΠΎΡ‚ΠΎΡ€Ρ‹Π΅ это Π΄Π΅Π»Π°ΡŽΡ‚. ΠΠ²Ρ‚ΠΎΠΌΠΎΠ±ΠΈΠ»ΡŒ Π½Π΅ сдСлаСт ΠΏΠΎΠ²ΠΎΡ€ΠΎΡ‚. Π’ΠΎΠ΄ΠΈΡ‚Π΅Π»ΡŽ Π½ΡƒΠΆΠ½ΠΎ Π±ΡƒΠ΄Π΅Ρ‚ ΠΏΠΎΠ²Π΅Ρ€Π½ΡƒΡ‚ΡŒ колСса, Π½ΠΎ машина ΠΏΡ€ΠΎΠ΄ΠΎΠ»ΠΆΠΈΡ‚ Π΄Π²ΠΈΠΆΠ΅Π½ΠΈΠ΅. Если Π±Ρ‹ ΠΏΠΎΠ²ΠΎΡ€ΠΎΡ‚ Π±Ρ‹Π» Π½Π°ΠΊΠ»ΠΎΠ½Π½Ρ‹ΠΌ, машина ΡΡŠΠ΅Ρ…Π°Π»Π° Π±Ρ‹ Π·Π° ΠΏΡ€Π΅Π΄Π΅Π»Ρ‹ ΠΏΠΎΠ²ΠΎΡ€ΠΎΡ‚Π°. Π΄ΠΎ Ρ‚Π΅Ρ… ΠΏΠΎΡ€, ΠΏΠΎΠΊΠ° Ρ†Π΅Π½Ρ‚Ρ€ΠΎΡΡ‚Ρ€Π΅ΠΌΠΈΡ‚Π΅Π»ΡŒΠ½ΠΎΠ΅ ускорСниС, Π½Π΅ΠΎΠ±Ρ…ΠΎΠ΄ΠΈΠΌΠΎΠ΅ для прСодолСния ΠΊΡ€ΠΈΠ²ΠΎΠΉ, Π½Π΅ станСт Ρ€Π°Π²Π½Ρ‹ΠΌ Ρ†Π΅Π½Ρ‚Ρ€ΠΎΡΡ‚Ρ€Π΅ΠΌΠΈΡ‚Π΅Π»ΡŒΠ½ΠΎΠ΅ ускорСниС, ΠΊΠΎΡ‚ΠΎΡ€ΠΎΠ΅ ΠΌΠΎΠΆΠ΅Ρ‚ Π²Ρ‹Π·Π²Π°Ρ‚ΡŒ Π°Π²Ρ‚ΠΎΠΌΠΎΠ±ΠΈΠ»ΡŒ. ΠŸΡ€Π΅Π΄ΠΏΠΎΠ»ΠΎΠΆΠΈΠΌ, машина ΠΌΠΎΠΆΠ΅Ρ‚ ΠΏΡ€ΠΎΡΠ²Π»ΡΡ‚ΡŒ Ρ†Π΅Π½Ρ‚Ρ€ΠΎΡΡ‚Ρ€Π΅ΠΌΠΈΡ‚Π΅Π»ΡŒΠ½ΠΎΠ΅ ускорСниС 0,88 g. ΠŸΠΎΡ‚ΠΎΠΌ;



Π‘Π»Π΅Π΄ΠΎΠ²Π°Ρ‚Π΅Π»ΡŒΠ½ΠΎ, R = 72.47ΠΌ. Π’ Π°Π²Ρ‚ΠΎΠΌΠΎΠ±ΠΈΠ»ΡŒ Π±ΡƒΠ΄Π΅Ρ‚ ΡΠΊΠΎΠ»ΡŒΠ·ΠΈΡ‚ΡŒ Π½Π°Ρ€ΡƒΠΆΡƒ ΠΏΠΎ ΠΊΡ€ΠΈΠ²ΠΎΠΉ с Π½Π°ΠΊΠ»ΠΎΠ½ΠΎΠΌ Π΄ΠΎ радиуса ΠΏΠΎΠ²ΠΎΡ€ΠΎΡ‚ Ρ€Π°Π²Π΅Π½ 72,47 ΠΌΠ΅Ρ‚Ρ€Π°. Или ΠΏΠΎΠΊΠ° Π·Π°Π±ΠΎΡ€ Π½Π΅ остановит ΠΌΠ°ΡˆΠΈΠ½Ρƒ.

НаТмитС здСсь для Π½Π΅ΠΊΠΎΡ‚ΠΎΡ€Ρ‹Ρ… физичСских ΠΏΡ€ΠΎΠ±Π»Π΅ΠΌ, связанных с автомобилями ΠΈ ΠΈΡ… ΡΠΏΠΎΡΠΎΠ±Π½ΠΎΡΡ‚ΡŒΡŽ ΠΏΡ€ΠΎΡ…ΠΎΠ΄ΠΈΡ‚ΡŒ ΠΏΠΎΠ²ΠΎΡ€ΠΎΡ‚Ρ‹. ΠŸΡ€ΠΈ Π½Π°ΠΆΠ°Ρ‚ΠΈΠΈ Π½Π° эту ссылку откроСтся Π½ΠΎΠ²ΠΎΠ΅ ΠΎΠΊΠ½ΠΎ. Π—Π°ΠΊΡ€ΠΎΠΉΡ‚Π΅, Ρ‡Ρ‚ΠΎΠ±Ρ‹ Π²Π΅Ρ€Π½ΡƒΡ‚ΡŒΡΡ Π½Π° эту страницу.

Π Π°Π²Π½ΠΎΠΌΠ΅Ρ€Π½ΠΎΠ΅ ΠΊΡ€ΡƒΠ³ΠΎΠ²ΠΎΠ΅ Π΄Π²ΠΈΠΆΠ΅Π½ΠΈΠ΅

Π Π°Π²Π½ΠΎΠΌΠ΅Ρ€Π½ΠΎΠ΅ ΠΊΡ€ΡƒΠ³ΠΎΠ²ΠΎΠ΅ Π΄Π²ΠΈΠΆΠ΅Π½ΠΈΠ΅

Π Π°Π²Π½ΠΎΠΌΠ΅Ρ€Π½ΠΎΠ΅ ΠΊΡ€ΡƒΠ³ΠΎΠ²ΠΎΠ΅ Π΄Π²ΠΈΠΆΠ΅Π½ΠΈΠ΅

ΠžΠ±ΡŠΠ΅ΠΊΡ‚, двиТущийся ΠΏΠΎ окруТности радиуса r с постоянной ΡΠΊΠΎΡ€ΠΎΡΡ‚ΡŒΡŽ v ускоряСтся.Π’ Π½Π°ΠΏΡ€Π°Π²Π»Π΅Π½ΠΈΠ΅ Π΅Π³ΠΎ Π²Π΅ΠΊΡ‚ΠΎΡ€Π° скорости всС врСмя мСняСтся, Π½ΠΎ Π²Π΅Π»ΠΈΡ‡ΠΈΠ½Π° Π²Π΅ΠΊΡ‚ΠΎΡ€ скорости остаСтся постоянным. Π’Π΅ΠΊΡ‚ΠΎΡ€ ускорСния Π½Π΅ ΠΌΠΎΠΆΠ΅Ρ‚ ΠΈΠΌΠ΅Ρ‚ΡŒ ΡΠΎΡΡ‚Π°Π²Π»ΡΡŽΡ‰Π΅ΠΉ Π² Π½Π°ΠΏΡ€Π°Π²Π»Π΅Π½ΠΈΠΈ Π²Π΅ΠΊΡ‚ΠΎΡ€Π° скорости, Ρ‚Π°ΠΊ ΠΊΠ°ΠΊ такая ΡΠΎΡΡ‚Π°Π²Π»ΡΡŽΡ‰Π°Ρ Π±ΡƒΠ΄Π΅Ρ‚ Π²Ρ‹Π·Π²Π°Ρ‚ΡŒ ΠΈΠ·ΠΌΠ΅Π½Π΅Π½ΠΈΠ΅ скорости. Π‘Π»Π΅Π΄ΠΎΠ²Π°Ρ‚Π΅Π»ΡŒΠ½ΠΎ, Π²Π΅ΠΊΡ‚ΠΎΡ€ ускорСния Π΄ΠΎΠ»ΠΆΠ΅Π½ Π±Ρ‹Ρ‚ΡŒ пСрпСндикулярно Π²Π΅ΠΊΡ‚ΠΎΡ€Ρƒ скорости Π² любой Ρ‚ΠΎΡ‡ΠΊΠ΅ окруТности. Π­Ρ‚ΠΎΡ‚ ускорСниС называСтся Ρ€Π°Π΄ΠΈΠ°Π»ΡŒΠ½ΠΎΠ΅ ускорСниС ΠΈΠ»ΠΈ Ρ†Π΅Π½Ρ‚Ρ€ΠΎΡΡ‚Ρ€Π΅ΠΌΠΈΡ‚Π΅Π»ΡŒΠ½ΠΎΠ³ΠΎ ускорСния , ΠΈ это ΡƒΠΊΠ°Π·Ρ‹Π²Π°Π΅Ρ‚ ΠΊ Ρ†Π΅Π½Ρ‚Ρ€Ρƒ ΠΊΡ€ΡƒΠ³Π°.Π’Π΅Π»ΠΈΡ‡ΠΈΠ½Π° Ρ†Π΅Π½Ρ‚Ρ€ΠΎΡΡ‚Ρ€Π΅ΠΌΠΈΡ‚Π΅Π»ΡŒΠ½ΠΎΠ³ΠΎ Π²Π΅ΠΊΡ‚ΠΎΡ€ ускорСния Ρ€Π°Π²Π΅Π½ c = v 2 / r. (ΠœΡ‹ пропускаСм Π²Ρ‹Π²ΠΎΠ΄ этого выраТСния.)

Бсылка: Π Π°Π²Π½ΠΎΠΌΠ΅Ρ€Π½ΠΎΠ΅ ΠΊΡ€ΡƒΠ³ΠΎΠ²ΠΎΠ΅ Π΄Π²ΠΈΠΆΠ΅Π½ΠΈΠ΅

ΠŸΡ€ΠΎΠ±Π»Π΅ΠΌΠ°:

ΠžΠΏΠΈΡˆΠΈΡ‚Π΅, ΠΊΠ°ΠΊ Π²ΠΎΠ΄ΠΈΡ‚Π΅Π»ΡŒ ΠΌΠΎΠΆΠ΅Ρ‚ ΡƒΠΏΡ€Π°Π²Π»ΡΡ‚ΡŒ Π°Π²Ρ‚ΠΎΠΌΠΎΠ±ΠΈΠ»Π΅ΠΌ, двиТущимся с постоянной ΡΠΊΠΎΡ€ΠΎΡΡ‚ΡŒΡŽ, Ρ‡Ρ‚ΠΎΠ±Ρ‹
(Π°) ускорСниС Ρ€Π°Π²Π½ΠΎ Π½ΡƒΠ»ΡŽ, ΠΈΠ»ΠΈ
(Π±) Π²Π΅Π»ΠΈΡ‡ΠΈΠ½Π° ускорСния остаСтся постоянной.

РСшСниС:

  • РассуТдСниС:
    (a) Π’ΠΎΠ΄ΠΈΡ‚Π΅Π»ΡŒ Π΄ΠΎΠ»ΠΆΠ΅Π½ Π·Π°ΡΡ‚Π°Π²ΠΈΡ‚ΡŒ ΠΌΠ°ΡˆΠΈΠ½Ρƒ Π΄Π²ΠΈΠ³Π°Ρ‚ΡŒΡΡ ΠΏΠΎ прямой линия.
    (b) Π’ΠΎΠ΄ΠΈΡ‚Π΅Π»ΡŒ Π΄ΠΎΠ»ΠΆΠ΅Π½ Π·Π°ΡΡ‚Π°Π²ΠΈΡ‚ΡŒ ΠΌΠ°ΡˆΠΈΠ½Ρƒ Π΄Π²ΠΈΠ³Π°Ρ‚ΡŒΡΡ ΠΏΠΎ ΠΊΡ€ΡƒΠ³Ρƒ. (ΠΈΠ»ΠΈ ΠΏΠΎ прямой с a = 0).
ΠŸΡ€ΠΎΠ±Π»Π΅ΠΌΠ°:

ΠžΡ€Π±ΠΈΡ‚Π° Π›ΡƒΠ½Ρ‹ Π²ΠΎΠΊΡ€ΡƒΠ³ ЗСмля ΠΏΡ€ΠΈΠ±Π»ΠΈΠ·ΠΈΡ‚Π΅Π»ΡŒΠ½ΠΎ круглая со срСдним радиусом 3,85 * 10 8 ΠΌ. Π›ΡƒΠ½Π° ΡΠΎΠ²Π΅Ρ€ΡˆΠ°Π΅Ρ‚ ΠΎΠ΄ΠΈΠ½ ΠΎΠ±ΠΎΡ€ΠΎΡ‚ Π²ΠΎΠΊΡ€ΡƒΠ³ Π›ΡƒΠ½Ρ‹ Π·Π° 27,3 дня. ЗСмля. Найти
(Π°) срСдняя ΠΎΡ€Π±ΠΈΡ‚Π°Π»ΡŒΠ½Π°Ρ ΡΠΊΠΎΡ€ΠΎΡΡ‚ΡŒ Π›ΡƒΠ½Ρ‹ ΠΈ
(Π±) Π΅Π³ΠΎ Ρ†Π΅Π½Ρ‚Ρ€ΠΎΡΡ‚Ρ€Π΅ΠΌΠΈΡ‚Π΅Π»ΡŒΠ½ΠΎΠ΅ ускорСниС.

РСшСниС:

  • РассуТдСниС:
    РасстояниС, ΠΊΠΎΡ‚ΠΎΡ€ΠΎΠ΅ Π»ΡƒΠ½Π° ΠΏΡ€ΠΎΡ…ΠΎΠ΄ΠΈΡ‚ Π·Π° ΠΎΠ΄ΠΈΠ½ ΠΎΡ€Π±ΠΈΡ‚Π°Π»ΡŒΠ½Ρ‹ΠΉ ΠΏΠ΅Ρ€ΠΈΠΎΠ΄ T, Ρ€Π°Π²Π½ΠΎ d = 2Ο€r.
    Π•Π³ΠΎ ΡΠΊΠΎΡ€ΠΎΡΡ‚ΡŒ v = расстояниС / врСмя = d / T.
    Π¦Π΅Π½Ρ‚Ρ€ΠΎΡΡ‚Ρ€Π΅ΠΌΠΈΡ‚Π΅Π»ΡŒΠ½ΠΎΠ΅ ускорСниС Π›ΡƒΠ½Ρ‹ v 2 / r.
  • Π”Π΅Ρ‚Π°Π»ΠΈ расчСта:
    (Π°) РасстояниС, ΠΊΠΎΡ‚ΠΎΡ€ΠΎΠ΅ Π»ΡƒΠ½Π° ΠΏΡ€ΠΎΡ…ΠΎΠ΄ΠΈΡ‚ Π·Π° 27,3 дня, Ρ€Π°Π²Π½ΠΎ d = 2Ο€r = 2,41 * 10 9. Π³.
    Π•Π³ΠΎ ΡΠΊΠΎΡ€ΠΎΡΡ‚ΡŒ v = d / (27,3 суток) = (d / (2,36 * 10 6 с)) = 1023 ΠΌ / с.
    (Π±) Π¦Π΅Π½Ρ‚Ρ€ΠΎΡΡ‚Ρ€Π΅ΠΌΠΈΡ‚Π΅Π»ΡŒΠ½ΠΎΠ΅ ускорСниС Π›ΡƒΠ½Ρ‹ v 2 / r = 2,725 * 10 -3 ΠΌ / с 2 .
ΠŸΡ€ΠΎΠ±Π»Π΅ΠΌΠ°:

Если Π²Ρ€Π°Ρ‰Π΅Π½ΠΈΠ΅ Π—Π΅ΠΌΠ»ΠΈ ΡƒΠ²Π΅Π»ΠΈΡ‡ΠΈΠ»ΠΎΡΡŒ Ρ‚Π°ΠΊ, Ρ‡Ρ‚ΠΎ Ρ†Π΅Π½Ρ‚Ρ€ΠΎΡΡ‚Ρ€Π΅ΠΌΠΈΡ‚Π΅Π»ΡŒΠ½ΠΎΠ΅ ускорСниС Π±Ρ‹Π»ΠΎ Ρ€Π°Π²Π½ΠΎ ΡƒΡΠΊΠΎΡ€Π΅Π½ΠΈΡŽ свободного падСния Π½Π° экваторС,
(Π°) ΠΊΠ°ΠΊΠΎΠ²Π° Π±ΡƒΠ΄Π΅Ρ‚ Ρ‚Π°Π½Π³Π΅Π½Ρ†ΠΈΠ°Π»ΡŒΠ½Π°Ρ ΡΠΊΠΎΡ€ΠΎΡΡ‚ΡŒ Ρ‡Π΅Π»ΠΎΠ²Π΅ΠΊΠ°, стоящСго Π½Π° экваторС, ΠΈ
Π±) ΠΊΠ°ΠΊ Π΄ΠΎΠ»Π³ΠΎ Π±ΡƒΠ΄Π΅Ρ‚ дСнь?

РСшСниС:

  • РассуТдСниС:
    Если Π±Ρ‹ Ρ†Π΅Π½Ρ‚Ρ€ΠΎΡΡ‚Ρ€Π΅ΠΌΠΈΡ‚Π΅Π»ΡŒΠ½ΠΎΠ΅ ускорСниС Π±Ρ‹Π»ΠΎ Ρ€Π°Π²Π½ΠΎ ΡƒΡΠΊΠΎΡ€Π΅Π½ΠΈΡŽ свободного падСния Π½Π° экваторС, Ρ‚ΠΎ v 2 / r = g.
  • Π”Π΅Ρ‚Π°Π»ΠΈ расчСта:
    (Π°) Радиус Π—Π΅ΠΌΠ»ΠΈ составляСт 6368 ΠΊΠΌ. Π₯ΠΎΡ‚ΠΈΠΌ v 2 / r = 9,8 ΠΌ / с 2 , ΠΈΠ»ΠΈ v 2 = (9,8 ΠΌ / с 2 ) * 6,368 * 10 6 ΠΌ, ΠΈΠ»ΠΈ v = 7900 ΠΌ / с.
    (b) ΠŸΡ€ΠΎΠ΄ΠΎΠ»ΠΆΠΈΡ‚Π΅Π»ΡŒΠ½ΠΎΡΡ‚ΡŒ дня Π±ΡƒΠ΄Π΅Ρ‚ t = 2Ο€r / v = 5065 s = 84,4 ΠΌΠΈΠ½.

НСравномСрноС ΠΊΡ€ΡƒΠ³ΠΎΠ²ΠΎΠ΅ Π΄Π²ΠΈΠΆΠ΅Π½ΠΈΠ΅

Π’Π΅Π»Π΅ΠΆΠΊΠ° амСриканских Π³ΠΎΡ€ΠΎΠΊ, двиТущаяся ΠΏΠΎ ΠΏΠ΅Ρ‚Π»Π΅, мСняСт ΡΠΊΠΎΡ€ΠΎΡΡ‚ΡŒ, Π° Ρ‚Π°ΠΊΠΆΠ΅ Π½Π°ΠΏΡ€Π°Π²Π»Π΅Π½ΠΈΠ΅ двиТСния.Π•Π³ΠΎ ускорСниС ΠΈΠΌΠ΅Π΅Ρ‚ Ρ€Π°Π΄ΠΈΠ°Π»ΡŒΠ½ΡƒΡŽ ΡΠΎΡΡ‚Π°Π²Π»ΡΡŽΡ‰ΡƒΡŽ ΠΈ Ρ‚Π°Π½Π³Π΅Π½Ρ†ΠΈΠ°Π»ΡŒΠ½ΡƒΡŽ ΡΠΎΡΡ‚Π°Π²Π»ΡΡŽΡ‰ΡƒΡŽ. ΠΊΠΎΠΌΠΏΠΎΠ½Π΅Π½Ρ‚, a = a r + Π° Ρ‚ . Π’ Ρ‚Π°Π½Π³Π΅Π½Ρ†ΠΈΠ°Π»ΡŒΠ½Π°Ρ ΡΠΎΡΡ‚Π°Π²Π»ΡΡŽΡ‰Π°Ρ Π²Ρ‹Π·Ρ‹Π²Π°Π΅Ρ‚ ΠΈΠ·ΠΌΠ΅Π½Π΅Π½ΠΈΠ΅ скорости, ΠΈ Π΅Π΅ Π²Π΅Π»ΠΈΡ‡ΠΈΠ½Π° опрСдСляСтся ΠΊΠ°ΠΊ Ρ‚. = dv / dt, Π° Ρ€Π°Π΄ΠΈΠ°Π»ΡŒΠ½Π°Ρ ΡΠΎΡΡ‚Π°Π²Π»ΡΡŽΡ‰Π°Ρ Π²Ρ‹Π·Ρ‹Π²Π°Π΅Ρ‚ ΠΈΠ·ΠΌΠ΅Π½Π΅Π½ΠΈΠ΅ направлСния ΠΈ Π΅Π³ΠΎ Π²Π΅Π»ΠΈΡ‡ΠΈΠ½Π° опрСдСляСтся ΠΊΠ°ΠΊ r = v 2 / r, Π³Π΄Π΅ r – радиус ΠΊΡ€ΠΈΠ²ΠΈΠ·Π½Π° Π² рассматриваСмой Ρ‚ΠΎΡ‡ΠΊΠ΅. ΠšΠΎΠΌΠΏΠΎΠ½Π΅Π½Ρ‚Ρ‹ , , , ΠΈ a t пСрпСндикулярны Π΄Ρ€ΡƒΠ³ Π΄Ρ€ΡƒΠ³Ρƒ ΠΈ Π²Π΅Π»ΠΈΡ‡ΠΈΠ½Π° a Ρ€Π°Π²Π½ΠΎ a = (a r 2 + a t 2 ) Β½ .

ΠŸΡ€ΠΎΠ±Π»Π΅ΠΌΠ°:

Ѐигуристка выполняСт Π²ΠΎΡΡŒΠΌΠ΅Ρ€ΠΊΡƒ, ΡΠΎΡΡ‚ΠΎΡΡ‰ΡƒΡŽ ΠΈΠ· Π΄Π²ΡƒΡ… Ρ€Π°Π²Π½Ρ‹Ρ… ΠΊΠ°ΡΠ°Ρ‚Π΅Π»ΡŒΠ½Ρ‹Ρ…. ΠΊΡ€ΡƒΠ³ΠΎΠ²Ρ‹Π΅ Π΄ΠΎΡ€ΠΎΠΆΠΊΠΈ. На протяТСнии ΠΏΠ΅Ρ€Π²ΠΎΠ³ΠΎ ΠΊΡ€ΡƒΠ³Π° ΠΎΠ½Π° Ρ€Π°Π²Π½ΠΎΠΌΠ΅Ρ€Π½ΠΎ ΡƒΠ²Π΅Π»ΠΈΡ‡ΠΈΠ²Π°Π΅Ρ‚ ΡΠΊΠΎΡ€ΠΎΡΡ‚ΡŒ, Π° Π²ΠΎ Π²Ρ‚ΠΎΡ€ΠΎΠΌ ΠΊΡ€ΡƒΠ³Π΅ ΠΎΠ½Π° двиТСтся с постоянной ΡΠΊΠΎΡ€ΠΎΡΡ‚ΡŒΡŽ. Π‘Π΄Π΅Π»Π°ΠΉΡ‚Π΅ набросок Π΅Π΅ Π²Π΅ΠΊΡ‚ΠΎΡ€ ускорСния Π² Π½Π΅ΡΠΊΠΎΠ»ΡŒΠΊΠΈΡ… Ρ‚ΠΎΡ‡ΠΊΠ°Ρ… ΠΏΠΎ ΠΏΡƒΡ‚ΠΈ двиТСния.

РСшСниС:

  • РассуТдСниС:
    На протяТСнии ΠΏΠ΅Ρ€Π²ΠΎΠ³ΠΎ ΠΊΠΎΠ½Ρ‚ΡƒΡ€Π° Π²Π΅Π»ΠΈΡ‡ΠΈΠ½Π° Ρ‚Π°Π½Π³Π΅Π½Ρ†ΠΈΠ°Π»ΡŒΠ½ΠΎΠ³ΠΎ ускорСния a t постоянно.Π’Π΅Π»ΠΈΡ‡ΠΈΠ½Π° Ρ€Π°Π΄ΠΈΠ°Π»ΡŒΠ½ΠΎΠ³ΠΎ ускорСния Π° r увСличиваСтся ΠΊΠ°ΠΊ v 2 / r, ΠΊΠΎΠ³Π΄Π° ΡΠΊΠΎΡ€ΠΎΡΡ‚ΡŒ ΠΊΠΎΠ½ΡŒΠΊΠΎΠ±Π΅ΠΆΡ†Π° увСличиваСтся ΠΊΠ°ΠΊ v = a t t = (2a t d) Β½ , Π³Π΄Π΅ d – ΠΏΡ€ΠΎΠΉΠ΄Π΅Π½Π½Ρ‹ΠΉ ΠΏΡƒΡ‚ΡŒ. Когда фигурист трогаСтся с мСста, Π²Π΅ΠΊΡ‚ΠΎΡ€ ускорСния Ρ€Π°Π²Π΅Π½ ΠΏΠΎ ΠΊΠ°ΡΠ°Ρ‚Π΅Π»ΡŒΠ½ΠΎΠΉ ΠΊ ΠΏΠ΅Ρ‚Π»Π΅. Но ΡƒΠ³ΠΎΠ» ΞΈ, ΠΊΠΎΡ‚ΠΎΡ€Ρ‹ΠΉ ΠΎΠ½ составляСт с ΠΊΠ°ΡΠ°Ρ‚Π΅Π»ΡŒΠ½ΠΎΠΉ увСличиваСтся ΠΊΠ°ΠΊ tanΞΈ = a r / a t = v 2 / (ra t ) = 2d / r. Π’ΠΎ Π²Ρ‚ΠΎΡ€ΠΎΠΌ Ρ†ΠΈΠΊΠ»Π΅ t = 0 ΠΈ r Ρ€Π°Π²Π½ΠΎ константа, Π° Π²Π΅ΠΊΡ‚ΠΎΡ€ ускорСния ΡƒΠΊΠ°Π·Ρ‹Π²Π°Π΅Ρ‚ Π½Π° Ρ†Π΅Π½Ρ‚Ρ€ ΠΊΠΎΠ½Ρ‚ΡƒΡ€Π°.
ΠŸΡ€ΠΎΠ±Π»Π΅ΠΌΠ°:

ΠΠ²Ρ‚ΠΎΠΌΠΎΠ±ΠΈΠ»ΡŒ, ΡΠΊΠΎΡ€ΠΎΡΡ‚ΡŒ ΠΊΠΎΡ‚ΠΎΡ€ΠΎΠ³ΠΎ увСличиваСтся со ΡΠΊΠΎΡ€ΠΎΡΡ‚ΡŒΡŽ 0,6 ΠΌ / с 2 , Π΅Π΄Π΅Ρ‚ ΠΏΠΎ ΠΊΡ€ΡƒΠ³ΠΎΠ²ΠΎΠΉ Π΄ΠΎΡ€ΠΎΠ³Π΅ радиусом 20 ΠΌ. Когда мгновСнная ΡΠΊΠΎΡ€ΠΎΡΡ‚ΡŒ машина 4 ΠΌ / с, Π½Π°ΠΉΡ‚ΠΈ
(Π°) ΠΊΠΎΠΌΠΏΠΎΠ½Π΅Π½Ρ‚ Ρ‚Π°Π½Π³Π΅Π½Ρ†ΠΈΠ°Π»ΡŒΠ½ΠΎΠ³ΠΎ ускорСния ΠΈ
(Π±) Π²Π΅Π»ΠΈΡ‡ΠΈΠ½Π° ΠΈ Π½Π°ΠΏΡ€Π°Π²Π»Π΅Π½ΠΈΠ΅ ΠΎΠ±Ρ‰Π΅Π³ΠΎ ускорСния.

РСшСниС:

  • РассуТдСниС:
    Π’Π°Π½Π³Π΅Π½Ρ†ΠΈΠ°Π»ΡŒΠ½ΠΎΠ΅ ускорСниС автомобиля ΠΈΠΌΠ΅Π΅Ρ‚ Π·Π°Π΄Π°Π½Π½ΡƒΡŽ ΠΏΠΎΡΡ‚ΠΎΡΠ½Π½ΡƒΡŽ Π²Π΅Π»ΠΈΡ‡ΠΈΠ½Ρƒ. Π•Π³ΠΎ Π½Π°ΠΏΡ€Π°Π²Π»Π΅Π½ΠΈΠ΅ ΠΊΠ°ΡΠ°Ρ‚Π΅Π»ΡŒΠ½ΠΎ окруТности.
    РадиальноС ускорСниС ΠΈΠΌΠ΅Π΅Ρ‚ Π²Π΅Π»ΠΈΡ‡ΠΈΠ½Ρƒ v 2 / r. Π•Π³ΠΎ Π½Π°ΠΏΡ€Π°Π²Π»Π΅Π½ΠΈΠ΅ пСрпСндикулярна ΠΊΠ°ΡΠ°Ρ‚Π΅Π»ΡŒΠ½ΠΎΠΌΡƒ ΡƒΡΠΊΠΎΡ€Π΅Π½ΠΈΡŽ ΠΈ ΡƒΠΊΠ°Π·Ρ‹Π²Π°Π΅Ρ‚ Π½Π° Ρ†Π΅Π½Ρ‚Ρ€ ΠΊΡ€ΡƒΠ³Π°.
  • Π”Π΅Ρ‚Π°Π»ΠΈ расчСта:
    (Π°) Π’Π°Π½Π³Π΅Π½Ρ†ΠΈΠ°Π»ΡŒΠ½ΠΎΠ΅ ускорСниС автомобиля ΠΈΠΌΠ΅Π΅Ρ‚ постоянная Π²Π΅Π»ΠΈΡ‡ΠΈΠ½Π° 0,6 ΠΌ / с 2 .
    (b) Когда v = 4 ΠΌ / с, Ρ€Π°Π΄ΠΈΠ°Π»ΡŒΠ½ΠΎΠ΅ ускорСниС ΠΈΠΌΠ΅Π΅Ρ‚ Π²Π΅Π»ΠΈΡ‡ΠΈΠ½Ρƒ a r = v 2 / r = (16 ΠΌ / с 2 ) / (20 ΠΌ) = 0.8 ΠΌ / с 2 , a = (a r 2 + a t 2 ) Β½ = 1 ΠΌ / с 2 .
    ΠŸΡƒΡΡ‚ΡŒ ΞΈ – ΡƒΠ³ΠΎΠ», ΠΊΠΎΡ‚ΠΎΡ€Ρ‹ΠΉ Π²Π΅ΠΊΡ‚ΠΎΡ€ ускорСния составляСт с Π»ΠΈΠ½ΠΈΠ΅ΠΉ, Π½Π°ΠΏΡ€Π°Π²Π»Π΅Π½Π½ΠΎΠΉ ΠΎΡ‚ ΠΏΠΎΠ»ΠΎΠΆΠ΅Π½ΠΈΠ΅ автомобиля ΠΏΠΎ Π½Π°ΠΏΡ€Π°Π²Π»Π΅Π½ΠΈΡŽ ΠΊ Ρ†Π΅Π½Ρ‚Ρ€Ρƒ ΠΊΡ€ΡƒΠ³Π°. Π’ΠΎΠ³Π΄Π° tanΞΈ = a t / a r = 0,75, ΞΈ = 36,9 o .
ΠŸΡ€ΠΎΠ±Π»Π΅ΠΌΠ°:

Π¦ΠΈΡ„Ρ€Π° справа прСдставляСт собой ΠΏΠΎΠ»Π½ΠΎΠ΅ ускорСниС двиТущСйся частицы. ΠΏΠΎ часовой стрСлкС ΠΏΠΎ ΠΊΡ€ΡƒΠ³Ρƒ радиуса 2.5 ΠΌ Π² Π΄Π°Π½Π½Ρ‹ΠΉ ΠΌΠΎΠΌΠ΅Π½Ρ‚ Π²Ρ€Π΅ΠΌΠ΅Π½ΠΈ. На это ΠΌΠ³Π½ΠΎΠ²Π΅Π½Π½ΠΎ, Π½Π°ΠΉΠ΄ΠΈ
Π°) Ρ†Π΅Π½Ρ‚Ρ€ΠΎΡΡ‚Ρ€Π΅ΠΌΠΈΡ‚Π΅Π»ΡŒΠ½ΠΎΠ΅ ускорСниС,
(Π±) ΡΠΊΠΎΡ€ΠΎΡΡ‚ΡŒ частицы, ΠΈ
(c) Π΅Π³ΠΎ Ρ‚Π°Π½Π³Π΅Π½Ρ†ΠΈΠ°Π»ΡŒΠ½ΠΎΠ΅ ускорСниС.

РСшСниС:

  • РассуТдСниС:
    Нам Π½ΡƒΠΆΠ½ΠΎ Π½Π°ΠΉΡ‚ΠΈ ΠΊΠΎΠΌΠΏΠΎΠ½Π΅Π½Ρ‚ Π²Π΅ΠΊΡ‚ΠΎΡ€Π° a .
  • Π”Π΅Ρ‚Π°Π»ΠΈ расчСта
    (а) a r = a cos (30 o ) = 12,99 м / с 2 .
    (Π±) Π° Ρ€ = v 2 / r, v 2 = ra r , v = 5.7 ΠΌ / с.
    (c) a t = a sin (30 o ) = 7,5 м / с 2 .

ΠœΠΎΠ΄ΡƒΠ»ΡŒ 3: Вопрос 2

ΠœΠΎΠΆΠ΅Ρ‚ Π»ΠΈ Ρ†Π΅Π½Ρ‚Ρ€ΠΎΡΡ‚Ρ€Π΅ΠΌΠΈΡ‚Π΅Π»ΡŒΠ½ΠΎΠ΅ ускорСниС ΠΈΠ·ΠΌΠ΅Π½ΠΈΡ‚ΡŒ ΡΠΊΠΎΡ€ΠΎΡΡ‚ΡŒ ΠΊΡ€ΡƒΠ³ΠΎΠ²ΠΎΠ³ΠΎ двиТСния?

ΠžΠ±ΡΡƒΠ΄ΠΈΡ‚Π΅ это со своими однокурсниками Π½Π° дискуссионном Ρ„ΠΎΡ€ΡƒΠΌΠ΅!


ΠžΠ±Ρ‰ΠΈΠ΅ Π΅Π΄ΠΈΠ½ΠΈΡ†Ρ‹ измСрСния ΡƒΠ³Π»ΠΎΠ²: градуса ΠΈ Ρ€Π°Π΄ΠΈΠ°Π½Π° . ΠœΡ‹ ΠΈΠΌΠ΅Π΅ΠΌ 360 o = 2Ο€ Ρ€Π°Π΄ΠΈΠ°Π½. Π”Π»ΠΈΠ½Π° окруТности радиуса r Ρ€Π°Π²Π½Π° 2Ο€r.Π”Π»ΠΈΠ½Π° Π΄ΡƒΠ³ΠΈ окруТности, ΡΠΎΡΡ‚Π°Π²Π»ΡΡŽΡ‰Π΅ΠΉ 180 o , составляСт Ο€r. Π”Π»ΠΈΠ½Π° Π΄ΡƒΠ³ΠΈ, содСрТащСй 90 o , составляСт (Ο€ / 2) r. ΠœΡ‹ Π²ΠΈΠ΄ΠΈΠΌ, Ρ‡Ρ‚ΠΎ Π΄Π»ΠΈΠ½Π° Π”ΡƒΠ³Π° окруТности, ΠΈΠΌΠ΅ΡŽΡ‰Π°Ρ ΡƒΠ³ΠΎΠ» ΞΈ, Ρ€Π°Π²Π½Π° ΞΈr, Π³Π΄Π΅ ΞΈ измСряСтся Π² Ρ€Π°Π΄ΠΈΠ°Π½Π°Ρ….

Для ΠΌΠ°Π»Ρ‹Ρ… ΡƒΠ³Π»ΠΎΠ² ΞΈ, sinΞΈ ~ ΞΈ, Ссли ΞΈ измСряСтся Π² Ρ€Π°Π΄ΠΈΠ°Π½Π°Ρ…. Π’Ρ‹ ΠΌΠΎΠΆΠ΅Ρ‚Π΅ ΡƒΠ±Π΅Π΄ΠΈΡ‚ΡŒΡΡ Π² этом, посмотрСв Π½Π° Ρ‚Π°Π±Π»ΠΈΡ†Ρƒ ΠΈΠ»ΠΈ Π³Ρ€Π°Ρ„ΠΈΠΊ.

ΡƒΠ³ΠΎΠ» (Π³Ρ€Π°Π΄.) ΡƒΠ³ΠΎΠ» (Ρ€Π°Π΄) sin (ΡƒΠ³ΠΎΠ»)
0 0 0
0.

ΠžΡΡ‚Π°Π²ΠΈΡ‚ΡŒ ΠΊΠΎΠΌΠΌΠ΅Π½Ρ‚Π°Ρ€ΠΈΠΉ